ശാസ്ത്രം

//ശാസ്ത്രം

'ഹൃദയം കൊണ്ട് ചിന്തിക്കുന്നില്ലേ' എന്ന് പല തവണ ചോദിക്കുന്ന ക്വുർആൻ വ്യക്തമായ അശാസ്ത്രീയതയല്ലേ പറയുന്നത്? യഥാർത്ഥത്തിൽ ചിന്തയും ഹൃദയവും തമ്മിൽ യാതൊരു ബന്ധവുമില്ലെന്നും മസ്തിഷ്‌കം മാത്രമാണ് ചിന്തയുടെ കേന്ദ്രമെന്നും പഠിപ്പിക്കുന്ന ആധുനിക വൈദ്യശാസ്ത്രത്തിന്റെ മുന്നിൽ ഈ ക്വുർആനിക പരാമർശങ്ങളെ എങ്ങനെ ന്യായീകരിക്കുവാൻ കഴിയും?

അബ്ദുൽ ലത്തീഫ് അഹ്മദ്

 

ഹൃദയത്തെക്കുറിച്ച് ക്വുർആൻ 110 തവണയെങ്കിലും പറയുന്നുണ്ട്. ചിന്തിക്കുവാനുള്ള കഴിവിനെ ഹൃദയവുമായി ബന്ധപ്പെടുത്തിയാണ് ക്വുർആൻ എല്ലായ്പ്പോഴും പരാമർശിക്കുന്നത്. ഏതാനും വചനങ്ങൾ നോക്കുക:

"ഇവര്‍ ഭൂമിയിലൂടെ സഞ്ചരിക്കുന്നില്ലേ? എങ്കില്‍ ചിന്തിച്ച്‌ മനസ്സിലാക്കാനുതകുന്ന ഹൃദയങ്ങളോ, കേട്ടറിയാനുതകുന്ന കാതുകളോ അവര്‍ക്കുണ്ടാകുമായിരുന്നു. തീര്‍ച്ചയായും കണ്ണുകളെയല്ല അന്ധത ബാധിക്കുന്നത്‌. പക്ഷെ, നെഞ്ചുകളിലുള്ള ഹൃദയങ്ങളെയാണ്‌ അന്ധത ബാധിക്കുന്നത്‌" (22: 46)

"നീ പറയുന്നത്‌ ശ്രദ്ധിച്ച്‌ കേള്‍ക്കുന്ന ചിലരും അവരുടെ കൂട്ടത്തിലുണ്ട്‌. എന്നാല്‍ അത്‌ അവര്‍ ഗ്രഹിക്കാത്ത വിധം അവരുടെ ഹൃദയങ്ങളിന്‍മേല്‍ നാം മൂടികള്‍ ഇടുകയും, അവരുടെ കാതുകളില്‍ അടപ്പ്‌ വെക്കുകയും ചെയ്തിരിക്കുന്നു. എന്തെല്ലാം ദൃഷ്ടാന്തങ്ങള്‍ കണ്ടാലും അവരതില്‍ വിശ്വസിക്കുകയില്ല. അങ്ങനെ അവര്‍ നിന്‍റെ അടുക്കല്‍ നിന്നോട്‌ തര്‍ക്കിക്കുവാനായി വന്നാല്‍ ആ സത്യനിഷേധികള്‍ പറയും; ഇത്‌ പൂര്‍വ്വികന്‍മാരുടെ കെട്ടുകഥകളല്ലാതെ മറ്റൊന്നുമല്ല എന്ന്‌." (6: 25)

"അല്ലാഹുവെപ്പറ്റി പറയപ്പെട്ടാല്‍ ഹൃദയങ്ങള്‍ പേടിച്ച്‌ നടുങ്ങുകയും, അവന്‍റെ ദൃഷ്ടാന്തങ്ങള്‍ വായിച്ചുകേള്‍പിക്കപ്പെട്ടാല്‍ വിശ്വാസം വര്‍ദ്ധിക്കുകയും, തങ്ങളുടെ രക്ഷിതാവിന്‍റെ മേല്‍ ഭരമേല്‍പിക്കുകയും ചെയ്യുന്നവര്‍ മാത്രമാണ്‌ സത്യവിശ്വാസികള്‍." (8: 2)

സ്വഹീഹായ നിരവധി ഹദീഥുകളിലും ചിന്തയെയും സന്മാർഗ - ദുർമാർഗങ്ങളുടെ സ്വീകരണത്തെയുമെല്ലാം ഹൃദയവുമായി ബന്ധപ്പെടുത്തി പരാമർശിച്ചത് കാണാൻ കഴിയും.

ശരീരത്തിന്റെ മൊത്തത്തിലുള്ള നിയന്ത്രണം നിർവഹിക്കുന്നത് മസ്തിഷ്കമാണെന്ന് ശരീരശാസ്ത്രം പഠിച്ചിട്ടുള്ളവർക്കറിയാം. കേന്ദ്രനാഡി വ്യവസ്ഥയാണ് ശരീരത്തെ മൊത്തത്തിൽ നിയന്ത്രിക്കുന്നതെങ്കിലും ഹൃദയത്തിന് അതിന്റേതായ ഒരു നിയന്ത്രണവ്യവസ്ഥയുണ്ട്. ഹൃദയനാഡീവ്യവസ്ഥയെന്നാണ് (cardiac nervous system) അതിനെ ശാസ്ത്രജ്ഞർ വിളിക്കാറുള്ളത്. സ്വന്തമായ നാഡികളും(neurons) നാഡീപ്രസാരകരും (neurotransmitters) പ്രോട്ടീനുകളും മറ്റു അനുബന്ധകോശങ്ങളുമുള്ള സ്വതന്ത്രമായ നാഡീവ്യവസ്ഥയാണിത്. ഈ നാഡീവ്യവസ്ഥക്ക് ഹൃദയമസ്തിഷ്‌കം (heart brain) എന്ന പേര് നൽകിയത് 1991ൽ പ്രസിദ്ധ കനേഡിയൻ നാഡീശാസ്ത്രജ്ഞനായ ഡോ: ആൻഡ്രു ആർമറാണ്. ഹൃദയവും തലച്ചോറും തമ്മിലുള്ള ബന്ധത്തെയും ഹൃദയത്തിന്റെ സ്വയംഭരണത്തെയും കുറിച്ച ഒരു പഠനശാഖ തന്നെ ഇന്നുണ്ട്. ഹൃദയനാഡീശാസ്ത്രം (neurocardiology) എന്നാണ് അതിന് പേര്. ഇവിഷയകമായ പഠനങ്ങളെ സമാഹരിച്ച് കൊണ്ട് ഡോ: ആൻഡ്രു ആർമറും ജെഫ്‌റി എൽ ആർഡറും കൂടി 1994 ൽ രചിച്ച പുസ്തകത്തിന്റെയും തലക്കെട്ട് 'ഹൃദയനാഡീശാസ്ത്രം' എന്ന് തന്നെയാണ്.

ഹൃദയം ചുരുങ്ങിയത് നാല് രൂപത്തിലെങ്കിലും മസ്തിഷ്കവുമായി ആശയവിനിമയം നടത്തുന്നുണ്ടെന്നാണ് ഹൃദയനാഡീശാസ്ത്രം പറയുന്നത്. നാഡീആവേഗങ്ങളിലൂടെയുള്ള (nerve impulses) നാഡീയവും ഹോർമോണുകളിലൂടെയും നാഡീപ്രസാരകരിലൂടെയുമുള്ള ജൈവരസതന്ത്രപരവും സമ്മർദ്ദതരംഗങ്ങളിലൂടെയുള്ള (pressure waves) ജൈവഭൗതികവും വിദ്യുത്കാന്തികക്ഷേത്രത്തിന്റെ (electromagnetic field) വ്യവഹാരങ്ങളിലൂടെയുള്ള ഉർജ്ജപരവുമായ ആശയക്കൈമാറ്റങ്ങൾ. ഈ ആശയക്കൈമാറ്റങ്ങളിലൂടെയാണ് ഹൃദയത്തിന് സ്വയംഭരണം സാധിക്കുന്നത്. ഈ സ്വയംഭരണത്തിൽ രക്തം പമ്പു ചെയ്യുകയെന്ന ഒരേയൊരു ധർമ്മം മാത്രമാണോ നിർവഹിക്കപ്പെടുന്നത്? ഈ ചോദ്യത്തിന് ഖണ്ഡിതമായ ഒരു ഉത്തരം നൽകാൻ ഇന്ന് നാഡീശാസ്ത്രം വളർന്നിട്ടില്ലെങ്കിലും 'അല്ല' എന്ന് പറയുന്നവരാണ് ആ രംഗത്തെ ഗവേഷകരിൽ പലരുമെന്നുള്ളതാണ് വസ്തുത.

പ്രസിദ്ധ ഡോക്യുമെന്ററി ഫിലിം നിർമ്മാതാവായ ഡേവിഡ് മാലോണിന്റെ 'ഹൃദയങ്ങളുടെയും മനസ്സുകളുടെയും' (Of Hearts and Minds) എന്ന ശാസ്ത്രഡോക്യൂമെന്ററിയിൽ ഇന്റർവ്യൂ ചെയ്ത പ്രഗത്ഭരായ പല നാഡീശാസ്ത്രവിദഗ്ധരും ഹൃദയശാസ്ത്രജ്ഞന്മാരും കരുതുന്നത് കേവലം രക്തം പമ്പു ചെയ്യുകയെന്ന ദൗത്യം മാത്രമല്ല ഹൃദയം നിർവഹിക്കുന്നത് എന്ന് തന്നെയാണ്. ഹൃദയത്തിൽ നിന്നുള്ള സിഗ്നലുകൾക്കനുസരിച്ചാണ് മസ്‌തിഷ്‌കത്തിനകത്തെ അമിഗ്ദാല ഭയം ഉത്പാദിപ്പിക്കുന്നതെന്ന കണ്ടെത്തൽ വികാരങ്ങളുടെ രൂപീകരണത്തിൽ ഹൃദയത്തിന് പങ്കുണ്ടെന്നു തന്നെയാണ് വ്യക്തമാക്കുന്നതെന്ന് അവർ അഭിപ്രായപ്പെടുന്നു. ഓക്സ്ഫോർഡ് സർവകലാശാലയിലെ പ്രൊഫസറായ ഡേവിഡ് പാറ്റേഴ്സണിന്റെ പഠനങ്ങൾ കാണിക്കുന്നത് വികാരങ്ങൾ സൃഷ്ടിക്കുന്നത് മസ്തിഷ്‌കം മാത്രമായിട്ടല്ല, ഹൃദയത്തിനു കൂടി അതിൽ പങ്കുണ്ടെന്നാണ്. ചുരുക്കത്തിൽ രക്തം പാമ്പു ചെയ്യുകയെന്ന ധർമ്മം മാത്രം നിർവഹിക്കുന്ന ഒരു ശാരീരികാവയവം മാത്രമാണ് ഹൃദയമെന്ന സങ്കൽപ്പമല്ല ഇന്ന് ശാസ്ത്രലോകത്തുള്ളത്. ചിന്തയുടെയും വികാരങ്ങളുടെയും നിർമാണത്തിൽ ഹൃദയം എങ്ങനെയൊക്കെയോ പങ്കു വഹിക്കുന്നുണ്ട്. എങ്ങനെയൊക്കെയാണെന്ന് പറയാൻ മാത്രം ഇന്ന് ശാസ്ത്രം വളർന്നിട്ടില്ലെന്ന് മാത്രമേയുള്ളൂ.

ഹൃദയനാഡീശാസ്ത്രം കൂടുതൽ വളരുമ്പോൾ, ക്വുർആനും ഹദീഥുകളും അർത്ഥശങ്കക്കിടയില്ലാത്തവണ്ണം വ്യക്തമാക്കുന്നത് പോലെ ഹൃദയം തന്നെയാണ് ചിന്തയുടെയും സത്യാസത്യവിവേചനത്തിന്റെയും കേന്ദ്രമെന്ന വസ്തുത ശാസ്ത്രലോകവും അംഗീകരിക്കുമെന്ന് നമുക്ക് പ്രത്യാശിക്കാം.

സൂര്യനെക്കുറിച്ചും ചന്ദ്രനെക്കുറിച്ചുമൊന്നും അറബികൾക്കുണ്ടായിരുന്നതിൽ കൂടുതലായ വിവരമൊന്നും ഖുർആൻ രചയിതാവിനുണ്ടായിരുന്നില്ലെന്ന് വ്യക്തമാക്കുന്ന നിരവധി വചനങ്ങൾ ഖുർആനിലുണ്ട്. അവയിലൊന്നാണ് 36 ആം അധ്യായത്തിലെ 39 ആം വചനം. ചന്ദ്രന്‍ ഈന്തപ്പനക്കുലയുടെ വളഞ്ഞ തണ്ടു പോലെയായിത്തീരുന്നുവെന്നാണ് ഈ വചനത്തിൽ പറഞ്ഞിരിക്കുന്നത്. യഥാർത്ഥത്തിൽ ചന്ദ്രൻ അങ്ങനെ ആയിത്തീരുന്നില്ലെന്നും സൂര്യപ്രകാശം ചന്ദ്രോപരിതലത്തിൽ പതിക്കുന്നത് ഒരു പ്രത്യേക രീതിയിലായതിനാൽ ഭൂമിയിലുള്ളവർക്ക് അങ്ങനെ തോന്നുക മാത്രമാണ് ചെയ്യുന്നതെന്നുമുള്ള വസ്തുത ഇന്ന് നമുക്കറിയാം. ദൈവീകമായിരുന്നു ഖുർആനെങ്കിൽ ഇത്തരം അബദ്ധ പരാമർശങ്ങൾ ഉണ്ടാകുമായിരുന്നുവോ?

- സുമേഷ് ചന്ദ്രൻ, കരുനാഗപ്പള്ളി

ന്ദ്രന്‍ ഈന്തപ്പനക്കുലയുടെ വളഞ്ഞ തണ്ടു പോലെയായിത്തീരുന്നുവെന്ന ഖുര്‍ആന്‍ സൂറത്ത് യാസീനിലെ മുപ്പത്തിയൊമ്പതാം വചനത്തിലെ പരാമര്‍ശം അശാസ്ത്രീയമാണെന്നാണ് വിമര്‍ശനം.

വിമര്‍ശിക്കപ്പെട്ട ഖുര്‍ആന്‍ വചനം ശ്രദ്ധിക്കുക. ''ചന്ദ്രന് നാം ചില ഘട്ടങ്ങള്‍ നിശ്ചയിച്ചിരിക്കുന്നു. അങ്ങനെ അത് പഴയ ഈന്തപ്പനക്കുലയുടെ വളഞ്ഞ തണ്ടു പോലെ ആയിത്തീരുന്നു.'' (36:39)

യഥാര്‍ഥത്തില്‍ ചന്ദ്രന്‍ ഈന്തപ്പനക്കുലയുടെ വളഞ്ഞ തണ്ടു പോലെയായിത്തീരുന്നി ല്ലെന്നും അങ്ങനെ ഭൂമിയിലുള്ള മനുഷ്യര്‍ക്ക് തോന്നുകയാണ് ചെയ്യുന്നതെന്നും അതുകൊണ്ടുതന്നെ ഈ ഖുര്‍ആന്‍ വചനം അശാസ്ത്രീയമാണ് എന്നുമാണ് വാദം. ഈ ഖുര്‍ആന്‍ വചനത്തെ സൂക്ഷ്മമായി അപഗ്രഥിച്ചാല്‍ ഈ വിമര്‍ശനം അടിസ്ഥാനരഹിതമാണെന്ന് ബോധ്യപ്പെടുമെന്നുള്ളതാണ് വസ്തുത.

മനുഷ്യര്‍ക്ക് അല്ലാഹു നല്‍കിയ അനുഗ്രഹങ്ങളെയും പ്രകൃതിയിലുള്ള ദൃഷ്ടാന്തങ്ങളെയും കുറിച്ച് വിവരിക്കുന്നതിനിടയിലാണ് സൂറത്തു യാസീനില്‍ ചന്ദ്രന് അല്ലാഹു കണക്കാക്കിയ ഘട്ടങ്ങളെ ക്കുറിച്ച് പരാമര്‍ശിക്കുന്നത്. 'ചന്ദ്രന്‍' എന്ന് പരിഭാഷപ്പെടുത്തിയിരിക്കുന്നത് 'ഖമര്‍' എന്ന അറബി പദത്തെയാണ്. ചന്ദ്രൻ പ്രകാശമാണെന്നാണ് ഖുര്‍ആനിലുടനീളം പറഞ്ഞിട്ടുള്ളത്. നൂര്‍, മുനീര്‍ എന്നിങ്ങനെ ചന്ദ്രനെ വിശേഷിപ്പിക്കുവാന്‍ ഖുര്‍ആന്‍ ഉപയോഗിച്ച പദങ്ങളെല്ലാം പ്രതിഫലിപ്പിക്കപ്പെടുന്ന പ്രകാശത്തെ ദ്യോതിപ്പിക്കുന്നവയാണ്. ഖമര്‍ (ചന്ദ്രന്‍) എന്നതുകൊണ്ടുള്ള വിവക്ഷ ആകാശത്തു നിലനില്‍ക്കുന്ന ഖരഗോളമെന്നതിലുപരിയായി ഭൂമിയിൽ നിന്ന് കാണുന്നതെന്താണോ അതാണെന്ന് ഇത് വ്യക്തമാക്കുന്നു.

ഖമറിന്റെ രൂപ വ്യത്യാസങ്ങള്‍ക്കനുസരിച്ച് ഹിലാല്‍ (ചന്ദ്രക്കല), ബദ്‌ർ (പൂര്‍ണ ചന്ദ്രന്‍) എന്നിങ്ങനെയുള്ള പ്രയോഗങ്ങളും അറബിയിലുണ്ട്. ചന്ദ്രന് ഘട്ടങ്ങളെ നിര്‍ണയിച്ചതായും അത് ഈന്തപ്പന ക്കുലയുടെ വളഞ്ഞ തണ്ടുപോലെ ആയിത്തീരുന്നതുമായുള്ള ഖുര്‍ആന്‍ പരാമര്‍ശങ്ങള്‍ ഭൂമിക്ക് ആപേക്ഷികമായി നടക്കുന്ന ചന്ദ്രപ്രതിഭാസങ്ങളെ കുറിക്കുന്നവയാണ്. ഖമര്‍ തന്നെയാണ് ബദ്‌ർ; ഖമര്‍ തന്നെയാണ് ഹിലാല്‍. ''ഖമര്‍ ഹിലാലായിത്തീരുന്നു''വെന്ന പരാമര്‍ശം ഒരു സാധാരണ അറബി പ്രയോഗമാണ്. ഹിലാലിന്റെ ഉപമാലങ്കാരമാണ് ''ചന്ദ്രന്‍ പഴയ ഈന്തപ്പനയുടെ വളഞ്ഞ തണ്ടുപോലെ'' ആയിത്തീരുകയെന്നത്. ''ഖമറിന്റെ ആദ്യഘട്ടമാണ് ഹിലാല്‍'' എന്ന പരാമര്‍ശം അശാസ്ത്രീയമല്ലാത്തതു പോലെത്തന്നെ ഈ ഉപമാലങ്കാരത്തിലും യാതൊരുവിധ അശാസ്ത്രീയതകളുമില്ല. ചന്ദ്രനിലെ ഖരപദാർത്ഥം വളഞ്ഞ ഈത്തപ്പനത്തണ്ട് പോലെയാകുന്നുവെന്നോ ഓരോ ചാന്ദ്രഘട്ടത്തിലും ചന്ദ്രൻ എന്ന ഖരവസ്തുവിനാണ് മാറ്റമുണ്ടാവുന്നതെന്നോ ഈ വചനത്തിൽ നിന്ന് സ്വഹാബിമാർ മുതൽ ഇന്ന് വരെയുള്ള വിശ്വാസികളൊന്നും മനസ്സിലാക്കിയിട്ടില്ല.

സൂര്യന്‍ ചെളിവെള്ളമുള്ള ഒരു ജലാശയത്തില്‍ മറഞ്ഞുപോവുന്നതായി ദുൽഖർനൈനിന്റെ കഥ പറയുമ്പോൾ ഖുര്‍ആൻ 18:86 ല്‍ പരാർശിക്കുന്നുണ്ട്. ഭൂമിയേക്കാള്‍ ലക്ഷക്കണക്കിന് ഇരട്ടി വലിപ്പമുള്ള സൂര്യന്‍ ഒരുജലാശയത്തില്‍ ആഴ്ന്നു പോവുകയെന്നു പറയുന്നത് വ്യക്തമായും അശാസ്ത്രീയമല്ലേ? ദുൽഖർനൈൻ സൂര്യോദയസ്ഥാനത്തും അസ്തമയസ്ഥാനത്തുമെല്ലാം എത്തിയതായും 18:86ലും 18:90ലും പറയുന്നുണ്ട്. ഇതെല്ലാം സൂര്യനെയും ഭൂമിയെയും ഉദയത്തിന്റെയും അസ്തമയത്തിന്റെയും പിന്നിലുള്ള ശാസ്ത്രത്തെയുമൊന്നും അറിയാത്ത ആരോ എഴുതിയതാണ് ക്വുർആൻ എന്നല്ലേ കാണിക്കുന്നത്?

- പ്രകാശൻ കെ.പി, അത്താണിക്കൽ, വള്ളിക്കുന്ന്

ദുൽഖർനൈനിന്റെ കഥ പറയുമ്പോഴുള്ള സൂര്യന്‍ ചെളിവെള്ളമുള്ള ഒരു ജലാശയത്തില്‍ മറഞ്ഞുപോവുന്നതായും സൂര്യോദയ സ്ഥാന ത്തും അസ്തമയസ്ഥാനത്തുമെല്ലാം അദ്ദേഹം എത്തിയതായുമുള്ള പരാമർശങ്ങൾ സൂര്യനെയും ഭൂമിയെയും ഉദയത്തിന്റെയും അസ്തമ യത്തിന്റെയും പിന്നിലുള്ള ശാസ്ത്രത്തെയുമൊന്നും അറിയാത്ത ആരോഎഴുതിയതാണ്ക്വുർആൻ എന്ന് വ്യക്തമാക്കുന്നതായാണ് വിമർശനം.

വിമര്‍ശിക്കപ്പെട്ട ഖുര്‍ആൻ വാക്യങ്ങള്‍ പരിശോധിക്കുക. ”അവര്‍ നിന്നോട് ദുല്‍ഖര്‍നൈനിയെപ്പറ്റി ചോദിക്കുന്നു. നീ പറയുക: അദ്ദേഹത്തെപ്പറ്റിയുള്ള വിവരം ഞാന്‍ നിങ്ങള്‍ക്ക് ഓതികേള്‍പിച്ച് തരാം. തീര്‍ച്ചയായും നാംഅദ്ദേഹത്തിന് ഭൂമിയില്‍ സ്വാധീനം നല്‍കുകയും, എല്ലാ കാര്യത്തിനുമുള്ള മാര്‍ഗം നാം അദ്ദേഹത്തിന് സൗകര്യ പ്പെടുത്തികൊടുക്കുകയും ചെയ്തു. അങ്ങനെ അദ്ദേഹം ഒരു മാര്‍ഗം പിന്തുടര്‍ന്നു. അങ്ങനെ അദ്ദേഹംസൂര്യാസ്തമനസ്ഥാനത്തെത്തിയപ്പോള്‍ അത് ചെളിവെള്ളമുള്ള ഒരു ജലാശയത്തില്‍ മറഞ്ഞ് പോകുന്നതായി അദ്ദേഹം കണ്ടു. അതിന്റെ അടുത്ത് ഒരു ജനവിഭാഗത്തെയും അദ്ദേഹം കണ്ടെത്തി. അദ്ദേഹത്തോട്) നാം പറഞ്ഞു: ഹേ, ദുല്‍ഖര്‍നൈൻ, ഒന്നുകില്‍ നിനക്ക് ഇവരെ ശിക്ഷിക്കാം. അല്ലെങ്കില്‍ നിനക്ക്അവരില്‍ നന്‍മയുണ്ടാക്കാം.” (18:83-86)

ഈ വചനത്തില്‍ സൂര്യന്‍ ചെളിവെള്ളത്തില്‍ ആഴ്ന്നു പോകുന്നുവെന്ന് പറഞ്ഞിട്ടില്ലെന്ന വസ്തുത ശ്രദ്ധിക്കുക. ദുല്‍ഖര്‍നൈനിയെ കുറി ച്ചും അദ്ദേഹത്തിന്റെ യാത്രകളെക്കുറിച്ചുമാണ് ഈ വചനങ്ങളിലെ പ്രതിപാദ്യം. അദ്ദേഹത്തിന്റെ യാത്രകള്‍ക്കിടയിൽ സൂര്യന്‍ അസ്ത മിക്കുന്ന സ്ഥലത്ത് എത്തിയപ്പോള്‍ ”സൂര്യന്‍ ചെളിവെള്ളമുള്ള ഒരു ജലാശയത്തില്‍ മറഞ്ഞു പോകുന്നതായി അദ്ദേഹം കണ്ടു” വെന്നാണ് ഖുര്‍ആൻ ഈ സൂക്തങ്ങളില്‍വ്യക്തമാക്കിയിട്ടുള്ളത്.

സൂര്യന്‍ ഉദിക്കുകയോ അസ്തമിക്കുകയോ ചെയ്യുന്നില്ല എന്ന് ഇന്ന് നമുക്കറിയാവുന്നതാണ്. ഭൂമിക്ക് ആപേക്ഷികമായി സൂര്യന്‍ നിശ്ചലാ വസ്ഥയിലാണെന്നും ഭൂമിയുടെ സ്വയംഭ്രമണം മൂലമാണ് സൂര്യന്‍ ഉദിക്കുന്നതുംഅസ്തമിക്കുന്നതുമായി നമുക്കനുഭവപ്പെടുന്നതെന്നുമുള്ള താണല്ലോ വസ്തുത. എന്നാല്‍ഭൂമിയില്‍ ജീവിക്കുന്ന ഓരോ മനുഷ്യരും സൂര്യോദയവും അസ്തമയവും അനുഭവിക്കുന്നുണ്ട്. ഭൂമിയിലു ള്ളവര്‍ക്ക് ആപേക്ഷികമായിസൂര്യന്‍ ഉദിക്കുകയും അസ്തമിക്കുകയും ചെയ്യുന്നുണ്ടെന്ന് സാരം. ഭൂമിയില്‍ ജീവിച്ചിരുന്ന ഒരാളായിരുന്ന ദുല്‍ഖര്‍നൈനിയും സൂര്യോദയവും സൂര്യാസ്തമയവും കണ്ടിട്ടുണ്ടാവണം. അദ്ദേഹത്തിന്റെ യാത്രയ്ക്കിടയിൽ സൂര്യാസ്തമയം നടക്കു ന്ന സ്ഥലത്തെത്തിയപ്പോള്‍ ചെളിവെള്ളമുള്ള ഒരു ജലാശയത്തിൽ സൂര്യൻ അസ്തമിക്കുന്നതായി അദ്ദേഹം കണ്ട കാര്യമാണ് ഖുര്‍ആനിൽ ഇവിടെ പരാമര്‍ശിച്ചിരിക്കുന്നത്.

‘ചെളിവെള്ളമുള്ള ജലാശയത്തില്‍ സൂര്യന്‍ മറഞ്ഞുപോയി’യെന്നത് ഖുര്‍ആനിന്റെ കേവല പരാമര്‍ശമല്ല, പ്രത്യുത ദൂര്‍ഖര്‍നൈനി കണ്ട കാര്യത്തിന്റെ പ്രതിപാദനം മാത്രമാണ്. ‘ഞാന്‍ ഇന്നലെ സൂര്യാസ്തമയ സമയത്ത്കോഴിക്കോട് കടപ്പുറത്ത് പോയപ്പോള്‍ സമുദ്രത്തില്‍ സൂര്യന്‍ മറഞ്ഞു പോകുന്നതായി കണ്ടു’വെന്ന പരാമര്‍ശത്തിൽ എന്തെങ്കിലും അശാസ്ത്രീയതയുണ്ടോ? ഇല്ലെങ്കില്‍ സൂചിപ്പിക്കപ്പെട്ട ഖുർആൻ വചനങ്ങളിലും യാതൊരുഅശാസ്ത്രീയതയുമില്ല.

ലിംഗനിര്‍ണയവുമായി ബന്ധപ്പെട്ട ക്വുര്‍ആന്‍ പരാമര്‍ശങ്ങള്‍ ശ്രദ്ധിക്കുക.

''ആണ്‍, പെണ്‍ എന്നീ രണ്ട് ഇണകളെ അവനാണ് സൃഷ്ടിച്ചതെന്നും.  ഒരു ബീജം സ്രവിക്കപ്പെടുമ്പോള്‍ അതില്‍ നിന്ന്.'' (53: 45-46) (1)

''പിന്നെ അവന്‍ ഒരു ഭ്രൂണമായി. എന്നിട്ട് അല്ലാഹു (അവനെ) സൃഷ്ടിച്ചു സംവിധാനിച്ചു. അങ്ങനെ അതില്‍ നിന്ന് ആണും പെണ്ണുമാകുന്ന രണ്ടു ഇണകളെ അവന്‍ ഉണ്ടാക്കി. അങ്ങനെയുള്ളവന്‍ മരിച്ചവരെ ജീവിപ്പിക്കാന്‍ കഴിവുള്ളവനല്ലെ?'' (75: 38-40)(2)

ഹദീഥുകളിലാണ് ലിംഗനിര്‍ണയത്തെപ്പറ്റി കുറേക്കൂടി വ്യക്തമായ പരാമര്‍ശമുള്ളത്.

  1. അനസില്‍ നിന്ന്: പ്രവാചകന്‍ മദീനയില്‍ വന്ന വിവരം അബ്ദുല്ലാഹിബ്‌നു സലാമിനു കിട്ടി. അദ്ദേഹം നബിയുടെ അടുത്തുവന്ന് പറഞ്ഞു: 'ഒരു പ്രവാചകനു മാത്രം അറിയാവുന്ന മൂന്നു കാര്യങ്ങള്‍ ഞാന്‍ താങ്കളോട് ചോദിക്കുകയാണ്..... ഇനി കുട്ടിക്ക് സാദൃശ്യം ലഭിക്കുന്ന കാര്യം; പുരുഷന്‍ സ്ത്രീയുമായി വേഴ്ച നടത്തുന്ന വേളയില്‍ അവന്റെ സ്രവം അവളുടെ സ്രവത്തെ അതിജയിച്ചാല്‍ കുട്ടിക്ക് സാദൃശ്യം അയാളോടായി. അവളുടെ സ്രവം അവന്റെ സ്രവത്തെയാണ് അതിജയിക്കുന്നതെങ്കില്‍ അവളോടും.' അബ്ദുല്ല പറഞ്ഞു: 'താങ്കള്‍ അല്ലാഹുവിന്റെ ദൂതനാണെന്നു ഞാന്‍ സാക്ഷ്യപ്പെടുത്തുന്നു.'(3)
  2. അനസ് ബ്‌നുമാലികി(റ)ല്‍ നിന്ന്: പുരുഷന് സ്വപ്‌നസ്ഖലനമുണ്ടാവുന്നതുപോലെ സ്ത്രീക്കും സ്വപ്‌നസ്ഖലനമുണ്ടായാല്‍ അവള്‍ എന്താണ് ചെേയ്യണ്ടത് എന്നതിനെ സംബന്ധിച്ച് ഉമ്മുസുലൈം പ്രവാചകനോട് ചോദിച്ചു. ...........നിശ്ചയമായും പുരുഷന്റെ‚ഇന്ദ്രിയം വെളുത്തതും കട്ടിയുള്ളതുമാണ്. സ്ത്രീയുടെ ഇന്ദ്രിയം മഞ്ഞനിറമുള്ളതും നേര്‍മയുള്ളതുമാണ്. ഏത് മുകളില്‍ വരുന്നുവോ അല്ലെങ്കില്‍ മുന്‍കടക്കുന്നുവോ അതിനോടാണ് കുട്ടിക്ക് സാദൃശ്യമുണ്ടാവുക.'(4)
  3. നബി (സ) സ്വാതന്ത്ര്യം നല്‍കിയ ഥൗബാനി(റ)ല്‍ നിന്ന്: ഞാന്‍ നബി(സ)യുടെ അടുക്കല്‍ നില്‍ക്കുമ്പോള്‍ƒജൂത പണ്ഡിതന്‍മാരില്‍ നിന്നുള്ള ഒരു പണ്ഡിതന്‍ വരികയും 'അസ്സലാമു അലൈക്ക യാ മുഹമ്മദ് (മുഹമ്മദ്, നിനക്ക് സമാധാനമുണ്ടാകട്ടെ)' എന്ന് പറയുകയും ചെയ്തു. ............ അയാള്‍ തുടര്‍ന്നു പറഞ്ഞു: 'ഭൂനിവാസികളില്‍നിന്നും ഒരു പ്രവാചകനോ അല്ലെങ്കില്‍ ഒന്നോ രണ്ടോ ആളുകള്‍ക്കോ അല്ലാതെ മറ്റൊരാക്കും അറിയാത്ത ഒരു കാര്യത്തെ സംബന്ധിച്ച് ചോദിക്കുവാനാണ് ഞാന്‍ വന്നിട്ടുള്ളത്.' നബി (സ) ചോദിച്ചു: 'ഞാനത് പറഞ്ഞാ ല്‍ നിനക്കത് ഉപകരിക്കുമോ?'. 'ഞാന്‍ എന്റ ചെവികള്‍ കൊണ്ട് കേള്‍ക്കും'. അയാള്‍ പറഞ്ഞു: '(പ്രസവിക്കപ്പെടുന്ന) ശിശുവിനെക്കു റിച്ച് ചോദിക്കുവാനാണ് ഞാന്‍ വന്നത്' നബി (സ) പറഞ്ഞു: 'പുരുഷന്റെ‚ ഇന്ദ്രിയം വെളുത്ത നിറത്തിലുളളതും സ്ത്രീയുടെ ഇന്ദ്രിയം മഞ്ഞനിറത്തിലുള്ളതുമാണ്. അത് രണ്ടും ഒരുമിച്ച് ചേരുകയും പുരുഷ ഇന്ദ്രിയം സ്ത്രീ ഇന്ദ്രിയത്തെ അതിജയിക്കുകയും ചെയ്താല്‍ അല്ലാഹുവിന്റെ അനുമതിയോടെ അത് ആണ്‍ കുട്ടിയായിതീരുന്നു. സ്ത്രീയുടെ ഇന്ദ്രിയം പുരുഷ ഇന്ദ്രിയത്തെ അതിജയിച്ചാല്‍ അല്ലാഹു വിന്റെ‚അനുമതിയോടെ അത് പെണ്‍കുട്ടിയായി തീരുന്നു.' ജൂതന്‍ പറഞ്ഞു: 'തീര്‍ച്ചയായും താങ്കള്‍ പറഞ്ഞത് സത്യമാണ്. തീര്‍ച്ചയായും താങ്കള്‍ ഒരു പ്രവാചകന്‍ തന്നെയാണ്'. പിന്നെ അയാള്‍ തിരിച്ചുപോയി. അപ്പോള്‍ നബി (സ) പറഞ്ഞു: 'അയാള്‍ എന്നോടു ചോദിച്ച കാര്യങ്ങളെക്കുറിച്ചൊന്നും അല്ലാഹു അറിയിച്ചുതരുന്നതുവരെ എനിക്ക് യാതൊരു വിവരവും ഉണ്ടായിരുന്നില്ല.'(5)
  4. ഹുദൈഫത്ത് ബ്‌നുഅസീദി(റ)ണ്‍ നിന്ന്: നബി (സ) പറഞ്ഞു: 'ഗര്‍ഭാശയത്തിണ്‍ ബീജം നാല്‍പത് ദിവസം അല്ലെങ്കില്‍ നാല്‍പത്തഞ്ച് ദിവസം ആയിത്തീരുമ്പോള്‍ അതിന്‍മേല്‍ ഒരു മലക്ക് പ്രവേശിക്കും. എന്നിട്ടവന്‍ ചോദിക്കും: രക്ഷിതാവേ, ദൗര്‍ഭാഗ്യവാനോ അതോ സൗഭാഗ്യവാനോ? എന്നിട്ട് അത് രേഖപ്പെടുത്തും. പിന്നെ ചോദിക്കും: രക്ഷിതാവേ, ആണോ അതോ പെണ്ണോ? എന്നിട്ട് അതും രേഖപ്പെ ടുത്തും. അവന്റെ കര്‍മവും അവന്റെ‚ ഫലവും, അവന്റെ‚അവധിയും, അവന്റെ‚ ഉപജീവനവും എഴുതപ്പെടും. പിന്നീട് ഏടുകള്‍ ചുരുട്ടപ്പെടും. അതില്‍ ഒന്നും വര്‍ദ്ധിപ്പിക്കപ്പെടുകയില്ല; ഒന്നും ചുരുട്ടപ്പെടുകയുമില്ല.'(6)
  5. അബ്ദാഹി ബ്‌നുമസ്ഊദി(റ)ല്‍ നിന്ന്: നബി (സ) പറയുന്നത് ഞാന്‍ കേട്ടു: 'ബീജത്തിന്‍മേല്‍ നാല്‍പത്തിരണ്ട് ദിവസം കഴിഞ്ഞാല്‍ അല്ലാഹു ഒരു മലക്കിനെ നിയോഗിക്കും. എന്നിട്ട് അവന്‍ അതിനെ രൂപപ്പെടുത്തുകയും, അതിന് കേള്‍വിയും കാഴ്ചയും ചര്‍മവും മാംസവും അസ്ഥിയും രൂപപ്പൈടുത്തുകയും ചെയ്യും. പിന്നീട് ആ മലക്ക് ചോദിക്കും: രക്ഷിതാവേ, ആണോ അതോ പെണ്ണോ? അപ്പോള്‍ നിന്റെ രക്ഷിതാവ് അവന്‍ ഉദ്ദേശിക്കുന്നത് വിധിക്കും. മലക്ക് അത് രേഖപ്പെടുത്തും. പിന്നീട് മലക്ക് ചോദിക്കും: രക്ഷിതാവേ ഇവന്റെ അവധി? അപ്പോള്‍ നിന്റെ രക്ഷിതാവ് അവന്‍ ഉദ്ദേശിച്ചത് പറയുകയും മലക്ക് അത് രേഖപ്പെടുത്തുകയും ചെയ്യും. പിന്നെ മലക്ക് ചോദി ക്കും: രക്ഷിതാവേ, ഇവന്റെ ഉപജീവനം? അപ്പോള്‍ നിന്റെ രക്ഷിതാവ് അവന്‍ ഉദ്ദേശിച്ചത് വിധിക്കുകയും മലക്ക് അത് രേഖപ്പെടുത്തു കയും ചെയ്യും. പിന്നീട് മലക്ക് തന്റെ‚കയ്യില്‍ ആ ഏടുമായി പോകും. കല്‍പിക്കപ്പെട്ടതിനേക്കാള്‍ വര്‍ദ്ധിപ്പിക്കുകയോ ചുരുക്കുകയോ ഇല്ല.'(7)
  6. അനസ് ബ്‌നുമാലികില്‍ (റ) നിന്ന്: നബി (സ) പറഞ്ഞു: 'പ്രതാപവാനും മഹാനുമായ അല്ലാഹു ഗര്‍ഭാശയത്തിന്റെ കാര്യം ഒരു മലക്കിനെ ഏല്‍പിച്ചിട്ടുണ്ട്. ആ മലക്ക് പറയും: രക്ഷിതാവേ, ബീജമാണ്. രക്ഷിതാവേ സിക്താണ്ഡമാണ്. രക്ഷിതാവേ മാംസപിണ്ഡമാണ്. അല്ലാഹു ഒരു സൃഷ്ടിയില്‍ വിധിക്കാന്‍ ഉദ്ദേശിച്ചാല്‍ മലക്ക് പറയും: രക്ഷിതാവേ, ആണോ പെണ്ണോ? ദൗര്‍ഭാഗ്യവാനോ അതോ സൗഭാഗ്യവാനോ? ഉപജീവനം എങ്ങനെയാണ്? അവധി എത്രയാണ്? അങ്ങനെ അവയെല്ലാം തന്റെ മാതാവിന്റെ വയറ്റിലായിരിക്കെ തന്നെ രേഖപ്പെടുത്ത പ്പെടും.(8)
  7. (നബി(സ)യോട് ചോദിക്കപ്പെട്ടു:) സ്വപ്‌നസ്ഖലനമുണ്ടായാല്‍ സ്ത്രീ കുളിക്കേണ്ടതുണ്ടോ? അപ്പോള്‍ നബി(സ) പറഞ്ഞു: 'അതെ; അവള്‍ ഇന്ദ്രിയം കണ്ടാല്‍'. അപ്പോള്‍ ഉമ്മുസുലൈം (റ) ചോദിച്ചു: 'സ്ത്രീക്ക് സ്ഖലനമുണ്ടാകുമോ?' അപ്പോള്‍ അദ്ദേഹം പറഞ്ഞു: 'എന്തൊരു കഷ്ടം! പിന്നെ? എങ്ങനെയാണ് കുട്ടിക്ക് അവളോട് സാദൃശ്യം ഉണ്ടാകുന്നത്?' മറ്റൊരു നിവേദനത്തില്‍ ആഇശ (റ) ഉമ്മുസുലൈം(റ)യോട് 'ഛെ! സ്ത്രീക്ക് അതുണ്ടാകുമോ?' എന്ന് ചോദിച്ചുവെന്നാണുള്ളത്. മറ്റൊരു റിപ്പോര്‍ട്ടില്‍, ഈ ഹദീഥിന്റെ‚അവസാന ഭാഗത്ത് ഇങ്ങനെ കൂടിയുണ്ട്. 'ഇന്ദ്രിയം കാരണമായിട്ടാണ് കുട്ടിക്ക് സാദൃശ്യമുണ്ടാകുന്നത്. സ്ത്രീയുടെ ഇന്ദ്രിയം പുരുഷന്റെ ഇന്ദ്രിയത്തിന് മുകളില്‍ വന്നാല്‍ കുട്ടിക്ക് മാതൃ സഹോദരന്‍മാരോട് സാദൃശ്യമുണ്ടാകും. പുരുഷന്റെ‚ ഇന്ദ്രിയം സ്ത്രീയുടെ ഇന്ദ്രിയത്തിന് മുകളില്‍ വന്നാല്‍ കുട്ടിക്ക് അവന്റെ പിതൃവ്യന്‍മാരോട് സാദൃശ്യമുണ്ടാകും.'(9)

മുകളില്‍ പറഞ്ഞ അല്ലാഹുവിന്റെയും അവന്റെ ദൂതന്റെയും വചനങ്ങളിലൊന്നും തന്നെ ആശാസ്ത്രീയമായ പരാമര്‍ശങ്ങളൊന്നുമില്ല. ലിംഗനിര്‍ണയത്തെപ്പറ്റിയുള്ള ഏറ്റവും പുതിയ വിവരങ്ങളുമായിപ്പോലും അവ പൂര്‍ണമായും യോജിച്ചു വരുന്നുവെന്നത് അത്ഭുതകരം തന്നെയാണ്.

  1. സ്രവിക്കപ്പെടുന്ന ശുക്ലത്തില്‍ നിന്നാണ് ആണും പെണ്ണുമുണ്ടാകുന്നതെന്ന് സൂറത്തുല്‍ ഖിയാമയിലെ 38 മുതല്‍ 40 വരെയുള്ള വചനങ്ങ ളില്‍ പറയുന്നു. ശുക്ലദ്രാവകത്തിലെ പുരുഷബീജം X ക്രോമസോം വഹിക്കുന്നതാണെങ്കില്‍ അത് അണ്ഡവുമായി ചേര്‍ന്നാല്‍ പെണ്‍കു ഞ്ഞും Y ക്രോമസോം വഹിക്കുന്നതാണെങ്കില്‍ അത് അണ്ഡവുമായി ചേര്‍ന്നാല്‍ ആണ്‍കുഞ്ഞുമുണ്ടാകുന്നു. ശുക്ലദ്രാവകമാണ് കുഞ്ഞ് ആണോ പെണ്ണോ എന്നു തീരുമാനിക്കുന്നത് എന്നര്‍ത്ഥം.
  2. സ്രവിക്കപ്പെടുന്ന ബീജത്തില്‍ തന്നെ ആണ്‍, പെണ്‍ എന്നിവയുണ്ടെന്നും അതാണ് ആണ്‍-പെണ്‍ ഇണകളുടെ ഉല്‍പത്തിക്ക് കാരണമാകുന്ന തെന്നും സൂറത്തുന്നജ്മിലെ 45,46 വചനങ്ങള്‍ വ്യക്തമാക്കുന്നു. സ്രവിക്കപ്പെടുന്ന ബീജത്തില്‍ തന്നെ ആണ്‍ ക്രോമസോമായ Yയെ വഹി ക്കുന്ന ബീജാണുക്കളും പെണ്‍ക്രോമസോമായ Xനെ വഹിക്കുന്ന ബീജാണുക്കുളുമുണ്ട്. ബീജദ്രാവകത്തിലെ Y ആണ്‍ബീജം അണ്ഡവു മായി ചേര്‍ന്നാല്‍ ആണ്‍കുട്ടിയും X പെണ്‍ബീജമാണ് അണ്ഡവുമായി ചേരുന്നതെങ്കില്‍ പെണ്‍കുട്ടിയുമാണുണ്ടാവുക.
  3. അനസില്‍ നിന്ന് ബുഖാരി നിവേദനം ചെയ്ത അബ്ദുല്ലാഹിബ്‌നു സലാമുമായി പ്രവാചകന്‍ (സ) നടത്തിയ സംഭാഷണത്തെക്കുറിച്ച് വിവരിക്കുന്ന ഹദീഥിലും അദ്ദേഹത്തില്‍ നിന്നുതന്നെ മുസ്‌ലിം നിവേദനം ചെയ്ത സ്വപ്നസ്ഖലനത്തെക്കുറിച്ച ഹദീഥിലും ഥൗബാ നി(റ)ല്‍ നിന്ന് മുസ്‌ലിം നിവേദനം ചെയ്ത ജൂതപുരോഹിതനു നല്‍കിയ മറുപടികയെക്കുറിച്ച് വിശദീകരിക്കുന്ന ഹദീഥിലും പുരുഷ ബീജം സ്ത്രീബീജത്തെ അതിജയിച്ചാല്‍ ആണ്‍കുഞ്ഞും, സ്ത്രീബീജം പുരുഷബീജത്തെയാണ് അതിജയിക്കുന്നതെങ്കില്‍ പെണ്‍കുട്ടിയുമാ ണുണ്ടാവുകയെന്ന് പ്രവാചകന്‍ (സ) പറഞ്ഞതായി ഉദ്ധരിച്ചിരിക്കുന്നു. ഈ പരാമര്‍ശത്തെ സുരതക്രിയയില്‍ പുരുഷനാണ് ആദ്യം സ്ഖലിക്കുന്നതെങ്കില്‍ ആണ്‍കുട്ടിയും സ്ത്രീക്കാണ് ആദ്യം സ്ഖലിക്കുകയെങ്കില്‍ പെണ്‍കുട്ടിയുമാണുണ്ടാവുകയെന്നാണ് പല പണ്ഡിത ന്‍മാരും വ്യാഖ്യാനിച്ചിരിക്കുന്നത്. രതിമൂര്‍ച്ചയോടനുബന്ധിച്ച് ചില സ്ത്രീകള്‍ക്ക് പാരായൂറിത്രല്‍ നാളിയില്‍ നിന്ന് പുറത്തേക്കുവ രുന്ന ദ്രാവകത്തിന് കുഞ്ഞിന്റെ ജനനത്തില്‍ യാതൊരു പങ്കുമില്ല എന്ന് ഇന്ന് നമുക്കറിയാം. പെണ്ണിന്റെ സ്ഖലനത്തിന് കുഞ്ഞിന്റെ ഉല്‍പത്തി പ്രക്രിയയില്‍ യാതൊരു പങ്കും വഹിക്കുവാനില്ലെങ്കില്‍ അതോടനുബന്ധിച്ചുണ്ടാകുന്ന ദ്രാവകം ആദ്യമോ പിന്നെയോ ഉണ്ടാകുന്നതെന്നത് ലിംഗനിര്‍ണയത്തെ ബാധിക്കുവാന്‍ സാധ്യതയൊന്നുമില്ല. ഈ ഹദീഥുകളില്‍ ബീജത്തിന്റെ അധീശത്വത്തെക്കുറിക്കു വാന്‍ പ്രയോഗിച്ചിരിക്കുന്നത് 'സബഖ'യെന്നും 'അലാ' എന്നുമുള്ള ക്രിയകളാണ്. ഒന്നിനുമേല്‍ മറ്റൊന്ന് മുന്‍കടക്കുന്നതിനോ ആദ്യമാകു ന്നതിനോ വിജയിക്കുന്നതിനോ അധികാരം സ്ഥാപിക്കുന്നതിനോ ആണ് 'സബഖ'യെന്നു പറയുകയെന്ന് അംഗീകൃത ഭാഷാ നിഘണ്ടുക്കള്‍ പരിശോധിച്ചാല്‍ വ്യക്തമാകും(10).

ഒന്നിനുമുകളില്‍ മറ്റൊന്ന് ആധിപത്യം സ്ഥാപിക്കുന്നതിനാണ് 'അലാ'യെന്ന് പ്രയോഗിക്കുകയെന്ന് ക്വുര്‍ആനില്‍നിന്നു തന്നെ വ്യക്തമാകു ന്നുണ്ട്. സൂറത്തുല്‍ മുഅ്മിനൂനിലെ 91-ാം വചനം നോക്കുക.

''അല്ലാഹു യാതൊരു സന്താനത്തെയും സ്വീകരിച്ചിട്ടില്ല. അവനോടൊപ്പം യാതൊരു ദൈവവുമുണ്ടായിട്ടില്ല. അങ്ങനെയായിരുന്നുവെങ്കില്‍ ഓരോ ദൈവവും താന്‍ സൃഷ്ടിച്ചതുമായി പോയിക്കളയുകയും, അവരില്‍ ചിലര്‍ ചിലരെ അടിച്ചമര്‍ത്തുകയും ചെയ്യുമായിരുന്നു. അവര്‍ പറഞ്ഞുണ്ടാക്കുന്നതില്‍ നിന്നെല്ലാം അല്ലാഹു എത്ര പരിശുദ്ധന്‍!'' (23: 91)(11)

ഈ വചനത്തില്‍ 'ചിലര്‍ ചിലരെ അടിച്ചമര്‍ത്തുകയും ചെയ്യുമായിരുന്നു'വെന്നു പരിഭാഷപ്പെടുത്തിയിരിക്കുന്നത് 'വ ലഅലാ ബഅദുഹും അലാ ബഅദിന്‍' എന്ന പ്രയോഗത്തെയാണ്. 'അലാ'യെന്നാല്‍ ആധിപത്യം സ്ഥാപിക്കുക, അടിച്ചമര്‍ത്തുക എന്നിങ്ങനെയാണ് യഥാര്‍ത്ഥത്തിലുള്ള സാരമെന്നര്‍ത്ഥം.

പുരുഷബീജത്തിലെ Y പെണ്‍ബീജത്തിലെ Xനുമേല്‍ ആധിപത്യം സ്ഥാപിക്കുമ്പോഴാണ് ആണ്‍കുഞ്ഞുണ്ടാകുന്നത് എന്നും പെണ്‍ബീജത്തിലെ X പുരുഷബീജത്തിലെ Yക്കുമേല്‍ ആധിപത്യം സ്ഥാപിക്കുമ്പോഴാണ് പെണ്‍കുഞ്ഞുണ്ടാകുന്നത് എന്നുമുള്ള ജനിതകശാസ്ത്ര വസ്തുതകളു മായി ഈ ഹദീഥുകള്‍ പൂര്‍ണമായും പൊരുത്തപ്പെടുന്നു. മധ്യകാലഘട്ടത്തിലുണ്ടായിരുന്ന അറിവിന്റെ അടിസ്ഥാനത്തില്‍ ഹദീഥ് മനസ്സിലാക്കിയവര്‍ ആണ്‍സ്ഖലനം ആദ്യം നടന്നാല്‍ ആണ്‍കുഞ്ഞും പെണ്‍സ്ഖലനം നടന്നാല്‍ പെണ്‍കുഞ്ഞുമുണ്ടാകുമെന്ന് ഇതില്‍നിന്ന് മനസ്സിലാക്കിയെന്നത് നബിവചനത്തിന്റെ ആശാസ്ത്രീയതയല്ല, അറിവിന്റെ കാലനിബന്ധതയെയാണ് വെളിപ്പെടുത്തുന്നത്. 'സബഖ' യെന്ന ക്രിയയെ വ്യാഖ്യാനിച്ചാല്‍ ആദ്യമുണ്ടാകുന്നത് ഏത് ദ്രവമാണോ അതിന്റെ അടിസ്ഥാനത്തിലാണ് ലിംഗനിര്‍ണയമെന്ന് വേണമെ ങ്കില്‍ പറയാനാകുമെങ്കിലും 'അലാ'യെന്ന പ്രയോഗം അത്തരമൊരു വ്യാഖ്യാനത്തിന് പഴുതുകളൊന്നും നല്‍കുന്നില്ല. ഈ ഹദീഥുകളെ ഒന്നിച്ചു പരിഗണിച്ചുകൊണ്ട്, നിലനില്‍ക്കുന്ന അറിവിന്റെ അടിസ്ഥാനത്തില്‍ വ്യാഖ്യാനിച്ചപ്പോഴാണ് പുരുഷ-പെണ്‍ സ്ഖലനങ്ങളുടെ ക്രമമാണ് ലിംഗനിര്‍ണയത്തിന് നിദാനമെന്നാണ് ഈ ഹദീഥുകള്‍ പഠിപ്പിക്കുന്നതെന്ന നിഗമനത്തില്‍ വ്യാഖ്യാതാക്കള്‍ എത്തിച്ചേര്‍ന്നത്. ഹദീഥുകളെ മൊത്തത്തിലെടുത്ത് പരിശോധിച്ചാല്‍ ഒരു ദ്രവത്തിനു മേലുള്ള മറ്റേ ദ്രവത്തിന്റെ ആധിപത്യം തന്നെയാണ് അവയില്‍ വ്യക്തമാക്കപ്പെട്ടിരിക്കുന്നതെന്ന് മനസ്സിലാകും. കഴിഞ്ഞ നൂറ്റാണ്ടിന്റെ പകുതിയില്‍ മാത്രം ശാസ്ത്രലോകത്തിന് മനസ്സിലായ ബീജത്തി ന്റെ ആധിപത്യമാണ് ലിംഗനിര്‍ണയത്തിന് കാരണമാകുന്നതെന്ന വസ്തുത എത്ര കൃത്യമായാണ് ഈ ഹദീഥുകള്‍ വരച്ച് കാണിക്കുന്നത്!

  1. മുസ്‌ലിം ഹുദൈഫത്തു ബ്‌നു അസീദില്‍ (റ) നിന്നും അബ്ദുല്ലാഹിബ്‌നു മസ്ഊദില്‍ (റ) നിന്നും നിവേദനം ചെയ്ത രണ്ട് വ്യത്യസ്ത ഹദീഥുകളില്‍ നിന്ന് ഗര്‍ഭസ്ഥശിശുവിലുളള ലിംഗമാറ്റത്തിനുവേണ്ടിയുള്ള മലക്ക് പ്രത്യക്ഷപ്പെടുന്നതും കുട്ടി ആണോ പെണ്ണോയെന്ന് ആത്യന്തികമായി തീരുമാനിക്കപ്പെടുന്നതും ബീജസങ്കലനത്തിന് ശേഷം നാല്‍പത് ദിവസങ്ങള്‍ക്കും നാല്‍പത്തിയഞ്ച് ദിവസങ്ങള്‍ക്കുമിടയിലാണെന്ന് വ്യക്തമാവുന്നു.

SRY ജീന്‍ പ്രവര്‍ത്തനക്ഷമമാകുന്നത് ആറാമത്തെ ആഴ്ചയാണെന്ന വിവരം നമുക്ക് ലഭിച്ചത് മൂന്നു പതിറ്റാണ്ടുകള്‍ക്ക് മുമ്പു മാത്രമാണ്. XX സിക്താണ്ഡമാണെങ്കിലും XY സിക്താണ്ഡമാണെങ്കിലും അപൂര്‍വമായുണ്ടാകുന്ന സിക്താണ്ഡങ്ങളാണെങ്കിലുമെല്ലാം അവയുടെ ലിംഗമെന്താ ണെന്ന് ആത്യന്തികമായി തീരുമാനിക്കപ്പെടുക SRY ജീന്‍ പ്രവര്‍ത്തനക്ഷമമാകുന്നതിന്റെ അടിസ്ഥാനത്തിലാണ്. ആറാമത്തെ ആഴ്ചയാണ് SRY ജീന്‍ പ്രവര്‍ത്തനക്ഷമമാവുന്നതെന്ന ഭ്രൂണശാസ്ത്രം 1985ല്‍ മാത്രം നമുക്കു പറഞ്ഞുതന്ന വിവരവും നാല്‍പതു ദിവസങ്ങള്‍ക്കും നല്‍പത്തിയഞ്ച് ദിവസങ്ങള്‍ക്കുമിടയിലാണ് ലിംഗതീരുമാനവുമായി മലക്ക് നിയോഗിക്കപ്പെടുന്നതെന്ന പതിനാലു നൂറ്റാണ്ടുകള്‍ക്ക് മുമ്പ് നബി (സ) പറഞ്ഞ വിവരവും എത്ര ക്രൃത്യമായാണ് ഇവിടെ യോജിച്ചുവരുന്നത്! എന്തുകൊണ്ടാണ് ഹദീഥുകളിലെ പരാമര്‍ശങ്ങള്‍ ഇത്ര യും കൃത്യമാകുന്നതെന്ന ചോദ്യത്തിന് ക്വുര്‍ആന്‍ തന്നെ ഉത്തരം നല്‍കിയിട്ടുണ്ട്.

''നിങ്ങളുടെ കൂട്ടുകാരന്‍ വഴിതെറ്റിയിട്ടില്ല. ദുര്‍മാര്‍ഗിയായിട്ടുമില്ല. അദ്ദേഹം തന്നിഷ്ടപ്രകാരം സംസാരിക്കുന്നുമില്ല. അത് അദ്ദേഹത്തിന് ദിവ്യസമ്പേശമായി നല്‍കപ്പെടുന്ന ഒരു ഉല്‍ബോധനം മാത്രമാകുന്നു.'' (53: 2-4)

കുറിപ്പുകൾ

  1. ക്വുര്‍ആന്‍ 53: 45-46
  2. ക്വുര്‍ആന്‍ 75: 38-40
  3. സ്വഹീഹുല്‍ ബുഖാരി, കിതാബു അഹാദീഥുല്‍ അംബിയാഅ്, ബാബു ഖല്‍ഖി ആദം വ ദുര്‍റിയ്യത്തിഹി, ഹദീഥ്
  4. സ്വഹീഹു മുസ്‌ലിം, കിതാബുല്‍ ഹൈദ്വ്, ബാബു വുജുബില്‍ ഗസ്‌ലി അലല്‍ മര്‍അത്തി ബി ഖുറൂജില്‍ മനിയ്യി മിന്‍ഹ, ഹദീഥ്
  5. സ്വഹീഹു മുസ്‌ലിം, കിതാബുല്‍ ഹൈദ്വ്, ബാബു ബയാനി സ്വിഫത്തില്‍ മനിയിര്‍റജുലി വല്‍ മര്‍അത്തി വ അന്നല്‍ വലദ മഖ്‌ലൂഖുന്‍ മിന്‍ മാഇ.
  6. സ്വഹീഹു മുസ്‌ലിം, കിതാബുല്‍ ക്വദ്ര്‍, ബാബു കൈഫിയ്യത്തില്‍ ഖല്‍ബില്‍ ആദമിയ്യി ഫീ ബദനി ഉമ്മിഹി വ കിതാബത്തി രിസ്‌കിഹി വ അജലിഹി, വ അമലിഹി വ ശകാവത്തിഹി വ സഅദത്തിഹി, ഹദീഥ്
  7. സ്വഹീഹു മുസ്‌ലിം, കിതാബുല്‍ ക്വദ്ര്‍, ബാബു കൈഫിയ്യത്തില്‍ ഖല്‍ബില്‍ ആദമിയ്യി ഫീ ബദനി ഉമ്മിഹി വ കിതാബത്തി രിസ്‌കിഹി വ അജലിഹി, വ അമലിഹി വ ശകാവത്തിഹി വ സഅദത്തിഹി, ഹദീഥ്
  8. സ്വഹീഹു മുസ്‌ലിം, കിതാബുല്‍ ക്വദ്ര്‍, ബാബു കൈഫിയ്യത്തില്‍ ഖല്‍ബില്‍ ആദമിയ്യി ഫീ ബദനി ഉമ്മിഹി വ കിതാബത്തി രിസ്‌കിഹി വ അജലിഹി, വ അമലിഹി വ ശകാവത്തിഹി വ സഅദത്തിഹി, ഹദീഥ്
  9. സ്വഹീഹു മുസ്‌ലിം, കിതാബുല്‍ ഹൈദ്വ്, ബാബു വുജുബില്‍ ഗസ്‌ലി അലല്‍ മര്‍അത്തി ബി ഖുറൂജില്‍ മനിയ്യി മിന്‍ഹ, ഹദീഥ്
  10. Edward William Lane : Arabic-English Lexicon, London, 1863, Book 1, Page 1300.
  11. ക്വുര്‍ആന്‍ 23: 91

ക്വുർആനിലെ 2: 2:233 വചനത്തില്‍ മുലകുടി പ്രായം രണ്ടു വര്‍ഷമാണെന്നും 46:15 വചനത്തില്‍ ഗര്‍ഭകാലവും മുലകുടി പ്രായവും കൂടി മുപ്പതു മാസമാണെന്നും പറഞ്ഞതിനെ താരതമ്യം ചെയ്താൽ ഗർഭകാലം ആറ് മാസമാണ് എന്നാണ് വന്നു ചേരുക. ഇത് വ്യക്തമായ അബദ്ധമല്ലേ ?

സൂറത്തുല്‍ ബക്വറയിലെ 2:233-ാം വചനത്തിലും സൂറത്തു ലുഖ്മാനിലെ പതിനാലാം വചനത്തിലും മുലകുടി പ്രായം രണ്ടു വര്‍ഷമാണെന്ന് വ്യക്തമാക്കുന്നുണ്ട്. മുലകുടി പൂർത്തിയായ്ക്കാനുദ്ദേശിക്കുന്നവർ കുഞ്ഞുങ്ങൾക്ക് രണ്ട് വര്‍ഷമാണ് മുല കൊടുക്കേണ്ടതെന്ന് ഈ വചനങ്ങൾ വ്യക്തമാക്കുന്നു. സൂറത്തുല്‍ അഹ്ക്വാഫിലെ പതിനഞ്ചാം വചനത്തിൽ "അവന്റെ ഗര്‍ഭകാലവും മുലകുടിനിര്‍ത്തലും കൂടി മുപ്പത് മാസക്കാലമാകുന്നു" എന്ന് വ്യക്തമായി പരാമർശിച്ചിട്ടുണ്ട്. ഇതിൽ നിന്ന് ഗർഭകാലം ആറ് മാസമാണ് എന്നല്ലേ മനസ്സിലാവുകയെന്നാണ് വിമർശനം. അങ്ങനെത്തന്നെയാണ് മനസ്സിലാക്കേണ്ടത് എന്നാണ് അതിനുള്ള വിശദീകരണം. ഒൻപത് മാസം കഴിഞ്ഞാണ് സാധാരണഗതിയിൽ പ്രസവം നടക്കാറുള്ളതെന്ന് ആരും പറഞ്ഞു കൊടുക്കാതെതന്നെ എല്ലാവർക്കും അറിയാവുന്ന കാര്യമാണ്. ഇക്കാര്യം മുഹമ്മദ് നബിക്ക് (സ) അറിയുമായിരുന്നില്ല എന്ന കരുതുന്നത് ശുദ്ധ അസംബന്ധമാണ്.

ഈ വചനങ്ങളിൽ നിന്ന് നാം മനസിലാക്കേണ്ടത് കുറഞ്ഞ ഗർഭകാലം ആറു മാനസമാണ് എന്നാണ്. ഇങ്ങനെ മനസ്സിലാക്കിയവരായിരുന്നു ആദ്യകാല മുസ്ലിംകൾ. ഒരു സംഭവം നോക്കുക:

വിവാഹത്തിനുശേഷം ആറുമാസങ്ങള്‍ കഴിയുന്നയുടനെ പ്രസവിച്ച ഒരു സ്ത്രീയെക്കുറിച്ച ഒരു പരാതി ഖലീഫ ഉമറിന്റെ (റ) അടുത്തെത്തി. പ്രസവിക്കപ്പെട്ട കുഞ്ഞിന് ജീവനും ആരോഗ്യവുമുള്ളതിനാല്‍ വിവാഹപൂര്‍വരതിയിലൂടെയുണ്ടായതാവണം അവരുടെ ഗര്‍ഭധാരണമെന്നും അതിനാല്‍ അവര്‍ക്ക് വ്യഭിചാരത്തിനുള്ള ശിക്ഷ നല്‍കണമെന്നുമായിരുന്നു പരാതിക്കാരുടെ പക്ഷം. പ്രശ്‌നത്തിനു പരിഹാരം തേടി പ്രവാചകാനുചരന്‍മാരുമായി ഉമര്‍ (റ) കൂടിയാലോചന നടത്തി. അബ്ദുല്ലാഹിബ്‌നു അബ്ബാസാണ് (റ) പ്രസ്തുത പ്രസവത്തെ ക്വുര്‍ആനിന്റെ അടിസ്ഥാനത്തില്‍ ന്യായീകരിച്ചത്. സൂറത്തുല്‍ ബക്വറയിലെ 2:233-ാം വചനവും സൂറത്തുല്‍ അഹ്ക്വാഫിലെ പതിനഞ്ചാം വചനവും ഉദ്ധരിച്ചുകൊണ്ട് ഈ വചനങ്ങള്‍പ്രകാരം കുറഞ്ഞ ഗര്‍ഭകാലം ആറുമാസമാണെന്ന് സ്ഥാപിക്കുകയാണ് അദ്ദേഹം ചെയ്തത്. സൂറത്തുല്‍ ബക്വറയിലെ വചനത്തില്‍ മുലകുടി പ്രായം രണ്ടു വര്‍ഷമാണെന്നും സൂറത്തുല്‍ അഹ്ക്വാഫില്‍ ഗര്‍ഭകാലവും മുലകുടി പ്രായവും കൂടി മുപ്പതു ദിവസമാണെന്നും പറഞ്ഞതിനെ താരതമ്യം ചെയ്തുകൊണ്ടാണ് പ്രവാചകാനുചരന്‍മാരിലെ ക്വുര്‍ആന്‍ വ്യാഖ്യാതാവെന്ന് അറിയപ്പെട്ടിരുന്ന ഇബ്‌നു അബ്ബാസ് (റ) കുറഞ്ഞ ഗര്‍ഭകാലം ആറു മാസമാണെന്ന് സമര്‍ത്ഥിച്ചത്. ഭരണാധികാരിയായ ഉമര്‍ (റ) അടക്കമുള്ള സ്വഹാബിമാരെല്ലാം അത് അംഗീകരിക്കുകയും കുറ്റാരോപിതയെ വെറുതെ വിടാന്‍ ഖലീഫ കല്‍പിക്കുകയും ചെയ്തു.(ഇമാം അബ്ദുര്‍റസാഖ് തന്റെ മുസന്നഫിലും (7: 352) ഇമാം സുയൂത്തി തന്റെ ദുര്‍റുല്‍ മന്‍സൂറിലും (7: 442) നാഫിഉ ബിന്‍ ജുബൈറില്‍ നിന്ന് നിവേദനം ചെയ്തത്.)

ഉഥ്മാന്റെ (റ) ഭരണകാലത്തും സമാനമായ സംഭവമുണ്ടായതായി ഇമാം മാലിക് (റ) നിവേദനം ചെയ്യുന്നുണ്ട്. ആറാം മാസം കഴിഞ്ഞയുടനെ ആരോഗ്യമുള്ള കുഞ്ഞിനെ പ്രസവിച്ച സ്ത്രീക്ക് വ്യഭിചാരക്കുറ്റത്തിന് ശിക്ഷ വിധിച്ച ഖലീഫയെ തിരുത്തിയത് അലി(റ)യാണ്. നടേ പറഞ്ഞ ആയത്തുകള്‍ ഉദ്ധരിച്ചുകൊണ്ട് ഖലീഫയുടെ വിധിയെ വിമര്‍ശിച്ചത് ഉഥ്മാന്‍ (റ) അംഗീകരിക്കുകയും സ്ത്രീയെ വെറുതെ വിടുകയും ചെയ്തു.(ഇമാം മാലികിന്റെ മുവത്വ 41: 11)

കുറഞ്ഞ ഗര്‍ഭകാലമെത്രയാണെന്ന കാര്യത്തില്‍ പ്രവാചകാനുചരന്‍മാരുടെ കാലം മുതല്‍ മുസ്‌ലിം ലോകത്ത് കാര്യമായ തര്‍ക്കങ്ങളുണ്ടായിട്ടില്ല. നാലു കര്‍മശാസ്ത്ര സരണികളും കുറഞ്ഞ ഗര്‍ഭകാലം ആറുമാസമാണെന്ന് അംഗീകരിക്കുന്നു. പിതൃത്വവും ശിക്ഷാവിധികളുമായി ബന്ധപ്പെട്ട മദ്ഹബീ നിയമങ്ങളിലെല്ലാം ഈ അംഗീകാരത്തിന്റെ സ്വാധീനം കാണാനാവും. മുസ്‌ലിം ലോകത്ത് പതിനാലു നൂറ്റാണ്ടുകളായി അംഗീകരിക്കപ്പെട്ടുവരുന്ന കുറഞ്ഞ ഗര്‍ഭകാലം തന്നെയാണ് ശരിയെന്ന വസ്തുത അംഗീകരിക്കുകയാണ് ആധുനിക സാങ്കേതികവിദ്യകളുടെ സഹായത്തോടെ ശാസ്ത്രം ഇന്നു ചെയ്യുന്നത്. കുറഞ്ഞ ഗര്‍ഭകാലത്തെക്കുറിച്ച സംവാദങ്ങളും തര്‍ക്കങ്ങളും ഭ്രൂണശാസ്ത്രലോകത്ത് സജീവമാണെങ്കിലും നിയമപരമായി അംഗീകരിക്കാവുന്ന കുറഞ്ഞ ഗര്‍ഭകാലം ആറുമാസമാണെന്ന വസ്തുത ഇന്ന് എല്ലാവരും സമ്മതിക്കുന്നുണ്ട്.

ഗര്‍ഭാശയത്തിനുപുറത്ത് ഗര്‍ഭസ്ഥശിശുവിന് ജീവിക്കാനുള്ള കഴിവിനെയാണ് ശിശുജീവനസാമര്‍ത്ഥ്യം (Fetal Viability) എന്നുവിളിക്കുന്നത്. ഗര്‍ഭകാലത്തെ മൂന്നു ത്രൈമാസിക യൂണിറ്റുകളായാണ് (trimester) ഭ്രൂണശാസ്ത്രജ്ഞന്‍മാര്‍ പഠിക്കുന്നത്. ആദ്യത്തെ ത്രൈമാസികത്തിലാണ് ഭ്രൂണത്തില്‍ അടിസ്ഥാനപരമായ മാറ്റങ്ങളെല്ലാം ഉണ്ടാകുന്നത്. ഒന്നാം ത്രൈമാസത്തിനകത്ത് പ്രസവിക്കപ്പെട്ടാല്‍ ശിശുജീവനസാമര്‍ത്ഥ്യം പൂജ്യമായിരിക്കും. അഥവാ അങ്ങനെ പ്രസവിക്കപ്പെടുന്ന കുഞ്ഞ് ഒരു കാരണവശാലും ജീവിച്ചിരിക്കുകയില്ല. രണ്ടാം ത്രൈമാസികത്തില്‍ നടക്കുന്നത് പ്രധാനമായും അവയവങ്ങളുടെ വികാസമാണ്. രണ്ടാമത്തെ തൈമാസികം അവസാനിക്കുമ്പോള്‍ പ്രസവിക്കപ്പെടുന്ന കുഞ്ഞിന് നല്ല പരിചരണം നല്‍കിയാല്‍ അത് ജീവിക്കും. ഈ സമയത്തെ ശിശുജീവനസാമര്‍ത്ഥ്യം (Fetal Viability) 90 ശതമാനമാണ്. നല്ല പരിചരണം നല്‍കിയാല്‍ കുഞ്ഞിനെ രക്ഷിക്കുവാനും കാര്യമാത്രപ്രസക്തമായ വൈകല്യങ്ങളൊന്നുമില്ലാതെ നിലനിര്‍ത്തുവാനും കഴിയുന്ന പ്രായമാണിത് എന്നര്‍ത്ഥം.

ഗര്‍ഭസ്ഥ ശിശുവിന് ഇരുപത്തിരണ്ടാമത്തെ ആഴ്ച പ്രായമാകുന്നതുമുതല്‍ തന്നെ ശിശുജീവനസാമര്‍ത്ഥ്യത്തിന് നേരിയ സാധ്യതകളുണ്ടെന്നാണ് പുതിയ പഠനങ്ങള്‍ വ്യക്തമാക്കുന്നത്. ഇരുപത്തിമൂന്നാമത്തെ ആഴ്ച ഇത് പത്തുമുതല്‍ മുപ്പത്തിയഞ്ച് വരെ ശതമാനവും ഇരുപത്തിനാലാമത്തെ ആഴ്ച ഇത് നാല്‍പത് മുതല്‍ എഴുപത് വരെ ശതമാനവും ഇരുപത്തിയഞ്ചാമത്തെ ആഴ്ച ഇത് അമ്പത് മുതല്‍ എണ്‍പതു വരെ ശതമാനവും ഇരുപത്തിയാറാമത്തെ ആഴ്ച ഇത് എണ്‍പത് മുതല്‍ തൊണ്ണൂറുവരെ ശതമാനവും ഇരുപത്തിയേഴാമത്തെ ആഴ്ച മുതല്‍ ഇത് തൊണ്ണൂറ് ശതമാനത്തിനു മുകളിലുമാണ്. ആറു മാസങ്ങള്‍ക്ക് മുമ്പുള്ള ശിശുജീവനസാമര്‍ത്ഥ്യത്തിന്റെ ശതമാനക്കകണക്ക് ഉയരാനുള്ള കാരണം ചികിത്സാരംഗത്തും സാങ്കേതിക വിദ്യയാലുമുണ്ടായ പുരോഗതിയാണ്. ഈ പുരോഗതിയുണ്ടായിട്ട് ഏതാണ്ട് പതിറ്റാണ്ടുകളേ ആയിട്ടുള്ളൂ. 1973ലെ പ്രസിദ്ധമായ ഒരു ഗര്‍ഭഛിദ്ര കേസില്‍ പോലും അമേരിക്കന്‍ സുപ്രീം കോടതി വിധിച്ചത് ശിശുജീവനസാമര്‍ത്ഥ്യം ഇരുപത്തിയെട്ട് ആഴ്ചകളെങ്കിലും പൂര്‍ത്തിയായാലേ ഉണ്ടാവുകയുള്ളുവെന്നാണ് പൊതുവെ കരുതി വരാറുള്ളതെന്നാണ്. ഇരുപത്തിനാല് ആഴ്ചകളെങ്കിലും പൂര്‍ത്തിയായാലേ ശിശുവിന് ജീവനസാമര്‍ത്ഥ്യമുണ്ടാകൂവെന്നാണ് ഇന്ന് പൊതുവെ ചികിത്സാരംഗത്തുള്ളവര്‍ പറയാറുള്ളതെങ്കിലും അതിനേക്കാള്‍ മുമ്പ് പ്രസവിക്കപ്പെട്ടിട്ടും ജീവിച്ച റിക്കാര്‍ഡുകളുണ്ട്.

2006 ഒക്‌ടോബര്‍ 24ന് ഫ്‌ളോഡിറിയില്‍ ഇരുപത്തിരണ്ട് ആഴ്ചകള്‍ മാത്രം കഴിഞ്ഞ് ജനിച്ച അമില്ലിയ ടൈലറെന്ന പെണ്‍കുട്ടിയാണ് ഏറ്റവും കുറഞ്ഞ ഗര്‍ഭകാലം കഴിഞ്ഞ് ജീവനസാമര്‍ത്ഥ്യത്തോടെയിരിക്കുകയും പിന്നീട് വളര്‍ന്നു വലുതാവുകയും ചെയ്തയാളായി രേഖപ്പെടുത്തപ്പെട്ടിരിക്കുന്നത്. ശ്വാസകോശങ്ങള്‍ക്കും ദഹനവ്യവസ്ഥക്കും തലച്ചോറിനുമെല്ലാം നിരവധി തകരാറുകളുണ്ടായിരുന്നുവെങ്കിലും മികച്ച സാങ്കേതികവിദ്യകളുടെ സഹായത്താല്‍ ഒരു കൂട്ടം ഭിഷഗ്വരന്‍മാര്‍ ഭഗീരഥപ്രയത്‌നം നടത്തി കുട്ടിയെ ജീവനോടെ നിലനിര്‍ത്തുകയാണുണ്ടായത്. നീണ്ട നാലുമാസങ്ങളില്‍ ആശുപത്രിയിലെ ശിശു തീവ്രപരിചരണ വിഭാഗത്തില്‍ കിടത്തിയുള്ള നിരന്തരമായ പരിശ്രമങ്ങളുടെ ഫലമായാണ് അവരുടെ മാതാപിതാക്കള്‍ക്ക് ജീവനുള്ള കുഞ്ഞിനെ ലഭിച്ചത് എന്നര്‍ത്ഥം.

രണ്ടാമത്തെ ത്രൈമാസം കഴിയുമ്പോഴേക്ക് ഗര്‍ഭസ്ഥശിശുവില്‍ ഒരുവിധം എല്ലാ ബാഹ്യാവയവങ്ങളും ആന്തരാവയവങ്ങളും വളര്‍ന്നുവന്നിരിക്കുമെന്നതിനാല്‍ തന്നെ അതിനുശേഷം പ്രസവിക്കപ്പെടുന്ന കുഞ്ഞുങ്ങള്‍ ജീവിച്ചിരിക്കുവാനുള്ള സാധ്യത അഥവാ ശിശുജീവനസാമര്‍ത്ഥ്യം തൊണ്ണൂറു ശതമാനത്തിനു മുകളിലാണ്. ഗര്‍ഭാശയത്തില്‍വെച്ചു തന്നെ പൂര്‍ണ വളര്‍ച്ചയെത്തി പുറത്തുവരുന്ന കുഞ്ഞ് മാതൃശരീരത്തിനകത്ത് തന്റെ ആദ്യകോശമുണ്ടാകുന്നതു മുതല്‍ മുപ്പത്തിയൊന്‍പത് ആഴ്ചക്കാലമാണ് കഴിച്ചുകൂട്ടുന്നത്. പൂര്‍ണമായ ഗര്‍ഭകാലമാണിത്. ഇതിനുമുമ്പ് ഏതു സമയത്തും കുഞ്ഞ് പ്രസവിക്കപ്പെടാം. ഗര്‍ഭാശയത്തിനകത്തും പുറത്തും കുഞ്ഞിന് വളരാനാവശ്യമായ സംവിധാനങ്ങളെല്ലാം ചെയ്തുവെച്ചിരിക്കുന്നവനാണ് സ്രഷ്ടാവ്. മാതൃശരീരത്തില്‍ നിന്ന് പുറത്തുവരുന്ന ശിശുവിന് പിന്നെ മാതാവുമായുള്ള ജൈവികബന്ധം അതിന്റെ മുലകുടിയാണ്. മനുഷ്യശിശുവിന്റെ മുലകുടി പ്രായം രണ്ടു വര്‍ഷമാണെന്ന കാര്യത്തില്‍ ശാസ്ത്രവും ക്വുര്‍ആനും ഒരേ അഭിപ്രായമാണ് പുലര്‍ത്തുന്നത്.

പൂര്‍ണമായ മുലകുടി പ്രായം രണ്ടു വര്‍ഷമാണെന്നു പറയുമ്പോള്‍ അതിനുമുമ്പ് ഏതുസമയത്തും മാതാവിന്റെയോ കുഞ്ഞിന്റെയോ ആരോഗ്യപരമായ കാരണങ്ങളാല്‍ മുലകുടി നിന്നുപോകുവാനുള്ള സാധ്യത ക്വുര്‍ആന്‍ അംഗീകരിക്കുന്നു. മുലകുടിയോടു കൂടി ബന്ധപ്പെടുത്തിയാണ് കുറഞ്ഞ ഗര്‍ഭകാലത്തെക്കുറിച്ച് ക്വുര്‍ആന്‍ പറഞ്ഞിരിക്കുന്നതെന്ന വസ്തുത ശ്രദ്ധേയമാണ്. മുലകുടിയും ഗര്‍ഭകാലവും കൂടി മുപ്പത് മാസമാണെന്ന ക്വുര്‍ആനിക പരാമര്‍ശമാണ് ചുരുങ്ങിയ ഗര്‍ഭകാലം ആറുമാസമാണെന്ന നിഗമനത്തിലെത്താന്‍ പ്രവാചകാനുചരന്മാരെ സഹായിച്ചത്. കാര്യമായ സാങ്കേതിക സഹായങ്ങളൊന്നുമില്ലെങ്കില്‍ പോലും, ആറു മാസങ്ങള്‍ പൂര്‍ത്തിയാക്കി ഗര്‍ഭാശയത്തിനകത്തുനിന്ന് പുറത്തുവരുന്ന കുഞ്ഞിന് ജീവിക്കുവാന്‍ കഴിയുമെന്നാണ് പുതിയ പഠനങ്ങളും വ്യക്തമാക്കുന്നത്. ആറുമാസം പൂര്‍ത്തിയാക്കുന്നതോടെ ശിശുവിന്റെ ജീവനസാമര്‍ത്ഥ്യം തൊണ്ണൂറ് ശതമാനമാണെന്നാണല്ലോ പഠനങ്ങള്‍ കാണിക്കുന്നത്.

ആറു മാസങ്ങള്‍ പൂര്‍ത്തീകരിക്കുന്നതിന് ഏതാനും ദിവസങ്ങള്‍ക്ക് മുമ്പ് പ്രസവിക്കുന്ന കുഞ്ഞുങ്ങളുടെ ജീവന്‍ രക്ഷിക്കുവാന്‍ സാങ്കേതിക സഹായങ്ങളോടെ സാധിക്കുമെന്നതിനാലാണ് കഴിഞ്ഞ അരനൂറ്റാണ്ടിനിടെ ജീവനസാമര്‍ത്ഥ്യത്തില്‍ കാര്യമായ പുരോഗതിയുണ്ടായിട്ടുണ്ടെന്നു പറയുന്നത്. പ്രസ്തുത പുരോഗതിയുടെ ഫലമായി ഇരുപത്തിനാല് ആഴ്ചകളെങ്കിലും പൂര്‍ത്തിയാക്കിയ കുഞ്ഞുങ്ങളെ രക്ഷിക്കാന്‍ കഴിഞ്ഞേക്കുമെന്ന ഒരു ധാരണ ചികിത്സാരംഗത്തുണ്ടായിട്ടുണ്ട്. വിദഗ്ധരായ ചികിത്സകരുടെ മേല്‍നോട്ടത്തില്‍ ശക്തമായ സാങ്കേതിക സഹായത്തോടെയാണ് പ്രസ്തുത രക്ഷിക്കല്‍ ശ്രമം നടക്കുന്നത്. അങ്ങനെ രക്ഷപെടുന്ന കുഞ്ഞുങ്ങള്‍ വ്യത്യസ്തതരം വൈകല്യങ്ങള്‍ക്ക് വിധേയമായിരിക്കും. തലച്ചോറ് വേണ്ടത്ര വികസിച്ചിട്ടില്ലാത്തതിനാല്‍ ഓട്ടിസമടക്കമുള്ള വൈകല്യങ്ങളുണ്ടാവാനുള്ള സാധ്യതയേറെയാണ്. ശ്വാസകോശങ്ങളുടെയും കണ്ഠനാളികളുടെയും വളര്‍ച്ച പൂര്‍ത്തീകരിച്ചു കഴിഞ്ഞിട്ടില്ലാത്തതിനാല്‍ അത്തരം ശിശുക്കള്‍ക്ക് മുല കുടിക്കുവാന്‍ പലപ്പോഴും കഴിയാറില്ല. മാതൃമുലപ്പാല്‍ പിഴിഞ്ഞ് വായിലേക്ക് ഉറ്റിച്ചുകൊടുക്കുകയോ സമാന്തര പോഷകങ്ങള്‍ നല്‍കിയോ ആണ് ചികിത്സകന്‍മാര്‍ ഈ പ്രശ്‌നം പരിഹരിക്കാറുള്ളത്. ആറു മാസങ്ങള്‍ പൂര്‍ത്തിയാക്കുന്നതിന് ഏതാനും ദിവസങ്ങള്‍ക്ക് മുമ്പ് പ്രസവിക്കുന്ന കുഞ്ഞുങ്ങളുടെ ജീവന്‍ രക്ഷിക്കാന്‍ കഴിയുമെങ്കിലും മുലകുടിയടക്കമുള്ള പല ശൈശവക്രിയകളും ചെയ്യാന്‍ അവയ്ക്ക് കഴിയുകയില്ലെന്നര്‍ത്ഥം.

മാതാപിതാക്കളോടുള്ള ബാധ്യതകളെക്കുറിച്ചു പറയുമ്പോള്‍ മാതാവ് തനിക്കുവേണ്ടി സഹിച്ച ത്യാഗങ്ങളെപ്പറ്റി ഓര്‍മിപ്പിച്ചുകൊണ്ടാണ് ക്വുര്‍ആന്‍ മുലകുടി പ്രായവും ഗര്‍ഭകാലവും കൂടി മുപ്പത് മാസങ്ങളാണെന്ന് പരാമര്‍ശിക്കുന്നതെന്ന വസ്തുത ശ്രദ്ധേയമാണ്. ആറുമാസമെങ്കിലുമുള്ള പൊക്കിള്‍കൊടി ബന്ധവും രണ്ടു വര്‍ഷത്തെ മുലകുടി ബന്ധവുമാണ് മാതൃശരീരവുമായി കുഞ്ഞിനുള്ള ജൈവികബന്ധമെന്ന ക്വുര്‍ആന്‍ പരാമര്‍ശം വളര്‍ന്നു വലുതായ ശേഷമുള്ള മാതാപിതാക്കളോടുള്ള ബാധ്യതയെക്കുറിച്ച് ഓര്‍മപ്പെടുത്തുന്നതിനിടയിലാണ് കടന്നുവരുന്നത്. മുലകുടി പ്രായവും കുറഞ്ഞ ഗര്‍ഭകാലവും കൂടി മുപ്പത് മാസങ്ങളാണെന്ന ക്വുര്‍ആന്‍ പരാമര്‍ശം ശാസ്ത്രീയമായ കൃത്യത മാത്രമല്ല വൈകാരിക ബന്ധത്തിനുണ്ടാവേണ്ട ആഴവും വ്യക്തമാക്കുന്നതാണ്.

അല്ല. ക്വുർആൻ പറഞ്ഞതാണ് ശരി!

ഭ്രൂണ ഘട്ടങ്ങളെക്കുറിച്ച് പറയുന്ന സൂറത്തുൽ മുഅമിനൂനിലെ വചനത്തിൽ പതിനാലാം വചനത്തിൽ നാം വായിക്കുന്നത് 'നാം ആ മാംസപിണ്ഡത്തെ (മുദ്‌അ) അസ്ഥികൂടമായി (ഇദ്വാമ്) രൂപപ്പെടുത്തി.' എന്നാണ്.

മുദ്അയില്‍ നിന്നാണ് എല്ലുകളുണ്ടാവുന്നതെന്നാണ് ഭ്രൂണവളര്‍ച്ചയെക്കുറിച്ചു പറയുമ്പോള്‍ ക്വുര്‍ആന്‍ വ്യക്തമാക്കുന്നത്.

ഗര്‍ഭസ്ഥശിശുവിന് അസ്ഥികളുണ്ടാകുവാനാരംഭിക്കുന്നത് നാല്‍പത്തിരണ്ടു ദിവസങ്ങള്‍ക്കുശേഷമാണെന്ന് നബി (സ) പഠിപ്പിച്ചതായി അബ്ദുല്ലാഹിബ്‌നു മസ്ഊദില്‍ (റ) നിന്ന് ഇമാം മുസ്‌ലിം നിവേദനം ചെയ്തിട്ടുണ്ട്.

അസ്ഥിരൂപീകരണവുമായി ബന്ധപ്പെട്ട് ആധുനിക ശാസ്ത്രം സാങ്കേതികസഹായത്തോടെ നമുക്ക് നല്‍കുന്ന അറിവുകള്‍ ക്വുര്‍ആനും നബിവചനങ്ങളും നല്‍കുന്ന വിവരങ്ങളുമായി പൂര്‍ണമായും യോജിച്ചു വരുന്നവെന്നതാണ് വസ്തുത. അസ്ഥിരൂപീകരണ പ്രക്രിയ അഥവാ ഓസിഫിക്കേഷന്‍ ആരംഭിക്കുന്നത് ആറ് ആഴ്ചകള്‍ക്കു ശേഷമാണെന്ന് പഠനങ്ങള്‍ വ്യക്തമാക്കുന്നു. നാല്‍പത്തിരണ്ടു ദിവസങ്ങള്‍ക്കുശേഷമാണ് അസ്ഥികള്‍ ഉണ്ടാകുന്നതെന്നാണ് നബി (സ) പഠിപ്പിച്ചത്. കടിച്ച മാംസപിണ്ഡത്തെപ്പോലെ തോന്നിപ്പിക്കുന്ന സോമൈറ്റുകള്‍ നിറഞ്ഞ ഭ്രൂണഘട്ടത്തിനുശേഷമാണ് അസ്ഥിരൂപീകരണം നടക്കുന്നതെന്നും സോമൈറ്റുകളില്‍ നിന്നാണ് നട്ടെല്ലുണ്ടാകുന്നതെന്നും ശാസ്ത്രം നമുക്ക് പറഞ്ഞുതരുന്നു. കടിച്ച മൃദുലമാംസപിണ്ഡം എന്നു അര്‍ത്ഥം വരുന്ന മുദ്വ്അയില്‍ നിന്നാണ് ഇദ്വാമ് (അസ്ഥികള്‍) ഉണ്ടാകുന്നതെന്ന് ക്വുര്‍ആന്‍ നമുക്ക് നല്‍കുന്ന വിവരം തന്നെയാണിത്. ക്വുര്‍ആനിക വിജ്ഞാനീയങ്ങളെല്ലാം ആധുനികശാസ്ത്രത്തിനുമുമ്പില്‍ അടിപതറാതെ നിലനില്‍ക്കുമെന്ന വസ്തുത ഇത് ഒരിക്കല്‍കൂടി വ്യക്തമാക്കുന്നു.

'മുദ്വ്അ'യെന്ന ഒരു ഘട്ടം ഭ്രൂണത്തിനുണ്ടെന്ന് ക്വുർആൻ പറയുന്നുണ്ടെന്നത് ശരിയാണ്. നുത്വഫ, അലഖ എന്നീ ഘട്ടങ്ങക്കു ശേഷമുള്ള മൂന്നാം ഘട്ടത്തെയാണ് ക്വുര്‍ആന്‍ 'മുദ്വ്അ'യെന്ന് വിളിക്കുന്നത്. അലഖയില്‍ നിന്നാണ് മുദ്വ്അയുണ്ടായതെന്ന് ക്വുര്‍ആന്‍ രണ്ടുതവണ പ്രസ്താവിക്കുന്നുണ്ട്. അവ ഇങ്ങനെയാണ്.

''പിന്നെ ആ ബീജത്തെ (നുത്വഫ) നാം ഒരു ഭ്രൂണമായി (അലഖ) രൂപപ്പെടുത്തി. ശേഷം ആ ഭ്രൂണത്തെ നാം ഒരു മാംസപിണ്ഡമായി (മുദ്വ്അ) രൂപപ്പെടുത്തി. (ക്വുര്‍ആന്‍ 23:14)

മനുഷ്യരേ, ഉയിര്‍ത്തെഴുന്നേല്‍പിനെ പറ്റി നിങ്ങള്‍ സംശയത്തിലാണെങ്കില്‍ (ആലോചിച്ച് നോക്കുക:) തീര്‍ച്ചയായും നാമാണ് നിങ്ങളെ മണ്ണില്‍ നിന്നും, പിന്നീട് ബീജത്തില്‍ നിന്നും, പിന്നീട് ഭ്രൂണത്തില്‍ നിന്നും, അനന്തരം രൂപം നല്‍കപ്പെട്ടതും രൂപം നല്‍കപ്പെടാത്തതുമായ മാംസപിണ്ഡത്തില്‍(മുദ്വ്അ) നിന്നും സൃഷ്ടിച്ചത്. (ക്വുര്‍ആന്‍ 22:5)

മീം, ദ്വ, ഗൊയ്‌ന് എന്നീ അക്ഷരത്രയങ്ങളില്‍ നിന്നാണ് 'മുദ്വ്അ'യെന്ന പദമുണ്ടായിരിക്കുന്നത്. ഈ അക്ഷരത്രയങ്ങളില്‍ നിന്നുണ്ടാകുന്ന പദങ്ങള്‍ക്കെല്ലാം വായിലിട്ട് ചവയ്ക്കുകയെന്ന ക്രിയയുമായി എന്തെങ്കിലും ബന്ധമുണ്ടായിരിക്കും. 'മുദ്വ്അ'യെന്നാല്‍ ചവയ്ക്കുകയെന്നര്‍ത്ഥം. ചവയ്ക്കപ്പെട്ടത് എന്ന അര്‍ത്ഥത്തില്‍ 'യുമ്ദുഅ്' എന്നും 'മംദൂഅ്വ്' എന്നും പ്രയോഗിക്കും. ചവയ്ക്കുന്നവന് മാദ്വിഅ് എന്നാണ് പറയുക. മുകളില്‍ പറഞ്ഞ അക്ഷരത്രയത്തില്‍ നിന്നുണ്ടായ ഒരു സവിശേഷനാമമാണ് 'മുദ്വ്അ'. 'ചവയ്ക്കാനുള്ളത്' എന്ന അര്‍ത്ഥത്തിലും 'ചവയ്ക്കപ്പെട്ടത്' എന്ന അര്‍ത്ഥത്തിലുമുപയോഗിക്കുന്ന നാമം. ചെറിയൊരു മാംസപിണ്ഡത്തിനും ഹൃദയത്തെയും നാവിനെയും പോലെയുള്ള ചവച്ചുതിന്നാന്‍ പറ്റുന്ന അവയവങ്ങളെയും മൃദുലമാംസത്തെയും ചവയ്ക്കാനുള്ള ച്യൂയിംഗം പോലുള്ള വസ്തുക്കളെയുമെല്ലാം 'മുദ്വ്അ'യെന്നു പറയും.

ഭ്രൂണത്തിന്റെ ഇരുപത് മുതല്‍ നാല്‍പതു വരെ ദിവസങ്ങളിലെ രൂപത്തെക്കുറിക്കുവാന്‍ 'മുദ്വ്അ'യെന്ന പദം വളരെ കൃത്യമാണെന്ന വസ്തുത അത്ഭുതകരമാണ്. ഏതാനും മില്ലീമീറ്റര്‍ മാത്രം നീളമുള്ള മൃദുലമായ ഒരു മാംസപിണ്ഡം. എല്ലില്ലാത്ത കടിച്ചു ചവയ്ക്കാന്‍ പാകത്തിലുള്ള ഒരു മുദ്വ്അ. ഇരുപതാം ദിവസം അതിന്മേല്‍ കടിച്ചതു പോലെയുള്ള അടയാളങ്ങളെപ്പോലെ തോന്നിക്കുന്ന ഒന്നാമത്തെ ജോഡി സോമൈറ്റുകള്‍ പ്രത്യക്ഷപ്പെടുന്നു. മുദ്വ്അയില്‍ ആദ്യത്തെ കടി വീണുവെന്ന് പറയാം. പിന്നെ ദിവസേന രണ്ടും മൂന്നും സോമൈറ്റുകള്‍ ഉണ്ടായിക്കൊണ്ടിരിക്കും. മുദ്വ്അയില്‍ രണ്ടും മൂന്നും നാലും കടികള്‍ വീണുകൊണ്ടിരിക്കും എന്നര്‍ത്ഥം. ചവയ്ക്കാനുള്ളത് എന്നും ചവയ്ക്കപ്പെട്ടത് എന്നും അര്‍ത്ഥം പറയാന്‍ കഴിയുന്ന 'മുദ്വ്അ'യെക്കാള്‍ കൃത്യമായ പദമേതാണുളളത്, ഈ ഘട്ടത്തെക്കുറിയ്ക്കുവാന്‍?

അലഖയില്‍ നിന്ന് മുദ്വ്അയുണ്ടാകുന്നതിനെക്കുറിച്ച് പ്രതിപാദിക്കുന്ന സൂറത്തുല്‍ ഹജ്ജിലെ അഞ്ചാമത്തെ വചനത്തില്‍ മുദ്വ്അയെ രണ്ടാക്കിത്തിരിച്ചത് പ്രത്യേകം ശ്രദ്ധേയമാണ്. മുദ്വ്അത്തിന് മുഖല്ലഖ വ ഗൊയ്‌റി മുഖല്ലഖയെന്നാണ് ഇവിടെ പ്രയോഗിച്ചിരിക്കുന്നത്. രൂപം നല്‍കപ്പെട്ടതും രൂപം നല്‍കപ്പെടാത്തതുമായ മുദ്വ്അയെന്നാണ് ഇതിനെ പരിഭാഷപ്പെടുത്തിയിരിക്കുന്നത്. മുഅല്ലഖ വഗൊയ്‌റി മുഅല്ലഖയെന്ന മുദ്വ്അക്കു നല്‍കിയ വിശേഷണങ്ങള്‍ ഭ്രൂണപരിണാമത്തിന്റെ വിവിധ ഘട്ടങ്ങളെക്കുറിക്കുന്നുവെന്ന് ഇബ്‌നു അബ്ബാസും ഖതാദ(റ)യും വിശദീകരിച്ചതായി ഇമാം തബ്‌രിയും ബഗാവിയും തങ്ങളുടെ ക്വുര്‍ആന്‍ വ്യാഖ്യാനഗ്രന്ഥങ്ങളില്‍ വ്യക്തമാക്കുന്നുണ്ട്. മുദ്വ്അയുടെ വിശേഷണങ്ങളായി മുഖല്ലഖയെന്നും ഗ്വൊയ്‌റു മുഖല്ലഖയെന്നും ഉപയോഗിച്ചതില്‍നിന്ന് ഇവരണ്ടും തന്നെ ആ ഘട്ടത്തിലെ രണ്ട് അവസ്ഥകളെയാണ് കുറിക്കുന്നതെന്ന് മനസ്സിലാകുന്നുണ്ട്.

മൂന്നാമത്തെ ആഴ്ച മുതല്‍ ആറാമത്തെ ആഴ്ച വരെയുള്ള കാലയളവില്‍ സോമൈറ്റുകളുടെ ഉല്‍പത്തിയോടൊപ്പം ഭ്രൂണത്തില്‍ നടക്കുന്ന മറ്റു മാറ്റങ്ങളെന്തൊക്കെയാണെന്ന് പരിശോധിക്കുമ്പോഴാണ് രൂപം നല്‍കപ്പെടാത്തതും രൂപം നല്‍കപ്പെട്ടതുമായ മുദ്വ്അയെന്ന പ്രയോഗത്തിന്റെ സൗന്ദര്യം മനസ്സിലാവുക. സോമൈറ്റുകളുണ്ടാക്കുവാന്‍ തുടങ്ങുന്ന ഇരുപതാം ദിവസം ഭ്രൂണം ഒരു കോശക്കൂട്ടം മാത്രമാണ്. സോമൈറ്റുകളുടെ ഉല്‍പത്തിയോടൊപ്പം തന്നെ ഭ്രൂണത്തിന്റെ രൂപം മാറാന്‍ തുടങ്ങുന്നു. ഇരുപത്തിമൂന്നാമത്തെ ദിവസം പത്ത് സോമൈറ്റ് ജോഡികള്‍ ഉണ്ടായതിനുശേഷവും ഗര്‍ഭാശയഭിത്തിയില്‍ പറ്റിക്കിടക്കുന്ന ഒരു ചെറിയ അട്ടയില്‍നിന്ന് കാര്യമായ രൂപവ്യത്യാസങ്ങളൊന്നും തന്നെ ഭ്രൂണത്തിനുണ്ടാവുകയില്ല. ഇരുപത്തിനാലാമത്തെ ദിവസം മുതല്‍ക്കാണ് പ്രകടമായ രൂപവ്യത്യാസം തുടങ്ങുന്നത്. അട്ടയുടെ തലയ്ക്കു കീഴിലായി മാംസകമാനങ്ങള്‍ പ്രത്യക്ഷപ്പെടുന്നതോടെയാണ് ഈ മാറ്റം പ്രകടമാവുക. ഫാരിന്‍ജിയല്‍ കമാനങ്ങള്‍ (pharyngeal arches) എന്നാണ് ഈ കമാനങ്ങളുടെ പേര്. ഒന്നാമത്തെയും രണ്ടാമത്തെയും കമാനങ്ങള്‍ പ്രകടമാവുന്നത് ഇരുപത്തിനാലാം ദിവസമാണ്. ഇരുപത്തിയഞ്ചാം ദിവസമാകുമ്പോള്‍ തലയും വാലും അല്‍പം മുന്നിലേക്കുവളഞ്ഞ് ഏകദേശം അര്‍ദ്ധവൃത്താകൃതിയിലേക്ക് പതിനാറ് സോമൈറ്റുകളുള്ള ഭ്രൂണം പരിണമിക്കുന്നു. ഇരുപത്തിയേഴാം ദിവസമാകുമ്പോഴേക്ക് മൂന്നാമത്തെ ഫാരന്‍ജിയല്‍ കമാനമുണ്ടാവുകയും അതിനുമുകളിലായി ആന്തരിക കര്‍ണമായിത്തീരാനുള്ള കര്‍ണദ്വാരം (oticpit) വ്യക്തമായി കാണാന്‍ തുടങ്ങുകയും ചെയ്യുന്നു. C ആകൃതിയിലുള്ള ഭ്രൂണത്തിനു മധ്യത്തില്‍ കൈമുകുളങ്ങളും (arm buds) കാണാന്‍ തുടങ്ങുന്നത് അന്നുതന്നെയാണ് ഇരുപത്തിയെട്ടാം ദിവസത്തില്‍ നാലാമത്തെ ആഴ്ചയുടെ അവസാനത്തിലാണ് ഫാരിന്‍ജിയല്‍ കമാനങ്ങളുടെ നാലാമത്തെ ജോഡിയും കാല്‍മുകുളങ്ങളും (leg buds)പ്രത്യക്ഷപ്പെടുന്നത്. തലച്ചോറിന്റെ വര്‍ദ്ധനവുകാരണമുള്ള തലയുടെ വളര്‍ച്ചയാണ് അഞ്ചാം ആഴ്ചയില്‍ പ്രധാനമായും നടക്കുന്നത്. തല വളര്‍ന്ന് അതിന്റെ മുഖം ഹൃദയഭാഗത്തെ സ്പര്‍ശിക്കുന്ന അവസ്ഥയിലെത്തുന്നു. ആറാമത്തെ ആഴ്ചയിലാകുമ്പോഴേക്ക് കൈപ്പത്തിയുടെയും കൈമുട്ടുകളുടെയും കൈവിരലുകളുടെയുമെല്ലാം പ്രാഗ്‌രൂപങ്ങള്‍ കാണാന്‍ കഴിയും. നാല്‍പതാമത്തെ ദിവസമാകുമ്പോള്‍ തല കുറേക്കൂടി വലുതാവുകയും ചെവിയായിത്തീരുവാനുള്ള കര്‍ണഅറയും (otic vesicle) കണ്ണിന്റെ ഭാഗത്ത് റെറ്റിനല്‍ വര്‍ണവും (retinal pigment) പ്രത്യക്ഷപ്പെടുകയും ചെയ്യും. സോമൈറ്റുകളെല്ലാം ഉത്ഭവിച്ചു കഴിയുമ്പോഴേക്ക് രൂപം നല്‍കെപ്പട്ട മുദ്വ്അയുടെ പൂര്‍ണരൂപം നമുക്ക് കാണാനാകുമെന്നര്‍ത്ഥം. അപ്പോഴുള്ള മുദ്വ്അക്ക് കണ്ണും കാതും തലയും കൈകാലുകളുമെല്ലാം ഉണ്ടായിരിക്കും.

ഇരുപതാം ദിവസം മുതല്‍ നാല്‍പതാം ദിവസം വരെയുള്ള ചെറിയ കാലയളവില്‍ ഭ്രൂണത്തിനകത്തും പുറത്തുമുണ്ടാകുന്നതുപോലെയുള്ള മാറ്റങ്ങള്‍ മറ്റൊരു ഭ്രൂണഘട്ടത്തിലുമുണ്ടാകുന്നില്ല. കേവലമൊരു കോശക്കൂട്ടമായി ഗര്‍ഭാശയഭിത്തിയില്‍ അള്ളിപ്പിടിച്ച് കിടക്കുകയായിരുന്ന കാര്യമാത്ര പ്രസക്തമായ രൂപങ്ങളൊന്നുമില്ലാതിരുന്ന ഭ്രൂണത്തിന് കൈകാലുകളും തലയും കണ്ണും കാതുമെല്ലാം കാണാന്‍ കഴിയുന്ന രീതിയിലായിത്തീരുന്നത് ഈ ഘട്ടത്തിലാണ്. രൂപം നല്‍കപ്പെട്ടതും (മുഖല്ലഖ) രൂപം നല്‍കപ്പെടാത്തതുമായ (ഗയ്‌റു മുഖല്ലഖ) ചവയ്ച്ച മാംസപിണ്ഡം (മുദ്വ്അ) എന്ന ക്വുര്‍ആനിക പ്രയോഗം എത്രമാത്രം കൃത്യമാണെന്ന് ഇത് മനസ്സിലാക്കിത്തരുന്നു.

മുദ്വ്അയെന്ന പദത്തിന് ചവയ്ക്കപ്പെട്ടത് എന്ന് അര്‍ത്ഥമില്ലെന്നും സോമൈറ്റുകളെക്കുറിച്ച് മനസ്സിലാക്കിയശേഷം ഇസ്‌ലാമിക പ്രബോധകര്‍ പടച്ചുണ്ടാക്കിയതാണ് പ്രസ്തുത അര്‍ത്ഥമെന്നും വാദിക്കുന്നവര്‍ക്ക് ആദ്യകാല ക്രൈസ്തവരേഖകള്‍ തന്നെ മറുപടി പറയുന്നുണ്ട്. ബൈസന്റൈന്‍കാരുടെ ഭരണപ്രദേശത്ത് ഇസ്‌ലാമില്‍നിന്ന് ക്രിസ്തുമതം സ്വീകരിപ്പിക്കുവാന്‍ ഉപയോഗിച്ചിരുന്ന ക്രിസ്താബ്ദം ഒന്‍പതാം നൂറ്റാണ്ടിലെ ഒരു ശാഖാ വര്‍ത്തമാനം ഇങ്ങനെയാണ് രേഖപ്പെടുത്തപ്പെട്ടിട്ടുള്ളത്. ''മനുഷ്യസൃഷ്ടിയെക്കുറിച്ച മുഹമ്മദിന്റെ പരികല്‍പനകളെ ഞാന്‍ തള്ളിക്കളയുന്നു. മനുഷ്യന്‍ പൊടിയില്‍ നിന്നും ദ്രാവകത്തുള്ളിയില്‍നിന്നും അട്ടകളില്‍നിന്നും ചവയ്ക്കപ്പെട്ടതുപോലുള്ള വസ്തുവില്‍ നിന്നുമാണ് സൃഷ്ടിക്കപ്പെട്ടത് എന്നാണ് അയാള്‍ പറയുന്നത്.''(A Byzantine anathema recorded during Muslim conversions to Christianity reads: I anathematize Muhammad’s teaching about the creation of man, where he says that man was created from dust and a drop of fluid , and leeches and chewed-like substance )

പരിശുദ്ധ ക്വുര്‍ആനിന് ഇസ്‌ലാം വിമര്‍ശകര്‍ നല്‍കിയ പരിഭാഷകളില്‍ പോലും അലഖിന് അട്ടയെന്നും മുദ്വ്അക്ക് ചവച്ചരയ്ക്കപ്പെട്ടത് എന്നുമാണ് അര്‍ത്ഥം നല്‍കിയിരുന്നത് എന്ന് ഇതില്‍നിന്ന് വ്യക്തമാണ്. മുസ്‌ലിംകള്‍ ക്വുര്‍ആന്‍ പരിഭാഷകള്‍ ഉപയോഗിക്കാന്‍ തുടങ്ങുന്നതിനു നൂറ്റാണ്ടുകള്‍ക്ക് മുമ്പുതന്നെ ഗ്രീക്കുഭാഷയിലേക്ക് ക്വുര്‍ആന്‍ ഭാഷാന്തരം ചെയ്ത ഇസ്‌ലാമിന്റെ ശത്രുക്കള്‍ക്ക് അലഖയുടെ അര്‍ത്ഥം അട്ടയെന്നാണെന്നും മുദ്വ്അയുടെ അര്‍ത്ഥം ചവയ്ക്കപ്പെട്ടത് എന്നാണെന്നും മനസ്സിലായിയെന്നാണ ല്ലോ ഇതു മനസ്സിലാക്കിത്തരുന്നത്. ശാസ്ത്രവസ്തുതകള്‍ക്കനുസരിച്ച് ക്വുര്‍ആന്‍ വ്യാഖ്യാനിക്കുകയല്ല, ക്വുര്‍ആന്‍ പ്രയോഗങ്ങളുടെ സത്യതയിലേക്ക് ശാസ്ത്രം തെളിക്കുന്ന വെളിച്ചത്തെക്കുറിച്ച് ബോധ്യപ്പെടുത്തുക മാത്രമാണ് ഇസ്‌ലാമിക പ്രബോധകര്‍ ചെയ്യുന്നതെന്ന യാഥാര്‍ത്ഥ്യമാണ് ഇവിടെ അനാവൃതമാകുന്നത്.

ല്ല. വളരെ കൃത്യമായ പരാമർശമാണ് ഇവിടെ ഖുർആൻ നടത്തുന്നത്. വിമർശക്കപ്പെട്ട വചനം നോക്കുക:

"നിങ്ങളുടെ മാതാക്കളുടെ വയറുകളില്‍ നിങ്ങളെ അവന്‍ സൃഷ്ടിക്കുന്നു. മൂന്ന്‌ തരം അന്ധകാരങ്ങള്‍ക്കുള്ളില്‍ സൃഷ്ടിയുടെ ഒരു ഘട്ടത്തിന്‌ ശേഷം മറ്റൊരു ഘട്ടമായിക്കൊണ്ട്‌." (39: 6)

മാതാക്കളുടെ വയറുകള്‍ക്കകത്തെ ഘട്ടംഘട്ടമായ മനുഷ്യസൃഷ്ടി നടക്കുന്നത് മൂന്നുതരം അന്ധകാരങ്ങള്‍ക്കകത്താണ് എന്നാണ് ക്വുര്‍ആന്‍ ഇവിടെ പ്രസ്താവിച്ചിരിക്കുന്നത്. അടിവയറും ഗര്‍ഭാശയവും ആംനിയോണ്‍-കോറിയോണ്‍ സ്തരവുമാണ് കുഞ്ഞിനെ സംരക്ഷിച്ചുനിര്‍ത്തുന്ന പാളികള്‍ എന്ന് നമുക്കറിയാം. മൂന്നുതരം ഇരുട്ടുകള്‍ എന്ന ക്വുര്‍ആനിക പ്രയോഗം എത്രമാത്രം കൃത്യമാണെന്ന് നോക്കുക. ഈ മൂന്ന് പാളികളാണ് ഭ്രൂണവളര്‍ച്ചയ്ക്കാവശ്യമായ ഇരുട്ട് പ്രദാനം ചെയ്യുന്നത് എന്നുകൂടി അറിയുമ്പോഴാണ് ഈ പ്രയോഗം എത്രത്തോളം സൂക്ഷ്മവും കൃത്യവുമാണെന്ന് മനസ്സിലാവുക. കുഞ്ഞിനെ സംരക്ഷിക്കുകയും അതിന്ന് വളരാനാവശ്യമായ അന്ധകാരം സൃഷ്ടിക്കുകയും ചെയ്തുകൊണ്ടുള്ള മൂന്നുതരം പാളികള്‍ക്കുള്ളിലാണ് കുഞ്ഞിന്റെ ഘട്ടങ്ങളായുള്ള വളര്‍ച്ച നടക്കുന്നതെന്ന് ക്വുര്‍ആനിന്റെ സമകാലികരോ പൂര്‍വികരോ ആയ ഭ്രൂണപഠിതാക്കളൊന്നും തന്നെ പറഞ്ഞതായി കാണാന്‍ കഴിയുന്നില്ല. പുരാതന ഗ്രീക്കുകാരോ ജൂതന്‍മാരോ മാത്രമല്ല, ആധുനിക കാലം വരെയുള്ള ഇവ്വിഷയകമായി പഠിച്ചവരൊന്നും തന്നെ കുഞ്ഞിന് അന്ധകാരവും സുരക്ഷയും നല്‍കുന്ന മൂന്ന് പാളികളെക്കുറിച്ച് പരാമര്‍ശിക്കുന്നില്ല. എന്നിട്ടും എങ്ങനെയാണ് നിരക്ഷനായ ഒരാളുടെ നാവില്‍ നിന്ന് ലോകം കേട്ട വചനങ്ങളില്‍ ഇത്ര കൃത്യമായ പരാമര്‍ശങ്ങളുണ്ടാവുന്നതെന്ന ചോദ്യത്തിന് ഒരൊറ്റ ഉത്തരം മാത്രമേയുള്ളു: ''ഈ ഗ്രന്ഥത്തിന്റെ അവതരണം സര്‍വലോകരക്ഷിതാവിങ്കല്‍ നിന്നാകുന്നു. ഇതില്‍ യാതൊരു സംശയവുമില്ല.''(32 : 2)

ക്വുർആൻ 86: 5 -7 വചനങ്ങൾ പറയുന്നത് നട്ടെലിന്റെയും വാരിയെല്ലിന്റെയും ഇടയിൽ നിന്നാണ് ബീജമുണ്ടാവുന്നത് എന്നാണല്ലോ. അരക്കെട്ടിലുള്ള വൃഷണത്തിലാണ് ബീജങ്ങൾ ഉത്പാദിപ്പിക്കപ്പെടുന്നത് എന്നിരിക്കെ ഈ പരാമർശം ശുദ്ധ അബദ്ധമല്ലേ? ഗ്രീക്ക് വൈദ്യനായിരുന്ന ഹിപ്പോക്രാറ്റസിന്റെ ഭ്രൂണ ശാസ്ത്ര ചിന്തകളെ മുഹമ്മദ് നബി (സ) കോപ്പിയടിച്ചത് കൊണ്ടാണ് ഈ അബദ്ധം സംഭവിച്ചത് എന്ന വിമര്ശനനത്തിന് എന്ത് മറുപടി പറയും ?

അല്പം വിശദമായി മറുപടി പറയേണ്ട വിഷയമാണിത്. നിലവിലുള്ള മലയാളം ഖുർആൻ പരിഭാഷകളിലും പ്രധാന ഇംഗ്ലീഷ് പരിഭാഷകളിലുമെല്ലാം ഈ വചനത്തിന് നൽകിയിരിക്കുന്ന ഭാഷാന്തരം വിമർശകൻ സൂചിപ്പിക്കുന്നത് പോലെത്തന്നെയാണ്. അത് ഇങ്ങനെയാണ്: ''എന്നാല്‍ മനുഷ്യന്‍ ചിന്തിച്ചു നോക്കട്ടെ താന്‍ എന്തില്‍ നിന്നാണ് സൃഷ്ടിക്കപ്പെട്ടിരിക്കുന്നത് എന്ന്. തെറിച്ചു വീഴുന്ന ഒരു ദ്രാവകത്തില്‍ നിന്നത്രെ അവന്‍ സൃഷ്ടിക്കപ്പെട്ടിരിക്കുന്നത്. മുതുകെല്ലിനും, വാരിയെല്ലുകള്‍ക്കുമിടയില്‍ നിന്ന് അത് പുറത്തു വരുന്നു.''(1)

വിഷയം കൃത്യമായി മനസ്സിലാക്കാൻ മുകളിലെ വിമർശനത്തെ മൂന്നായി തരം തിരിച്ച് പരിശോധിക്കാം എന്ന് തോന്നുന്നു:

1) മുഹമ്മദ് നബി(സ)ക്ക് ബീജത്തിന്റെ ഉല്‍ഭവസ്ഥാനമാണ് വൃഷണങ്ങള്‍ എന്ന് അറിയുമായിരുന്നില്ല.

2) വാരിയെല്ലുകളുടെയും മുതുകെല്ലിന്റെയും മധ്യെ നിന്നാണ് ശുക്ലമുണ്ടാവുന്നതെന്ന ഹിപ്പോക്രാറ്റിസിന്റെ ഭ്രൂണപരിണാമ ചിന്തകളെ പകര്‍ത്തുകയാണ് ഖുര്‍ആനിൽ ചെയ്തിരിക്കുന്നത്.

3 ) ക്വുർആനിലെ ഈ പരാമർശം പരമാബദ്ധമാണ്.

ഈ ധാരണകള്‍ എത്രത്തോളം ശരിയാണ്? നാം പരിശോധിക്കുക:

1) ഖുര്‍ആനിന്റെ അവതരണകാലത്ത് ജീവിച്ചിരുന്ന അറബികള്‍ക്കെല്ലാം അറിയാമായിരുന്ന ഒരു യാഥാര്‍ഥ്യമായിരുന്നു പുനരുല്‍പാദനത്തിന് കാരണമാകുന്ന ബീജങ്ങളുടെ ഉല്‍പാദനം നടക്കുന്നത് വൃഷണങ്ങളിലാണെന്ന വസ്തുത. വരിയുടയ്ക്കലിനെ (castration) സംബന്ധിച്ച് അക്കാലത്ത് നിലനിന്നിരുന്ന ധാരണകളില്‍ നിന്ന് ഇക്കാര്യം ബോധ്യപ്പെടും. പച്ചിലകള്‍ ചേര്‍ത്ത ചൂടുവെള്ളത്തില്‍ ഇരുത്തിയശേഷം മെല്ലെമെല്ലെ വൃഷണത്തില്‍ തടവുകയും പിന്നെ ശക്തി ഉപയോഗിച്ച് വൃഷണം ഉടച്ചുകളയും ചെയ്യുന്ന രീതിയായിരുന്നു വ്യാപകമായി നിലവിലുണ്ടായിരുന്ന ഷണ്ഡീകരണ രീതി. ഇങ്ങനെ വന്ധ്യംകരണം നടത്തുന്നവര്‍ക്ക് വൃഷണങ്ങള്‍ക്കകത്താണ് പുനരുല്‍പാദനത്തിന് കാരണമാകുന്ന ബീജം ഉല്‍പാദിപ്പിക്കപ്പെടുന്നതെന്ന് കൃത്യമായി അറിയാമായിരുന്നുറപ്പാണല്ലോ.

മുഹമ്മദ് നബി(സ)ക്കും സമകാലികര്‍ക്കുമെല്ലാം വരിയുടച്ച് ഷണ്ഡീകരിക്കുന്ന രീതിയെക്കുറിച്ച് അറിയാമായിരുന്നുവെന്ന് വ്യക്തമാക്കുന്ന നിരവധി ഹദീഥുകളുണ്ട്. സ്വഹീഹുല്‍ ബുഖാരി, കിത്താബുത്തഫ്‌സീറിലും സ്വഹീഹു മുസ്‌ലിം കിത്താബുന്നികാഹിലും ഇബ്‌നു മസ്ഊദിൽ (റ) നിന്ന് നിവേദനം ചെയ്യുന്ന ഹദീഥും സ്വഹീബുല്‍ ബുഖാരി, കിത്താബുന്നികാഹിലും സ്വഹീഹു മുസ്‌ലിം, കിത്താബുന്നികാഹിലും അബൂഹുറൈറയിൽ (റ) നിന്ന് നിവേദനം ചെയ്യുന്ന ഹദീഥും ഇതേ കിതാബുകളിൽ സഅദിൽ (റ) നിന്ന് നിവേദനം ചെയ്യുന്ന ഹദീഥും ഇവയിൽ ചിലതാണ്. പ്രത്യുല്‍പാദനത്തിന് കാരണമാകുന്ന ബീജത്തിന്റെ ആവിര്‍ഭാവം വൃഷണത്തില്‍ നിന്നാണെന്ന് അറിയാമായിരുന്നു എന്നതുകൊണ്ടാണല്ലോ വൃഷണമുടച്ച് ലൈംഗികശേഷിയെയും പ്രത്യുല്‍പാദനത്തെയും ഇല്ലാതാക്കുന്നത് വിരോധിക്കപ്പെട്ടത്. വൃഷണങ്ങളാണ് ബീജോല്‍പാദനത്തിന്റെ കേന്ദ്രമെന്ന വസ്തുത മുഹമ്മദ് നബി(സ) ക്കും അനുചരന്‍മാര്‍ക്കും അക്കാലത്ത് ജീവിച്ചിരുന്ന അറബികള്‍ക്കുമെല്ലാം അറിയാമായിരുന്നുവെന്നാണ് ഇതില്‍നിന്ന് വ്യക്തമായി മനസ്സിലാകുന്നത്.

2) ഗ്രീക്ക് വൈദ്യനായിരുന്ന ഹിപ്പോക്രാറ്റസിന്റെ ബീജോല്‍പാദനത്തെക്കുറിച്ച അഭിപ്രായങ്ങളുള്ളത് The seed and the Nature of the child, On Generation എന്നീ പ്രബന്ധങ്ങളിലാണ്. ഇവയിലൊന്നും തന്നെ നെഞ്ചെല്ലിനും മുതുകെല്ലിനും മധ്യെ നിന്നാണ് ബീജോല്‍പാദനം നടക്കുന്നതെന്ന് അദ്ദേഹം അഭിപ്രായപ്പെടുന്നില്ല. ബീജോല്‍പാദനത്തെക്കുറിച്ച് ഹിപ്പോക്രാറ്റസ് പറയുന്ന കാര്യങ്ങളെ ഇങ്ങനെ സംക്ഷേപിക്കാം.

a) ശരീരത്തിലാകമാനം വ്യാപിച്ചുകിടക്കുന്ന രസത്തിന്റെ ഏറ്റവും ശക്തമായ ഭാഗമാണ് ബീജമായി പുറത്തേക്കു വരുന്നത്.

b) ശരീരത്തിന്റെ എല്ലാഭാഗത്തുനിന്നും ലിംഗത്തിലേക്ക് നീളുന്ന ധമനികളുണ്ട്. അതിലൂടെയാണ് രസം പ്രവഹിക്കുന്നത്.

c) തലച്ചോറില്‍നിന്ന് അരക്കെട്ടുവരെയുള്ള ശരീരഭാഗങ്ങളിലെല്ലാം വിശേഷിച്ചും നട്ടെല്ലിന്റെ മജ്ജയില്‍ രസം വ്യാപിക്കുന്നു.

d) സുഷുമ്‌നാ മജ്ജയിലെത്തുന്ന ശുക്ലം വൃക്കയിലൂടെയുള്ള ധമനികളിലൂടെ കടന്നുപോകുന്നു.

e) വൃക്കയില്‍നിന്നും ഇത് വൃഷണങ്ങളിലൂടെ ലിംഗത്തിലേക്ക് ഒഴുകുന്നു.

ഇതിലെവിടെയും തന്നെ വാരിയെല്ലിന്റെയും മുതുകെല്ലിന്റെയും മധ്യത്തുനിന്നാണ് ബീജം ഉല്‍പാദിപ്പിക്കപ്പെടുന്നതെന്നു പറയുന്നില്ല. ഹിപ്പോക്രാറ്റസിന് ഇല്ലാത്ത ഒരു വാദം അദ്ദേഹത്തില്‍നിന്ന് കോപ്പിയടിക്കുന്നതെങ്ങനെയാണ്? ഹിപ്പോക്രാറ്റസിന്റെ ഭ്രൂണശാസ്ത്രചിന്തകള്‍ അറബികള്‍ക്കിടയില്‍ പ്രചാരത്തിലുണ്ടായിരുന്നുവെന്നത് തെളിവുകളൊന്നും ഇല്ലാത്ത ഒരു ഊഹംമാത്രമാണെന്ന വസ്തുത കൂടി ഇതോട് ചേർത്ത് വായിക്കുക.

മുഹമ്മദ് നബി(സ) ജീവിച്ചിരുന്ന കാലത്ത് വാരിയെല്ലുകള്‍ക്കും മുതുകെല്ലിനുമിടയില്‍ നിന്നാണ് ബീജം ഉല്‍പാദിപ്പിക്കപ്പെടുന്നതെന്ന വിശ്വാസം അറബികള്‍ക്കിടയില്‍ നിലനിന്നിരുന്നുവെങ്കില്‍ ഇസ്‌ലാം പൂര്‍വകാലത്തെ രചനകളിലോ പ്രവാചകശിഷ്യന്‍മാരുടെ വര്‍ത്തമാനങ്ങളിലോ അതിന്റെ സൂചനകളെന്തെങ്കിലുമുണ്ടാകുമായിരുന്നു. അങ്ങനെ ഇല്ലെന്നുമാത്രമല്ല, ഇത്തരമൊരു പരാമര്‍ശം നടത്തുന്ന ഒരേയൊരു കൃതി ഖുര്‍ആനാണുതാനും. അറബികള്‍ക്കിടയില്‍ പ്രചാരത്തിലുണ്ടായിരുന്ന ഹിപ്പോക്രാറ്റിയന്‍ ചിന്തകളെ പകര്‍ത്തുകയാണ് ഖുര്‍ആന്‍ ചെയ്യുന്നതെന്ന വാദം തീരെ അടിസ്ഥാന രഹിതമാണെന്നർത്ഥം.

3) വിമര്‍ശിക്കപ്പെടുന്ന വചനങ്ങള്‍ പരിശോധിക്കുക:

''ഖുലിഖ മിന്‍ മാഇന്‍ ദാഫിഖ്. യഖ്‌റുജു മിന്‍ ബൈനിസ്സ്വുല്‍ബി വ ത്തറാഇബ്''

''തെറിച്ചു വീഴുന്ന ഒരു ദ്രാവകത്തില്‍ നിന്നത്രെ അവന്‍ സൃഷ്ടിക്കപ്പെട്ടിരിക്കുന്നത്. സ്വുല്‍ബിനും, തറാഇബിനുമിടയില്‍ നിന്ന് അത് പുറത്തുവരുന്നു.''

ഈ വചനങ്ങൾക്ക് പ്രവാചകനോ(സ) അനുചരന്മാരോ വല്ല വ്യാഖ്യാനവും നൽകിയിട്ടുണ്ടോ? ഉണ്ടെങ്കിൽ എന്താണ് ? അതാണ് ആദ്യം പരിശോധിക്കേണ്ടത്. സൂറത്തുത്വാരിഖിലുള്ള 'ബൈന സ്വുല്‍ബി വത്തറാഇബി' എന്ന പ്രയോഗത്തിന് 'പുരുഷന്റെ സ്വുല്‍ബില്‍നിന്നും സ്ത്രീയുടെ തറാഇബുകളില്‍നിന്നും എന്ന അര്‍ഥമാണ് ആദ്യകാല ഖുര്‍ആന്‍ വ്യാഖ്യാതാക്കളില്‍ മിക്കവരും നല്‍കിയിട്ടുള്ളത്. സ്വഹാബിമാരില്‍ നിന്നുള്ള ഖുര്‍ആന്‍ വ്യാഖ്യാതാക്കളില്‍ പ്രമുഖനായ ഇബ്‌നു അബ്ബാസ്(റ) ഈ വചനത്തിന് നല്‍കിയ വ്യാഖ്യാനം ''സ്വുല്‍ബുര്‍റജുലി വതറാഇബുല്‍ മര്‍അത്തി വല്‍ വലദു ലാ യകൂനു ഇല്ലാ മിനല്‍ മാഅയ്‌നി'' എന്നാണെന്ന് ഇമാം ത്വബ്‌റാനി ഉദ്ധരിച്ചതായി കാണാം. 'പുരുഷന്റെ സ്വുല്‍ബില്‍ നിന്നും സ്ത്രീയുടെ തറാഇബില്‍ നിന്നുമുള്ള രണ്ട് ദ്രാവകങ്ങളില്‍ നിന്നുമായാണ് കുഞ്ഞുണ്ടാവുന്നത്' എന്നര്‍ഥം. മറ്റൊരു സ്വഹാബിയായ ഇക്‌രിമ (റ)വും ഇതേ അര്‍ഥം പറഞ്ഞതായി ഇമാം ത്വബ്‌രി തന്റെ തഫ്‌സീറില്‍ ഉദ്ധരിക്കുന്നുണ്ട്. സ്വഹാബിമാരുടെ അഭിപ്രായങ്ങളുടെ വെളിച്ചത്തില്‍, പ്രസിദ്ധ ഖുര്‍ആന്‍ വ്യാഖ്യാതാക്കളായ ത്വബ്‌രി, സമഖ്ശരി, ത്വബ്‌റാനി, റാസി, ഖുര്‍തുബി, ഇബ്‌നു കഥീര്‍, ജലാലൈനി, ശൗക്കാനി തുടങ്ങിയവരെല്ലാം 'പുരുഷന്റെ സ്വുല്‍ബില്‍ നിന്നും സ്ത്രീയുടെ തറാഇബില്‍ നിന്നുമുള്ള ദ്രാവകങ്ങളുടെ സംയോജനത്തിൽ നിന്നാണ് കുഞ്ഞുണ്ടാവുന്നത് എന്നാണ് ഈ ആയത്തുകള്‍ക്ക് നല്‍കിയ വ്യാഖ്യാനം. പ്രവാചകനിൽ നിന്ന് നേർക്ക് നേരെ ഖുർആൻ വ്യാഖ്യാനം മനസ്സിലാക്കിയ ഇബ്നു അബ്ബാസും ഇക്‌രിമയും (റ) മനസ്സിലാക്കിയത് സ്ത്രീയുടെ സ്രവവും പുരുഷന്റെ സ്രവവും കൂട്ടിച്ചെർന്നാണ് കുഞ്ഞുണ്ടാവുന്നതെന്ന വസ്തുതയാണ് ഈ വചങ്ങൾ പഠിപ്പിക്കുന്ന ആശയം എന്നാണ് ഇത് വ്യക്തമാക്കുന്നത്.

എന്താണ് സ്വുല്‍ബ് ? സ്വാദ്, ലാമ്, ബാഅ് തുടങ്ങിയ മൂലാക്ഷരങ്ങളില്‍നിന്ന് നിഷ്പന്നമായ സ്വുല്‍ബ് എന്ന പദം ക്രിയാരൂപത്തില്‍ വരുമ്പോള്‍ ഉറച്ചതായിത്തീരുക, ശക്തമാവുക, മാറ്റമില്ലാതാവുക എന്നെല്ലാമാണ് അർഥം. സ്വുല്‍ബ് എന്ന് നട്ടെല്ലിനും (back bone) അരക്കെട്ടിനും (loins) പറയാറുണ്ട്. Loins എന്ന ഇംഗ്ലീഷ് പദത്തിന് The Concise Oxford Arabic-English Dictionary യും Hippocrene Standard Dictionary (Arabic-English) യും നല്‍കുന്ന അറബി അര്‍ഥം സ്വുല്‍ബ് എന്നാണ്. അടിയിലെ വാരിയെല്ലുകള്‍ക്കും (lower ribs) വസ്തിപ്രദേശത്തിനു (pelvis) മിടയ്ക്കുള്ള ശരീരഭാഗമാണ് അരക്കെട്ട് അഥവാ കടിപ്രദേശം (loins). 'സ്വുല്‍ബ്' എന്ന ഖുര്‍ആനിക പ്രയോഗത്തിന് (loin) എന്ന് അര്‍ഥം പറഞ്ഞവരാണ് മിക്ക ഇംഗ്ലീഷ് പരിഭാഷകരുമെന്നതാണ് വസ്തുത. ഇതില്‍ ആദ്യകാല ഇംഗ്ലീഷ് പരിഭാഷകനും ഓറിയന്റലിസ്റ്റുമായ ജോര്‍ജ് സെയിലും ഉള്‍പ്പെടുന്നു. ക്രിസ്തുമതത്തില്‍നിന്ന് ഇസ്‌ലാം സ്വീകരിച്ച ഖുര്‍ആന്‍ പരിഭാഷകനായ മുഹമ്മദ് മാര്‍മഡ്യൂക് പിക്താളും ജൂതമതത്തില്‍ നിന്ന് ഇസ്‌ലാം സ്വീകരിച്ചശേഷം ഖുര്‍ആന്‍ പരിഭാഷപ്പെടുത്തിയ മുഹമ്മദ് അസദും ബ്രിട്ടീഷ് ഓറിയന്റലിസ്റ്റായ ഖുര്‍ആന്‍ പരിഭാഷകന്‍ ആര്‍തര്‍ ജോണ്‍ ആര്‍ബെറിയുമെല്ലാം സ്വുല്‍ബിന് നല്‍കിയിട്ടുള്ള അര്‍ഥം loins എന്നു തന്നെയാണ്.

മനുഷ്യരെക്കുറിച്ച് 'പുരുഷന്റെ അരക്കെട്ടില്‍ നിന്ന് ഉണ്ടാകുന്നവര്‍' എന്ന അര്‍ഥത്തില്‍ 'സ്വുല്‍ബില്‍ നിന്നുള്ളവര്‍' എന്ന പ്രയോഗം മധ്യപൂര്‍വദേശത്ത് വ്യാപകമായിരുന്നു. ഇക്കാര്യം ബൈബിളില്‍ നിന്ന് വ്യക്തമായി മനസ്സിലാകുന്നുണ്ട്. ഒരു ബൈബിള്‍ വചനം കാണുക: ''ദൈവം പിന്നെയും അവനോട് ഞാന്‍ സര്‍വശക്തിയുള്ള ദൈവമാകുന്നു. നീ സന്താനപുഷ്ടിയുള്ളവനായി പെരുകുക. ഒരു ജാതിയും ജാതികളുടെ കൂട്ടവും നിന്നില്‍ നിന്ന് ഉല്‍ഭവിക്കും. രാജാക്കന്മാരും നിന്റെ അരക്കെട്ടിൽ നിന്ന് പുറപ്പെടും.''(ഉല്‍ 35:11)

മക്കളെക്കുറി ച്ച് 'അരക്കെട്ടിൽ നിന്നുണ്ടാവുന്നവർ' എന്ന പ്രയോഗം രാജാക്കന്മാർ 8:19, 2 ദിനവൃത്താന്തം 6:9 പുറപ്പാട് 1:5 എന്നീ പഴയനിയമ ഉദ്ധരണികളിൽ കാണാം. പുതിയനിയമത്തിലെ അപ്പോസ്തലപ്രവൃത്തികള്‍2:30, എബ്രായര്‍7:5, എബ്രായര്‍7:10 എന്നീ വചനങ്ങളിലും സമാനമായ പ്രയോഗങ്ങൾ കാണാം. ഉല്‍പത്തി 35:11ലെ "നിന്റെ അരക്കെട്ടിൽ നിന്ന് പുറപ്പെടും" എന്ന പ്രയോഗത്തിന് അറബി ബൈബിളില്‍ നല്‍കപ്പെട്ടിരിക്കുന്ന പരിഭാഷ 'മിന്‍ സ്വുല്‍ബിക്ക' എന്നാണ്. 'നിന്റെ കടിപ്രദേശത്തുനിന്നുണ്ടാകുന്നവര്‍' എന്ന അര്‍ഥത്തില്‍ 'മിന്‍ സ്വുല്‍ബിക്ക' എന്ന പ്രയോഗം അറബികള്‍ക്കിടയിലും തത്തുല്യമായ പ്രയോഗങ്ങള്‍ മധ്യപൂര്‍വപ്രദേശത്തെ മറ്റു ഭാഷക്കാര്‍ക്കിടയിലും വ്യാപകമായിരുന്നുവെന്നാണ് ഇത് വ്യക്തമാക്കുന്നത്.

ഇത്തരത്തിലുള്ള ചില പ്രയോഗങ്ങള്‍ ഖുര്‍ആനിലും ഹദീഥുകളിലുമെല്ലാം കാണാന്‍ കഴിയും. ഖുര്‍ആന്‍ സൂറത്തുന്നിസാഇലെ ഇരുപത്തിമൂന്നാമത്തെ വചനത്തിലും സ്വഹീഹു മുസ്‌ലിം, കിത്താബുല്‍ ഖദ് റിൽ ആയിഷയിൽ (റ) നിന്ന് നിവേദനം ചെയ്ത ഹദീഥിലും സമാനമായ പ്രയോഗമാണ് കാണാം. സ്വുല്‍ബിന്റെ ബഹുവചനമായ അസ്വ്്‌ലാബ് എന്നാണ് ഇവിടെ പ്രയോഗിച്ചിരിക്കുന്നത്. ഇവയിലെ 'മിന്‍ അസ്വ്്‌ലാബിക്കും' എന്ന പ്രയോഗത്തിന് 'നിങ്ങളുടെ അണക്കെട്ടുകളിൽ നിന്നുള്ള ' എന്ന അര്‍ഥമാണുള്ളത്. ഖുര്‍ആനിന്റെ അവതരണകാലത്ത് മക്കളെ ഉദ്ദേശിച്ചുകൊണ്ട് 'സ്വുല്‍ബില്‍ നിന്നുള്ളവര്‍' എന്ന പ്രയോഗം വ്യാപകമായിരുന്നുവെന്ന വസ്തുതയാണിത് കാണിക്കുന്നത്. സ്വുല്‍ബ് എന്ന പദത്തിന് ഉറച്ചത്, നട്ടെല്ല്, കടിപ്രദേശം തുടങ്ങിയ അര്‍ഥങ്ങളുണ്ടെന്നും, മനുഷ്യജനനവുമായി ബന്ധപ്പെട്ട് പ്രയോഗിക്കുമ്പോള്‍ 'സ്വുല്‍ബില്‍ നിന്ന്' എന്ന പ്രയോഗം ഖുര്‍ആനിന്റെ അവതരണത്തിന് മുമ്പുതന്നെ മധ്യപൂര്‍വദേശത്ത് വ്യാപകമായിരുന്നുവെന്നും ഈ പ്രയോഗത്തിന് അവര്‍ അര്‍ഥമാക്കിയത് അരക്കെട്ടിൽ നിന്ന് എന്നാണെന്നുമുള്ള വസ്തുതകളാണ് ഇതിൽ നിന്നെല്ലാം മനസ്സിലാവുന്നത് സൂറത്തു ത്വാരിഖിലെ ഏഴാമത്തെ വചനത്തിലെ 'സ്വുല്‍ബി'നും അരക്കെട്ട് (loins) എന്ന അര്‍ഥം തന്നെയാണ് പ്രമുഖരായ ഇംഗ്ലീഷ് പരിഭാഷകരെല്ലാം നല്‍കിയിരിക്കുന്നത്. ആദ്യകാല ഖുർആൻ വ്യാഖ്യാതാക്കളും നൽകിയ അർഥം അത് തന്നെ!

എന്താണ് തറാഇബ്? ത,റ,ബ എന്നീ മൂലാക്ഷരങ്ങളില്‍ നിന്ന് നിഷ്പന്നമായ 'തരീബത്തി'ന്റെ ബഹുവചനമായ 'തറാഇബി'നെയാണ് ഈ വചനത്തില്‍ വാരിയെല്ലുകള്‍ എന്ന് പരിഭാഷപ്പെടുത്താറുള്ളത്. ഇംഗ്ലീഷ് പരിഭാഷകളിൽ തറാഇബി'ന് breast bone എന്നും ribs എന്നും അര്‍ഥം നല്‍കിയിട്ടുണ്ട്.. ഇതില്‍നിന്ന് വ്യത്യസ്തമായി മുഹമ്മദ് അസദ് നല്‍കിയിട്ടുള്ള പരിഭാഷ pelvic arch എന്നാണ്; ഇത് സ്ത്രീ ശരീരത്തെ കുറിക്കുന്ന പദപ്രയോഗമാണെന്നാണ് ഖുര്‍ആനില്‍ ഉപയോഗിക്കപ്പെട്ടിരിക്കുന്ന അസാധാരണ പ്രയോഗങ്ങളെക്കുറിച്ച് ഗവേഷണം നടത്തിയിട്ടുള്ളവരുടെ അഭിപ്രായമെന്നും മുഹമ്മദ് അസദ് രേഖപ്പെടുത്തുന്നുണ്ട്.

തറാഇബ് എന്ന പ്രയോഗം അറബിസാഹിത്യങ്ങളില്‍ സാധാരണയായി കാണപ്പെടാത്തതാണ്. ഖുര്‍ആനില്‍ ഒരു തവണ മാത്രമെ ഇങ്ങനെ പ്രയോഗിച്ചിട്ടുള്ളൂ. പ്രവാചകവചനങ്ങളില്‍ വ്യാപകമായി 'തറാഇബ്' എന്നോ അതിന്റെ ഏകവചനരൂപമായ 'തരീബത്ത്' എന്നോ പ്രയോഗിക്കപ്പെട്ടിട്ടില്ല. ഖുര്‍ആനിനു മുമ്പുള്ള കവിതകളില്‍നിന്ന് ഇംറുല്‍ ഖൈസിന്റെ ഒരു കവിതയെ ഉദ്ധരിച്ചുകൊണ്ടാണ് ഇത് സ്ത്രീയുടെ ശരീരഭാഗങ്ങളെ കുറിക്കുവാന്‍ മാത്രമുപയോഗിക്കുന്നതാണെന്ന് ലിസാനുന്‍ അറബ്, താജുല്‍ അറൂസ് തുടങ്ങിയ ആധികാരിക അറബിഭാഷാ നിഘണ്ടുകള്‍ അഭിപ്രായപ്പെടുന്നത്. ആധുനിക അറബിഭാഷാ നിഘണ്ടുകളില്‍ പലതിലും ഈ പദം തന്നെയില്ല. വാരിയെല്ലിനെ കുറിക്കുന്നതിന് 'ള്വില്‍അ്' എന്നും അതിന്റെ ബഹുവചനമായി 'ള്വുലൂഅ്' എന്നുമാണ് അവയിലുള്ളത്. 'സ്വുല്‍ബി'നെപ്പോലെ വ്യാപകമായ പ്രയോഗത്തിലുള്ളതോ സാഹിത്യങ്ങളില്‍ നിറഞ്ഞുനില്‍ക്കുന്നതോ ആയ പദമല്ല 'തറാഇബ്' എന്നര്‍ഥം.

വാരിയെല്ലുകള്‍ക്ക് മാത്രമാണോ ഇങ്ങനെ പ്രയോഗിക്കാറുള്ളത്? അല്ലെന്ന വസ്തുത പുരാതന അറബി നിഘണ്ടുകളെല്ലാം സാക്ഷ്യപ്പെടുത്തുന്നുണ്ട്. സ്തനങ്ങള്‍, സ്തനങ്ങള്‍ക്കിടയിലുള്ള സ്ഥലം, നെഞ്ചെല്ല്, മാറിടം, വാരിയെല്ലുകളില്‍ താഴത്തെ നാലെണ്ണം, സ്തനങ്ങള്‍ക്കും, പൂണെല്ലിനു (collar bone) മിടയിലുള്ള സ്ഥലം, പൂണെല്ലിനടുത്തുള്ള വാരിയെല്ലുകള്‍, രണ്ടു കൈകളും രണ്ടുകാലുകളും രണ്ട് കണ്ണുകളും ഇങ്ങനെ വിവിധ അര്‍ഥങ്ങളില്‍ തറാഇബ് എന്ന് പ്രയോഗിക്കാമെന്നാണ് നിഘണ്ടുകള്‍ വ്യക്തമാക്കുന്നത്. വില്യം ലെയിനിന്റെ പ്രസിദ്ധമായ ലെക്‌സിക്കണില്‍ നല്‍കുന്ന അര്‍ഥങ്ങളും ഇങ്ങനെത്തന്നെയാണ്

എന്താണ് 'തറാഇബ്' എന്ന വിഷയത്തില്‍ ആദ്യകാലം മുതല്‍ക്കേ പണ്ഡിതന്‍മാര്‍ക്ക് അഭിപ്രായവ്യത്യാസമുണ്ടായിരുന്നുവെന്ന വസ്തുത ഇമാം റാസി തന്റെ തഫ്‌സീറില്‍ വ്യക്തമാക്കിയിട്ടുണ്ട്. എന്നാല്‍ സ്വഹാബിമാരും ആദ്യകാല വ്യാഖ്യാതാക്കളുമെല്ലാം ഇത് സ്ത്രീയുടെ ശരീരഭാഗമാണെന്ന നിലയ്ക്കു തന്നെയാണ് കാര്യങ്ങള്‍ വിശദീകരിച്ചിരിക്കുന്നത്. സ്ത്രീയുടെ കഴുത്തിനു താഴെയുള്ള ശാരീരികാവയവങ്ങളിലേതോ ഒന്നാണ് തറാഇബു കൊണ്ട് വിവക്ഷിച്ചത് എന്ന് ഇവയില്‍നിന്ന് മനസ്സിലാക്കാം.

സ്വുല്‍ബ്, തറാഇബ് എന്നീ പദങ്ങളെക്കുറിച്ച പഠനത്തെ ഇങ്ങനെ സംഗ്രഹിക്കാം:

1) ബലിഷ്ഠമായത്, നട്ടെല്ല്, അരക്കെട്ട്, എന്നിങ്ങനെ അര്‍ഥങ്ങളുള്ള അറബിപദമാണ് സ്വുല്‍ബ്. പുരുഷന്റെ സ്വുല്‍ബില്‍നിന്നാണ് കുഞ്ഞുങ്ങളുണ്ടാകുന്നെതന്ന മധ്യപൂര്‍വദേശത്ത് നൂറ്റാണ്ടുകളായി നിലനിന്ന പ്രയോഗം അവന്റെ അരക്കെട്ടിനെ ഉദ്ദേശിച്ചുകൊണ്ടുള്ളതാണ്. അരക്കെട്ടിലുള്ള വൃഷണമായിരിക്കണം സൂറത്തുത്വാരിഖിലെ സ്വുല്‍ബു കൊണ്ടുള്ള വിവക്ഷ.

2) വാരിയെല്ലുകള്‍, സ്തനങ്ങള്‍, കൈകാലുകള്‍, വസ്തികമാനം എന്നിങ്ങനെയുള്ള അര്‍ഥങ്ങളില്‍ പ്രയോഗിക്കപ്പെട്ടിരുന്ന അപൂര്‍വപദങ്ങളിലൊന്നാണ് 'തറാഇബ്'. അറബി ഭാഷാകാരന്‍മാരും ആദ്യകാല ഖുര്‍ആന്‍ വ്യാഖ്യാതാക്കളുടെയും അഭിപ്രായപ്രകാരം ഈ പദപ്രയോഗം മാലയിടുന്ന ഭാഗത്തുള്ള ഏതോ പെണ്ണവയവത്തെക്കുറിക്കുന്നതാണ്. അന്ധരാശയം സ്ഥിതിചെയ്യുന്ന ഭാഗത്തെക്കുറിച്ച കൃത്യമായ പ്രയോഗമാണിത്.

ഇനി പ്രവാചകനിൽ (സ) നിന്ന് ക്വുർആൻ പഠിച്ച ഇബ്നു അബ്ബാസും ഇക്രിമയുമെല്ലാം മനസ്സിലാക്കിയ രൂപത്തിൽ സൂറത്തുത്വാരികിലെ വചനങ്ങളുടെ മലയാള അർഥം ഇങ്ങനെ എഴുതാം:

''തെറിച്ചു വീഴുന്ന ദ്രാവകത്തില്‍ നിന്നാണ് അവന്‍ സൃഷ്ടിക്കപ്പെട്ടിരിക്കുന്നത്. പുരുഷന്റെ അരക്കെട്ടിൽ നിന്നും സ്ത്രീയുടെ മാലയിടുന്ന ഭാഗത്ത് നിന്നുമായി അത് പുറത്ത് വരുന്നു."

പ്രവാചകനിൽ നിന്ന് സ്വഹാബിമാരും അവരിൽ നിന്ന് താബിഉകളുമെല്ലാം ഈ അർത്ഥമാണ് മനസ്സിലാക്കിയതെങ്കിൽ പിന്നെയെങ്ങനെയാണ് നട്ടെല്ലിനും വാലിയെല്ലിനുമിടയിൽ നിന്ൻ പുറത്തുവരുന്ന ശുക്ലത്തിൽ നിന്നാണ് കുഞ്ഞുണ്ടാവുന്നതെന്നാണ് ഈ വചനം അര്ഥമാക്കുന്നതെന്ന വ്യാഖ്യാനമുണ്ടായത് എന്ന ചോദ്യം പ്രസക്തമാണ്. സ്വഹാബിമാരുടെ വ്യാഖ്യാനത്തിൽ നിന്ന് വ്യത്യസ്തമായി പുരുഷാവയവത്തിനും 'തറാഇബ്' എന്നു പറയാമെന്നും ശുക്ളത്തെക്കുറിച്ച് മാത്രമായിരിക്കണം ഈ വചനങ്ങളിൽ പറഞ്ഞിരിക്കുന്നതെന്നും വ്യാഖ്യാനിച്ച പണ്ഡിതന്‍മാരുമുണ്ട്. ഇബ്‌നുല്‍ഖയ്യിം (റഹ്) അവരില്‍ പ്രധാനിയാണ്. പുരുഷന്റെ തന്നെ സ്വുല്‍ബിനും തറാഇബിനുമിടയില്‍നിന്ന് പ്രവഹിക്കുന്ന ശുക്ലദ്രാവകത്തെ കുറിച്ചാണ് സൂറത്തുത്വാരിഖിലെ പരാമര്‍ശമെന്നാണ് അദ്ദേഹത്തിന്റെ പക്ഷം. താബിഉകളില്‍പെട്ട ഖത്താദ (റഹ്) യില്‍നിന്നും ഇത്തരമൊരു അഭിപ്രായം ജലാലുദ്ദീന്‍ സുയൂത്വി (റഹ്) തന്റെ ദുര്‍റുല്‍ മന്‍ഥൂറില്‍ ഉദ്ധരിച്ചിട്ടുമുണ്ട്. സ്ത്രീയുടെ തറാഇബില്‍നിന്നും പുരുഷന്റെ സ്വുല്‍ബില്‍നിന്നും തെറിച്ചുവീഴുന്ന ദ്രാവകങ്ങളുടെ സംഗമത്തെക്കുറിച്ചാണ് സൂറത്തു ത്വാരിഖിലെ വചനങ്ങള്‍ വ്യക്തമാക്കുന്നതെന്ന വ്യാഖ്യാനത്തില്‍ സംശയം പ്രകടിപ്പിച്ചവര്‍ പ്രധാനമായും ഉന്നയിച്ചത് 'സ്ത്രീയുടെ ദ്രാവകം തെറിച്ചുവീഴുന്നതല്ലല്ലോ'യെന്ന ന്യായമായിരുന്നു.

സ്ത്രീയുടെ ദ്രാവകം കൊണ്ട് വിവക്ഷിക്കുന്നത് രതിബാഹ്യലീലകള്‍ നടക്കുമ്പോള്‍ പുറത്തുവരുന്ന ബര്‍ത്തോലിന്‍ സ്രവമോ (bartholin fluid) രതിമൂര്‍ച്ഛാ സമയത്ത് പാരായുറിത്രല്‍ നാളികളില്‍നിന്ന് (paraurethral ducts) പുറത്തുവരുന്ന ദ്രാവകമോ ആണെന്ന് ധരിച്ചവരായിരുന്നു ഈ വാദമുന്നയിച്ച വ്യാഖ്യാതാക്കള്‍. ഈ രണ്ട് ദ്രാവകങ്ങളും തെറിച്ചു വീഴുന്നതല്ലെന്ന് അവർ മനസ്സിലാക്കിയിരുന്നു. അണ്ഡോല്‍സര്‍ജനത്തെയോ ആ സമയത്തുണ്ടാവുന്ന ശാരീരിക മാറ്റങ്ങളെയോ കുറിച്ച അറിവ് അവര്‍ ഈ വാദമുന്നയിക്കുന്ന കാലത്ത് ഉണ്ടായിരുന്നില്ല. അതുകൊണ്ടുതന്നെ സ്ത്രീപുരുഷ ദ്രാവകങ്ങളുടെ സമന്വയത്തെപ്പറ്റിത്തന്നെയാണോ സ്വുല്‍ബില്‍നിന്നും തറാഇബില്‍നിന്നും പുറത്തുവരുന്ന ദ്രാവകത്തെകുറിച്ച് ഖുര്‍ആന്‍ പരാമര്‍ശിക്കുമ്പോള്‍ അര്‍ഥമാക്കുന്നതെന്ന് അവര്‍ സംശയിച്ചു. പുരുഷദ്രാവകം തെറിച്ചു വീഴുന്നതാണെന്ന് അവര്‍ക്ക് അറിയാമായിരുന്നു. എന്നാല്‍ സ്ത്രീയുടെ ദ്രാവകം തെറിച്ചുവീഴുന്നതല്ലെന്ന അന്നത്തെ അറിവിന്റെ വെളിച്ചത്തില്‍ ഈ വചനം പുരുഷബീജത്തെക്കുറിച്ചു മാത്രമാണെന്ന് വാദിക്കുകയാണ് അവര്‍ ചെയ്തത്. എന്നാല്‍ ഇന്നു നമുക്കറിയാം, ബര്‍ത്തോലിന്‍ സ്രവമോ പാരായുറിത്രല്‍ സ്രവമോ കുഞ്ഞിന്റെ പിറവിയില്‍ പങ്കാളിയാവുന്നില്ലെന്നും അതില്‍ പങ്കാളിയാവുന്നത് അണ്ഡാശയത്തിനകത്ത് ഉത്പാദിപ്പിക്കപ്പെടുന്ന അണ്ഡദ്രാവകം മാത്രമാണെന്നും.

ശുക്ലദ്രാവകത്തെപ്പോലെ അണ്ഡദ്രാവകവും തെറിച്ചു വീഴുന്നതാണോ? പെൺശരീരത്തിന്റെ ആന്തരിക ഭാഗത്ത് നടക്കുന്ന അണ്ഡോൽസർജനത്തെപ്പറ്റി ഈയടുത്ത കാലം വരെ നമുക്ക് കാര്യമായൊന്നും അറിയില്ലായിരുന്നു. ആന്തരികാവയവങ്ങളിൽ നടക്കുന്നതെന്താണെന്ന് ക്രത്യമായി മനസ്സിലാക്കാൻ ഇന്ന് മാർഗങ്ങളുണ്ട്. ആര്‍ത്തവചക്രത്തിന്റെ പതിനാലാം ദിവസം ഹൈപോതലാമസിന്റെ ഉദ്ദീപനത്തിന് വിധേയമാകുന്ന അണ്ഡാശയത്തിനകത്തെ പൂര്‍ണവളര്‍ച്ചയെത്തിയ ഫോളിക്കിളില്‍ പ്രത്യക്ഷപ്പെടുന്ന സ്റ്റിഗ്മയെന്ന ദ്വാരത്തിലൂടെ പ്രായപൂര്‍ത്തിയായ അണ്ഡത്തെ വഹിച്ചുകൊണ്ട് ഫോളിക്കുളാര്‍ ദ്രവവും ക്യുമുലസ് കോശങ്ങളും പുറത്തേക്ക് തെറിക്കുകയും അത് ഫലോപ്പിയന്‍ നാളിയുടെ അറ്റത്തുള്ള ഫിംബ്രയകളില്‍ പതിക്കുകയും ചെയ്യുന്ന പ്രക്രിയയാണ് അണ്ഡോല്‍സര്‍ജനം (ovulation). ഇങ്ങനെ ഉല്‍സര്‍ജിക്കപ്പെട്ടഅണ്ഡമാണ് പുരുഷബീജവുമായി സംയോജിക്കുന്നത്. ഇതെല്ലാം നടക്കുന്നത് വാരിയെല്ലിന്റെ കൂടിനുതാഴെ വസ്തികമാനത്തിന് മുകളിലായി സ്ഥിതിചെയ്യുന്ന ഗര്‍ഭാശയത്തിന്റെ രണ്ടറ്റത്തായി കാണപ്പെടുന്ന അണ്ഡാശയങ്ങളിലും അനുബന്ധ അവയവങ്ങളിലുമായാണ്. പുരുഷലിംഗത്തിൽ നിന്ന് ശുക്ലം തെറിച്ചു വീഴുന്നതുപോലെ ഫോളിക്യൂൾ പൊട്ടി അണ്ഡദ്രാവകം തെറിച്ചു വീഴുന്നത് ഇന്ന് നമുക്ക് കാണാൻ കഴിയും. കാണണമെന്നാഗ്രഹിക്കുന്നവർ ഈ ലിങ്ക് സന്ദർശിക്കുക: https://www.youtube.com/watch?v=dq3MdeSDDC4

വിമര്ശിക്കപ്പെട്ടുകൊണ്ടിരിക്കുന്ന വചനത്തിന് സ്വഹാബിമാർ നൽകിയ അർഥം നമുക്കൊന്ന് കൂടി വായിക്കാം:

'തെറിച്ചു വീഴുന്ന ദ്രാവകത്തില്‍ നിന്നാണ് അവന്‍ സൃഷ്ടിക്കപ്പെട്ടിരിക്കുന്നത്. പുരുഷന്റെ അരക്കെട്ടിൽ നിന്നും സ്ത്രീയുടെ മാലയിടുന്ന ഭാഗത്ത് നിന്നുമായി അത് പുറത്ത് വരുന്നു."

താഴെപറയുന്ന വസ്തുതകൾ ശ്രദ്ധിക്കുക:

1) രതിമൂര്‍ച്ഛയിലെത്തുമ്പോള്‍ അനുതപ്ത നാഡീവ്യവസ്ഥയുടെ ഉദ്ദീപനപ്രകാരം വൃഷണത്തില്‍നിന്ന് ബീജാണുക്കള്‍ ബീജവാഹിനിക്കുഴലിലൂടെ മുകളിലേക്ക് കയറി സ്ഖലനനാളിയിലെത്തുകയും അതോടടുത്തുള്ള പ്രോസ്റ്റേറ്റ്, സെമിനല്‍ വെസിക്കിളുകള്‍, കൗപേഴ്‌സ് ഗ്രന്ഥി എന്നിവയില്‍ നിന്ന് ഉല്‍പാദിപ്പിക്കപ്പെടുന്ന സ്രവങ്ങളുമായിച്ചേര്‍ന്ന് സ്ഖലനനാളിയില്‍ നിന്ന് ലിംഗനാളിയിലൂടെ പുറത്തേക്ക് തെറിക്കുകയും ചെയ്യുന്നു. നട്ടെല്ലിന്റെ വാലിന് (coccyx) മുന്‍പിലായാണ് സെമിനല്‍ വെസിക്കിളുകളും പ്രോസ്റ്റേറ്റും കൗപേഴ്‌സ് ഗ്രന്ഥിയും സ്ഖലനനാളിയുമെല്ലാം സ്ഥിതിചെയ്യുന്നത്. പുരുഷശുക്ലം പുറത്തേക്ക് തെറിക്കുന്നത് അരക്കെട്ടിലെ വ്യത്യസ്ത അവയവങ്ങളുടെ ഒന്നിച്ചുള്ള പ്രവര്‍ത്തനം വഴിയാണ്. കൃത്യമായും അരക്കെട്ടിൽ നിന്നുതന്നെയാണ് പുരുഷദ്രാവകം പുറത്തേക്ക് തെറിക്കുന്നത് സ്വുല്‍ബില്‍നിന്നുതന്നെ!

2) ആര്‍ത്തവചക്രത്തിന്റെ പതിനാലാം ദിവസം ഹൈപോതലാമസിന്റെ ഉദ്ദീപനത്തിന് വിധേയമാകുന്ന അണ്ഡാശയത്തിനകത്തെ പൂര്‍ണവളര്‍ച്ചയെത്തിയ ഫോളിക്കിളില്‍ പ്രത്യക്ഷപ്പെടുന്ന സ്റ്റിഗ്മയെന്ന ദ്വാരത്തിലൂടെ പ്രായപൂര്‍ത്തിയായ അണ്ഡത്തെ വഹിച്ചുകൊണ്ട് ഫോളിക്കുളാര്‍ ദ്രവവും ക്യുമുലസ് കോശങ്ങളും പുറത്തേക്ക് തെറിക്കുകയും അത് ഫലോപ്പിയന്‍ നാളിയുടെ അറ്റത്തുള്ള ഫിംബ്രയകളില്‍ പതിക്കുകയും ചെയ്യുന്നു. ഇത് നടക്കുന്നത് വാരിയെല്ലിന്റെ കൂടിനുതാഴെ വസ്തികമാനത്തിന് മുകളിലായി സ്ഥിതിചെയ്യുന്ന ഗര്‍ഭാശയത്തിന്റെ രണ്ടറ്റത്തായി കാണപ്പെടുന്ന അണ്ഡാശയങ്ങളിലും അനുബന്ധ അവയവങ്ങളിലുമായാണ്. ഖുര്‍ആന്‍ 'തറാഇബ്' എന്നു വിളിച്ച സ്ഥലത്തുവെച്ചുതന്നെയാണ് അണ്ഡദ്രാവകം (ovular fluid) അണ്ഡാശയത്തില്‍നിന്ന് പുറത്തേക്ക് തെറിക്കുന്നത്. കൃത്യമായും 'തറാഇബിൽ നിന്ന് തന്നെ !

ഇനി നാം ചിന്തിക്കുക. ക്വുർആനിനാണോ വിമര്ശകര്ക്കാനോ തെറ്റുപറ്റിയതെന്ന്...!!!
വിഷയവുമായി ബന്ധപ്പെട്ട വീഡിയോ

ഇണകളെ കുറിച്ച ഖുര്‍ആന്‍ വചനങ്ങളിൽ എല്ലാ വസ്തുക്കളും ഇണകളായാണ് സൃഷ്ടിക്കപ്പെട്ടത് എന്ന് പറയുന്നുണ്ടല്ലോ. ജീവനുള്ളവയും ഇല്ലാത്തവയുമെല്ലാം എലാ വസ്തുക്കളെയും എന്ന് പറഞ്ഞതിൽ ഉൾപ്പെടെണ്ടതാണ്. അജൈവവസ്തുക്കളിൽ എങ്ങനെയാണ് ഇണകളുണ്ടാവുക? സസ്യങ്ങളെല്ലാം ഇണകളായാണ് സൃഷ്ടിക്കപ്പെട്ടത് എന്ന് പറയുന്ന ക്വുർആനിന് അലൈംഗിക പ്രത്യുത്പാദനം നടക്കുന്ന സസ്യങ്ങളെക്കുറിച്ചറിയില്ലെന്ന് വ്യക്തമാണ്. പിന്നെയെങ്ങനെ ഖുർആൻ സസ്യങ്ങളെയെല്ലാം സൃഷ്ടിച്ച ദൈവത്തിൽ നിന്നുള്ളതാവും ?

ണകളായാണ് എല്ലാം സൃഷ്ടിക്കപ്പെട്ടതെന്ന് ക്വുർആൻ പറയുന്നുണ്ട്; എന്നാൽ എല്ലാം ഉണ്ടാകുന്നത് ഇണകൾ തമ്മിലുള്ള ബന്ധം വഴിയാണെന്ന് ഖുർആനിലെവിടെയും പറയുന്നില്ല. ഇണകൾ എന്ന് പറയുമ്പോഴേക്ക് അത് പ്രത്യുത്പാദനവുമായി ബന്ധപ്പെട്ട പരാമര്ശമാണെന്ന് തെറ്റിദ്ധരിക്കുന്നതു കൊണ്ടാണ് ഈ വിമര്ശനമുണ്ടാവുന്നത്.

ഇണകളെക്കുറിച്ച് പറയുന്ന ഒരു ക്വുർആൻ സൂക്തം നോക്കുക:

''എല്ലാ വസ്തുക്കളില്‍ നിന്നും ഈരണ്ട് ഇണകളെ നാം സൃഷ്ടിച്ചിരിക്കുന്നു. നിങ്ങള്‍ ആലോചിച്ച് മനസ്സിലാക്കുവാന്‍ വേണ്ടി.'' (51:49)

എല്ലാവസ്തുക്കളിലും ഇണകളുണ്ട് എന്ന വചനത്തെ ബാഹ്യമായി അപഗ്രഥിച്ചാല്‍ ജീവിവര്‍ഗങ്ങളിലും സസ്യജാലങ്ങളിലും പെട്ട ഇണകളെകുറിച്ചാകാം ഇതെന്ന് ആര്‍ക്കും മനസ്സിലാവും. മനുഷ്യരിലും സസ്യവര്‍ഗങ്ങളിലും എല്ലാം പെട്ട ഇണകളെകുറിച്ച് ഖുര്‍ആന്‍ പ്രത്യേകം എടുത്ത് പറയുന്നുമുണ്ട്.

''നിങ്ങള്‍ക്ക് വേണ്ടി ഭൂമിയെ തൊട്ടിലാക്കുകയും, നിങ്ങള്‍ക്ക് അതില്‍ വഴികള്‍ ഏര്‍പെടുത്തിത്തരികയും, ആകാശത്ത് നിന്ന് വെള്ളം ഇറക്കിത്തരികയും ചെയ്തവനത്രെ അവന്‍. അങ്ങനെ അത് വഴി വ്യത്യസ്ത തരത്തിലുള്ള സസ്യങ്ങളുടെ ഇണകൾ നാം (അല്ലാഹു) ഉല്‍പാദിപ്പിക്കുകയും ചെയ്തിരിക്കുന്നു.'' (20:53)

''നിങ്ങള്‍ക്ക് സമാധാനപൂര്‍വ്വം ഒത്തുചേരേണ്ടതിനായി നിങ്ങളില്‍ നിന്ന് തന്നെ നിങ്ങള്‍ക്ക് ഇണകളെ സൃഷ്ടിക്കുകയും, നിങ്ങള്‍ക്കിടയില്‍ സ്‌നേഹവും കാരുണ്യവും ഉണ്ടാക്കുകയും ചെയ്തതും അവന്റെ ദൃഷ്ടാന്തങ്ങളില്‍ പെട്ടതത്രെ. തീര്‍ച്ചയായും അതില്‍ ചിന്തിക്കുന്ന ജനങ്ങള്‍ക്ക് ദൃഷ്ടാന്തങ്ങളുണ്ട്.'' (30:21)

മനുഷ്യരില്‍ നിന്നുള്ള ഇണകളെ കുറിച്ച് പരാമര്‍ശിക്കുമ്പോള്‍ ഖുര്‍ആന്‍ വളരെ കൃത്യമായ ചില പ്രയോഗങ്ങള്‍ നടത്തുന്നുണ്ട്. ആണിനെയും പെണ്ണിനെയും വ്യവഛേദിക്കുന്നത് സ്രവിക്കപ്പെടുന്ന ബീജമാണെന്ന വസ്തുത ഖുര്‍ആന്‍ വ്യക്തമാക്കുന്നു.

''ആണ്‍, പെണ്‍ എന്നീ രണ്ട് ഇണകളെ അവനാണ് സൃഷ്ടിച്ചതെന്നും ഒരു ബീജം സ്രവിക്കപ്പെടുമ്പോള്‍ അതില്‍ നിന്ന്...'' (53:45,46)

പുരുഷ ബീജത്തിലെ ക്രോമോസോമുകളാണ് കുഞ്ഞിന്റെ ലിംഗനിര്‍ണയം നടത്തുന്നതിന്റെ അടിത്തറയായി വര്‍ത്തിക്കുന്നതെന്ന വസ്തുത ഇന്ന് നമുക്കറിയാം. പെണ്‍കോശങ്ങളില്‍ ലിംഗക്രോമോസോമായ x മാത്രമെ കാണൂ; ഒരേ തരത്തിലുള്ള രണ്ട് ക്രോമസോമുകള്‍. അതിന് ഊനഭംഗം നടന്നുണ്ടാവുന്ന ലിംഗ കോശത്തില്‍-അണ്ഡം-ഒരേയൊരു x ക്രോമസോം മാത്രമെയുണ്ടാവൂ. എന്നാല്‍ ആണ്‍ കോശങ്ങളില്‍ XY എന്നീ രണ്ട് ലിംഗ ക്രോമസോമുകളുമുണ്ടാവൂം. ഊനഭംഗത്തിലൂടെ പുംബീജങ്ങളുണ്ടാവുമ്പോള്‍ അതില്‍ പകുതി x ക്രോമസോം ഉള്‍ക്കൊള്ളുന്നതും പകുതി Y ക്രോമസോം ഉള്‍ക്കൊള്ളുന്നതുമായിരിക്കും. x ഉള്‍ക്കൊള്ളുന്ന ബീജമാണ് അണ്ഡവുമായി യോജിക്കുന്നതെങ്കില്‍ അതുമൂലമുണ്ടാകുന്ന സിക്താണ്ഡം വളര്‍ന്ന് പെണ്‍കുട്ടിയും Y ക്രോമസോം ഉള്‍ക്കൊള്ളുന്ന ബീജവുമായാണ് അണ്ഡവുമായി സങ്കലിക്കുന്നതെങ്കില്‍ അത് ആണ്‍കുട്ടിയുമായിത്തീരുമെന്നതാണ് പൊതുവായ അവസ്ഥ. സ്രവിക്കപ്പെടുന്ന ബീജത്തില്‍ നിന്നാണ് ആണ്‍, പെണ്‍ തുടങ്ങിയ ഇണകളുണ്ടായിത്തീരുന്നതെന്ന ഖുര്‍ആനിക പരാമര്‍ശം എത്ര കൃത്യം! സൂക്ഷ്മം! ''ആണ്‍, പെണ്‍ എന്നീ രണ്ട് ഇണകളെ അവനാണ് സൃഷ്ടിച്ചതെന്നും ഒരു ബീജം സ്രവിക്കപ്പെടുമ്പോള്‍ അതില്‍ നിന്ന്...'' (53:45,46)

കൂറേക്കൂടി സൂക്ഷ്മമായ പരിശോധനയില്‍ ഓരോ തവണ സ്രവിക്കപ്പെടുന്ന ബീജങ്ങളെയും നമുക്ക് ആണ്‍ ബീജങ്ങളായും പെണ്‍ബീജങ്ങളായും വിഭജിക്കുവാനാകുമെന്ന് ബോധ്യപ്പെടുന്നു. x ക്രോമസോം ഉള്‍ക്കൊള്ളുന്നവ പെണ്‍ബീജങ്ങള്‍; Y ക്രോമസോം ഉള്‍കൊള്ളുന്നവ ആണ്‍ബീജങ്ങള്‍. സ്രവിക്കപ്പെടുന്ന ബീജത്തില്‍ തന്നെ ആണ്‍, പെണ്‍ എന്നീ രണ്ടു തരം ഇണകളുമുണ്ടെന്ന ഖുര്‍ആനിക പരാമര്‍ശം വളരെ കൃത്യമാണെന്ന് സൂക്ഷ്മ പരിശോധനയില്‍ തെളിയുന്നു.

നടേ ഉദ്ധരിച്ച ഇണകളെ കുറിച്ച് പരാമര്‍ശിക്കുന്ന ഖുര്‍ആന്‍ സൂക്തം പരിശോധിക്കുക. ''എല്ലാ വസ്തുക്കളില്‍ നിന്നും ഈരണ്ട് ഇണകളെ നാം സൃഷ്ടിച്ചിരിക്കുന്നു. നിങ്ങള്‍ ആലോചിച്ച് മനസ്സിലാക്കുവാന്‍ വേണ്ടി.'' (51:49).

എല്ലാ വസ്തുക്കളില്‍ നിന്നും ഈരണ്ട് ഇണകളെ സൃഷ്ടിച്ചിരിക്കുന്നുവെന്നാണ് ഖുര്‍ആന്‍ ഇവിടെ പറയുന്നത്. വസ്തുകളെല്ലാം നിര്‍മിക്കപ്പെട്ടിരിക്കുന്നത് ആറ്റങ്ങളെ കൊണ്ടാണെന്ന് ഇന്ന് നമുക്കറിയാം. എന്തുകൊണ്ടാണ് ആറ്റങ്ങള്‍ നിര്‍മിക്കപ്പെട്ടിരിക്കുന്നത്? പോസിറ്റീവ് ചാര്‍ജുള്ള ന്യൂക്ലിയസിന് പുറത്ത് പിടികൊടുക്കാതെ തെന്നിമാറിക്കൊണ്ടിരിക്കുന്ന ഇലക്‌ട്രോണ്‍ മേഘപടലമാണ് ആറ്റമെന്ന ചിത്രമാണ് ക്വാണ്ടം ബലതന്ത്രത്തിന്റേത്. പോസിറ്റീവ് ചാര്‍ജുള്ള പ്രോട്ടോണുകളും അതിനു തുല്യമായ എണ്ണം നെഗറ്റീവ് ചാര്‍ജുള്ള ഇലക്‌ട്രോണുകളും ചേര്‍ന്നാണ് ആറ്റത്തിന്റെ ഘടനയും സ്വഭാവങ്ങളുമെല്ലാം നിര്‍ണയിക്കുന്നത്. ഇലക്‌ട്രോണുകളും പ്രോട്ടോണുകളുമാകുന്ന ഇണകളുടെ പാരസ്പര്യമാണ് ആറ്റോമികലോകത്ത് നടക്കുന്നത്. ഖുര്‍ആന്‍ പറഞ്ഞതാണ് ശരി. എല്ലാ വസ്തുക്കളിലും പെട്ട ഇണകളെ സൃഷ്ടിച്ചവന്‍ എത്ര പരിശുദ്ധന്‍!

നമുക്ക് അറിയുന്നതും അറിയാത്തതുമായ വസ്തുകളെല്ലാം നിലനില്‍ക്കുന്നത് ഇണകളുടെ പാരസ്പര്യത്താലാണെന്നാണ് ഖുര്‍ആന്‍ നല്‍കുന്ന സൂചന. സൂറത്തു യാസീനിലെ ശ്രദ്ധേയമായ ഒരു വചനം ശ്രദ്ധിക്കുക. ''ഭൂമി മുളപ്പിക്കുന്ന സസ്യങ്ങളിലും, അവരുടെ സ്വന്തം വര്‍ഗങ്ങളിലും, അവര്‍ക്കറിയാത്ത വസ്തുക്കളിലും പെട്ട എല്ലാ ഇണകളെയും സൃഷ്ടിച്ചവന്‍ എത്ര പരിശുദ്ധന്‍!'' (36:36) ഈ വചനത്തിലെ 'അവര്‍ക്കറിയാന്‍ പറ്റാത്ത വസ്തുക്കളിലും പെട്ട എല്ലാ ഇണകളെയും സൃഷ്ടിച്ചവന്‍' എന്ന പരാമര്‍ശം ഏറെ ശ്രദ്ധേയമാണ്.

നമുക്ക് അറിയാവുന്നതും അല്ലാത്തതുമായ വസ്തുക്കളെല്ലാം സൃഷ്ടിക്കപ്പട്ടിട്ടുള്ളത് ഇണകളായിട്ടാണ് എന്ന വസ്തുതയാണ് ആറ്റോമിക് ഭൗതികം നമുക്ക് നല്‍കിക്കൊണ്ടിരിക്കുന്ന അറിവുകളിലൊന്ന്. ഇലക്‌ട്രോണ്‍, പ്രോട്ടോണ്‍ എന്നീ ഇണകളുടെ പാരസ്പര്യത്താലാണ് ആറ്റത്തിന്റെ നിലനില്‍പെന്ന് പറഞ്ഞുവല്ലോ. ന്യൂട്രോണുകളും പ്രോട്ടോണുകളും നിര്‍മിക്കപ്പെട്ടിരിക്കുന്നത് ആറു തരം ക്വാര്‍ക്കുകളെ കൊണ്ടാണ്. ഈ ക്വാര്‍ക്കുകളെ വേര്‍പിരിക്കുവാന്‍ സാധ്യമല്ല. ന്യൂട്രോണുകള്‍ക്കും പ്രോട്ടോണുകള്‍ക്കുമകത്തുള്ള ഓരോ ക്വാര്‍ക്കും അതിന്റെ ആന്റിക്വാര്‍ക്കുമായി പരസ്പരം ഇണചേര്‍ന്നു കിടക്കുകയാണ്. അവയെ വേര്‍പിരിക്കുവാനേ സാധ്യമല്ല. ഒരിക്കലും വേര്‍പിരിക്കാനാവാത്ത ഈ ഇണചേരലിനെയാണ് 'ഇന്‍ഫ്രാറെഡ് അടിമത്തം' (infrared slavery) അല്ലെങ്കില്‍ 'വര്‍ണപരിമിതപ്പെടുത്തല്‍' (colour confinement) എന്നു വിളിക്കുന്നത്. ക്വാര്‍ക്കുകള്‍ തമ്മിലുള്ള അതിശക്തമായ ഇണചേരലിനെ കുറിച്ച പഠനശാഖയാണ് ക്വാണ്ടം ക്രോമോഡൈനാമിക്‌സ് (quantum chromodynamics). അത് പഠിക്കുമ്പോൾ ഖുര്‍ആനിനോടൊപ്പം നമ്മളും പറഞ്ഞു പോകുന്നു, നമുക്കറിയാത്ത വസ്തുക്കളില്‍ പോലും ഇണകളെ സൃഷ്ടിച്ചവന്‍ എത്ര പരിശുദ്ധന്‍!

ഇങ്ങനെ, അറിയും തോറും എല്ലാ വസ്തുകളിലുമുള്ള ഇണകളെ പറ്റി നമുക്ക് കൂടുതല്‍ കൂടുതല്‍ മനസ്സിലാവുന്നു! ഇപ്പോള്‍ നടന്നുകൊണ്ടിരിക്കുന്ന ലാര്‍ജ് ഹൈഡ്രോണ്‍ കൊളൈഡര്‍ പരീക്ഷണം ഇത്തരമൊരു ഇണയെക്കൂടി തിരഞ്ഞുകൊണ്ടുള്ളതാണല്ലോ. പ്രപഞ്ചത്തെ വിശദീകരിക്കുവാന്‍ ഇന്ന് ഉപയോഗിക്കുന്ന സ്റ്റാന്റേര്‍ഡ് മോഡല്‍ പ്രകാരം, ശ്യാമഊര്‍ജത്തെയും ശ്യാമദ്രവ്യത്തെയും കുറിച്ച് കൃത്യമായി അറിയുവാന്‍ ഉപയോഗിക്കുന്ന സൂപ്പര്‍ സിമ്മട്രിയിലെ ഓരോ കണത്തിനുമുള്ള സൂപ്പര്‍ പങ്കാളികളെ (super partners) കണ്ടെത്തുകയാണല്ലോ ആയിരം കോടി ഡോളര്‍ ചെലവു ചെയ്തു നിര്‍മിച്ച എല്‍.എച്ച്.സി യുടെ ലക്ഷ്യങ്ങളിലൊന്ന്. വസ്തുക്കള്‍ നിര്‍മിക്കാനുപയോഗിക്കപ്പെട്ട കൂടുതല്‍ സൂക്ഷ്മമായ ഇണകളെ കുറിച്ച് ലോകം അറിഞ്ഞുകൊണ്ടിരിക്കുകയാണ്. അഗാധതകളിലേക്ക് പോകുമ്പോള്‍ ഇണകളുടെ പാരസ്പര്യമാണ് സൃഷ്ടിപ്രപഞ്ചത്തിലെ എല്ലാത്തിനും നിദാനമെന്ന് മാനവരാശി മനസ്സിലാക്കിക്കൊണ്ടിരിക്കുന്നു; ഒപ്പം തന്നെ ഒരിക്കലും തെറ്റുപറ്റാത്ത വചനങ്ങളാണ് ഖുര്‍ആനിലുള്ളതെന്നും.

എല്ലാ സസ്യങ്ങളെയും ഇണകളായി സൃഷ്ടിച്ചുവെന്ന ഖുര്‍ആനിക പരാമര്‍ശം കാണ്ഡം മുറിച്ച് നടുന്ന സസ്യങ്ങളുണ്ടെന്ന വസ്തുതയുടെ വെളിച്ചത്തില്‍ അബദ്ധമാണെന്നാണ് മറ്റൊരു വിമർശനം.

സസ്യങ്ങള്‍ക്കിടയില്‍ ഇണകളുണ്ടെന്ന് വ്യക്തമാക്കുന്ന ഖുര്‍ആന്‍ സൂക്തങ്ങളുടെ സാരം ഇങ്ങനെയാണ്. ''നിങ്ങള്‍ക്ക് വേണ്ട ഭൂമിയെ തൊട്ടിലാക്കുകയും, നിങ്ങള്‍ക്ക് അതില്‍ വഴികള്‍ ഏര്‍പെടുത്തിത്തരികയും, ആകാശത്ത് നിന്ന് വെള്ളം ഇറക്കിത്തരികയും ചെയ്തവനത്രെ അവന്‍. അങ്ങനെ അത് (വെള്ളം) മൂലം വ്യത്യസ്ത തരത്തിലുള്ള സസ്യങ്ങളുടെ ജോടികള്‍ നാം (അല്ലാഹു) ഉല്‍പാദിപ്പിക്കുകയും ചെയ്തിരിക്കുന്നു.'' (20:53)

''ഭൂമി മുളപ്പിക്കുന്ന സസ്യങ്ങളിലും, അവരുടെ സ്വന്തം വര്‍ഗങ്ങളിലും, അവര്‍ക്കറിയാത്ത വസ്തുക്കളിലും പെട്ട എല്ലാ ഇണകളെയും സൃഷ്ടിച്ചവന്‍ എത്ര പരിശുദ്ധന്‍!'' (36:36)

ഈ സൂക്തങ്ങളിലൊന്നും തന്നെ സസ്യങ്ങളിലെല്ലാം പ്രത്യുല്‍പാദനം നടക്കുന്നത് ഇണകള്‍ തമ്മിലുള്ള ലൈംഗികബന്ധം വഴിയാണെന്ന സൂചനകളൊന്നും തന്നെയില്ല. സസ്യങ്ങള്‍ക്കിടയില്‍ ഇണകളുണ്ടെന്ന് മാത്രമാണ് ഈ സൂക്തങ്ങള്‍ വ്യക്തമാക്കുന്നത്. ലൈംഗിക പ്രത്യുല്‍പാദനവും അലൈംഗിക പ്രത്യുല്‍പാദനവും സസ്യങ്ങള്‍ക്കിടയില്‍ നടക്കുന്നുണ്ടെന്ന വസ്തുതയെ ഈ വചനങ്ങള്‍ നിഷേധിക്കുന്നില്ല.

പൂക്കളാണ് സസ്യങ്ങളിലെ പ്രത്യുല്‍പാദന കേന്ദ്രം. രണ്ടുതരം പൂക്കളുണ്ട്. ഏകലിംഗികളും (unisexual) ദ്വിലിംഗികളും (bisexual). ആണ്‍ ലൈംഗികാവയവമായ കേസരങ്ങളോ (androecium) പെണ്‍ലൈംഗികാവയവമായ ജനിയോ (gynoecium) മാത്രമുള്ള പുഷ്പങ്ങളാണ് ഏകലിംഗികള്‍. ഒരേ പുഷ്പത്തില്‍ തന്നെ ഇവ രണ്ടുമുണ്ടെങ്കില്‍ അവയെ ദ്വിലിംഗികള്‍ എന്നും വിളിക്കുന്നു. കേസരങ്ങളിലെ പരാഗികളില്‍ (anther) നിന്ന് പരാഗം ജനിയില്‍ പതിക്കുമ്പോഴാണ് ബീജസങ്കലനം നടക്കുന്നത്. പരാഗം ജനിയില്‍ പതിക്കുന്ന പ്രക്രിയക്കാണ് പരാഗണം (pollination) എന്നു പറയുന്നത്. ഒരു പുഷ്പത്തിലെ പരാഗം അതേ പുഷ്പത്തിലെ ജനിയില്‍ പതിക്കുന്നതിന് സ്വയംപരാഗണം എന്നും മറ്റൊരു പുഷ്പത്തിലെ ജനിയില്‍ പതിക്കുന്നതിന് പരപരാഗണം എന്നും പറയുന്നു. ചില ചെടികള്‍ സ്വയം പരാഗണം നടത്തുന്നു; മറ്റു ചിലവ പരപരാഗണവും. ഇങ്ങനെ പരാഗണം നടത്തുന്ന ചെടികളില്‍ ചിലതിനെ കാണ്ഡത്തില്‍ നിന്ന് മാത്രമായി വളര്‍ത്തിയെടുക്കാന്‍ കഴിയും. മരച്ചീനിയും ചെമ്പരത്തിയും റോസാചെടിയുമെല്ലാം ഇതിന് ഉദാഹരണങ്ങളാണ്. ഇവയില്‍ പുഷ്പങ്ങളും അതില്‍ ലൈംഗികാവയവങ്ങളുമുണ്ട്. അവ തമ്മില്‍ പരാഗണം നടക്കുന്നുണ്ടെങ്കിലും കായുണ്ടാകുന്നതിന് അത് നിമിത്തമാകുന്നില്ല; അതിന് മറ്റുചില ധര്‍മങ്ങളാണുള്ളത്. മുറിച്ച് നട്ടുകൊണ്ട്, കാണ്ഡത്തില്‍ നിന്നാണ് പുതിയ ചെടിയുണ്ടാവുന്നത്. ചെടിയുണ്ടാവുന്നത് ലൈംഗിക പ്രത്യുല്‍പാദനം വഴിയല്ലെങ്കിലും ഇവയിലും പൂക്കളുണ്ട്, അവയില്‍ ആണവയവങ്ങളും പെണ്ണവയവങ്ങളുമുണ്ട്. അവയും ഇണകളായാണ് സ്ഥിതി ചെയ്യുന്നത് എന്ന് സാരം.

അലൈംഗിക പ്രത്യുല്‍പാദനം മാത്രം നടത്തിവരുന്ന ജീവികളായി വ്യവഹരിക്കപ്പെട്ടു പോന്നിരുന്ന അമീബയെപ്പോലുള്ള ജീവികളില്‍ പോലും ചില ലൈംഗിക പെരുമാറ്റങ്ങളുണ്ടെന്ന് ഈയിടെയായി ശാസ്ത്രജ്ഞന്മാര്‍ നിരീക്ഷിച്ചിട്ടുണ്ട്. ചില അമീബകള്‍ മറ്റു ചിലവയുടെ ഇണകളായി വര്‍ത്തിക്കുന്നുണ്ടത്രെ! എഡിന്‍ ബര്‍ഗ് സര്‍വകലാശാലയുടെ ഔദ്യോഗിക വെബ്‌സൈറ്റ് (www.ed.ac.uk) ഇക്കാര്യം സ്ഥിരീകരിച്ചുകൊണ്ടുള്ള പ്രബന്ധങ്ങള്‍ പ്രസിദ്ധീകരിച്ചത് കാണാം. എല്ലാം ഇണകളായാണ് സൃഷ്ടിക്കപ്പെട്ടത് എന്ന ഖുര്‍ആനിക പരാമര്‍ശത്തിന്റെ കൃത്യതയിലേക്കാണ് ഈ ഗവേഷണങ്ങളെല്ലാം വിരല്‍ചൂണ്ടുന്നത്.

സൂര്യനെ വിളക്കായും ചന്ദ്രനെ പ്രകാശമായും വിശേഷിപ്പിച്ച ക്വുർആൻ സൂര്യൻ പ്രക്സശസ്രോതസ്സാണെന്നും ചന്ദ്രൻ അതിന്റെ പ്രകാശം പ്രതിഫലിപ്പിക്കുകയാണ് ചെയ്യുന്നതെന്നുമാണ് വ്യക്തമാക്കുന്നതെന്നും കൃത്യമായ പരാമർശങ്ങളാണ് ക്വുർആൻ നടത്തുന്നതെന്ന് ഇതിന്റെ അടിസ്ഥാനത്തിൽ വാദിക്കുന്നത് ശുദ്ധ തട്ടിപ്പാനിന്നും യുക്തിവാദികൾ പറയുന്നു. ഖുര്‍ആന്‍ 33:45,46ല്‍ മുഹമ്മദ് നബിയെ വിളക്കായും (സിറാജ്) 24:35ല്‍ അല്ലാഹുവിനെ പ്രകാശമായും (നൂര്‍) ഉപമിച്ചിട്ടുണ്ട്. മുഹമ്മദ് നബിയാണ് പ്രകാശ സ്രോതസ്സെന്നും അല്ലാഹു അദ്ദേഹത്തിന്റെ പ്രകാശം പ്രതിഫലിപ്പിക്കുകയാണെന്നുമല്ലേ നടേ പറഞ്ഞ വ്യാഖ്യാനം അംഗീകരിച്ചാല്‍ വന്നു ചേരുക. യുക്തിവാദികളുടെ വിമര്ശനത്തെപ്പറ്റി എന്ത് പറയുന്നു?

സൂര്യനെ വിളക്കായും ചന്ദ്രനെ പ്രകാശമായും ക്വുർആൻ വിശേഷിപ്പിച്ചിട്ടുണ്ട്. സൂര്യൻ പ്രക്സശസ്രോതസ്സാണെന്ന് അതിനെ വിശേഷിപ്പിച്ച സിറാജ് എന്ന പദം തന്നെ വ്യക്തമാക്കുന്നുണ്ട്. വിളക്ക് എന്നാണ് ആ പദത്തിന്റെ നേർക്ക് നേരെയുള്ള അർഥം. എന്നാൽ ചന്ദ്രനെ വിശേഷിപ്പിച്ച നൂർ എന്ന പദത്തിന് നേർക്ക് നേരെ പ്രതിഫലിക്കപ്പെട്ട പ്രകാശം എന്ന അർത്ഥമില്ല; പ്രകാശം എന്ന് മാത്രമാണ് അതിന്റെ അർത്ഥം . ചന്ദ്രൻ സൂര്യന്റെ പ്രകാശം പ്രതിഫലിപ്പിക്കുകയാണ് ചെയ്യുന്നതെന്ന് നമുക്കറിയാം. ഏത് തരം പ്രകാശമായാലും അതിന് നൂർ എന്ന് പറയും; പ്രതിഫലിക്കപ്പെട്ടതാവട്ടെ അല്ലാത്തതാകട്ടെ. സൂര്യനെക്കുറിച്ച് മാത്രമേ സിറാജ് എന്ന് ക്വുർആൻ പ്രയോഗിച്ചിട്ടുള്ളൂവെന്നും ചന്ദ്രനെക്കുറിച്ച് നൂർ എന്നും മുനീർ എന്നുമാണ് പ്രയോഗിച്ചതെന്നുമുള്ള വസ്തുതകൾ വ്യക്തമാക്കുന്നത് പ്രകാശസ്രോതസ്സാണ് സൂര്യനെന്ന വസ്തുത അറിയാവുന്നവനിൽ നിന്നാണ് അത് അവതരിപ്പിക്കപ്പെട്ടിരിക്കുന്നത് എന്നാണ്. ഇതാണ് ഇവ്വിഷയകമായി ഇസ്‌ലാമിക പ്രബോധകർ പറയാറുള്ളത്. രാത്രിയിൽ കാണുന്ന ചന്ദ്രനെയാണ് വിളക്ക് എന്ന അർത്ഥത്തിൽ സിറാജ് എന്ന് വിളിക്കാൻ ഒരു മരുഭൂനിവാസിക്ക് അനുയോജ്യമെങ്കിലും ഖുർആൻ ഒരിക്കലും ചന്ദ്രനെ അങ്ങനെ വിളിക്കുന്നില്ലെന്നതാണ് അതിലെ പദപ്രയോഗങ്ങളിലെ കൃത്യതയും സൂക്ഷ്മതയും അങ്ങനെ ദൈവികതയും വ്യക്തമാക്കുന്നത്.

ഖുര്‍ആന്‍ 33:45,46ല്‍ മുഹമ്മദ് നബിയെ വിളക്കായും (സിറാജ്) 24:35ല്‍ അല്ലാഹുവിനെ പ്രകാശമായും (നൂര്‍) ഉപമിച്ചിട്ടുണ്ടെന്നും മുഹമ്മദ് നബിയാണ് പ്രകാശ സ്രോതസ്സെന്നും അല്ലാഹു അദ്ദേഹത്തിന്റെ പ്രകാശം പ്രതിഫലിപ്പിക്കുകയാണെന്നുമല്ലേ നടേ പറഞ്ഞ വിശദീകരണം അംഗീകരിച്ചാല്‍ വന്നു ചേരുകയെന്നുമാണ് വിമര്‍ശകർ ചോദിക്കുന്നത്.

ഉദ്ധരിക്കപ്പെട്ട ഖുര്‍ആന്‍ വചനങ്ങള്‍ പരിശോധിക്കുക. ''നബിയേ, തീര്‍ച്ചയായും നിന്നെ നാം ഒരു സാക്ഷിയും സന്തോഷവാര്‍ത്ത അറിയിക്കുന്നവനും, താക്കീതുകാരനും ആയിക്കൊണ്ട് നിയോഗിച്ചിരിക്കുന്നു. അല്ലാഹുവിന്റെ ഉത്തരവനുസരിച്ച് അവങ്കലേക്ക് ക്ഷണിക്കുന്നവനും, പ്രകാശം നല്‍കുന്ന ഒരു വിളക്കും ആയിക്കൊണ്ട്.'' (33:45,46)

''അല്ലാഹു ആകാശങ്ങളുടെയും ഭൂമിയുടെയും പ്രകാശമാകുന്നു. അവന്റെ പ്രകാശത്തിന്റെ ഉപമയിതാ: (ചുമരില്‍ വിളക്ക് വെക്കാനുള്ള) ഒരു മാടം അതില്‍ ഒരു വിളക്ക്. വിളക്ക് ഒരു സ്ഫടികത്തിനകത്ത്. സ്ഫടികം ഒരു ജ്വലിക്കുന്ന നക്ഷത്രം പോലെയിരിക്കുന്നു. അനുഗൃഹീതമായ ഒരു വൃക്ഷത്തില്‍ നിന്നാണ് അതിന് (വിളക്കിന്) ഇന്ധനം നല്‍കപ്പെടുന്നത്. അതായത് കിഴക്ക് ഭാഗത്തുള്ളതോ പടിഞ്ഞാറ് ഭാഗത്തുള്ളതോ അല്ലാത്ത ഒലീവ് വൃക്ഷത്തില്‍ നിന്ന്. അതിന്റെ എണ്ണ തീ തട്ടിയില്ലെങ്കില്‍ പോലും പ്രകാശിക്കുമാറാകുന്നു. (അങ്ങനെ) പ്രകാശത്തിന്‍മേല്‍ പ്രകാശം. അല്ലാഹു തന്റെ പ്രകാശത്തിലേക്ക് താന്‍ ഉദ്ദേശിക്കുന്നവരെ നയിക്കുന്നു. അല്ലാഹു ജനങ്ങള്‍ക്ക് വേണ്ടി ഉപമകള്‍ വിവരിച്ചുകൊടുക്കുന്നു. അല്ലാഹു ഏത് കാര്യത്തെപ്പറ്റിയും അറിവുള്ളവനത്രെ.'' (24:35)

ഈ രണ്ട് വചനങ്ങളും രണ്ട് സ്വതന്ത്ര വചനങ്ങളാണ്; ഒന്ന് മറ്റേതിന്റെ ബാക്കിയോ വിശദീകരണമോ അല്ല. സൂറത്തു അഹ്‌സാബിലെ 45,46 വചനങ്ങള്‍ മുഹമ്മദ് നബി(സ)യുടെ സവിശേഷതകള്‍ വിവരിക്കുകയാണ് ചെയ്യുന്നത്. അദ്ദേഹം സാക്ഷിയും സന്തോഷവാര്‍ത്ത അറിയിക്കുന്നവനും താക്കീതുകാരനും അല്ലാഹുവിന്റെ ഉത്തരവനുസരിച്ച് അവങ്കലേക്ക് ക്ഷണിക്കുന്നവനുമാണ്; അതോടൊപ്പംതന്നെ അദ്ദേഹം ജനങ്ങള്‍ക്ക് പ്രകാശം നല്‍കുന്നവനും സ്വയം തന്നെ പ്രകാശിക്കുന്നവനമാണ്. അത് ക്പന്റാണ് അദ്ദേഹത്തെ സിറാജന്‍ മുനീറാ എന്ന് വിളിച്ചിരിക്കുന്നത്. ഇതൊരു ഉപമാലങ്കാരമാണ്. മുഹമ്മദ് നബി (സ) സ്വയം പ്രകാശിക്കുന്ന വിളക്കാണ് എന്ന കാര്യത്തില്‍ സംശയമൊന്നുമില്ല. അദ്ദേഹത്തിന്റെ കര്‍മ്മങ്ങളും നിര്‍ദ്ദേശങ്ങളും അനുവാദങ്ങളുമെല്ലാം അവസാനനാളുവരെയുള്ള മനുഷ്യര്‍ക്കെല്ലാം വെളിച്ചമായിത്തീരുന്നവയാണ്. മുഹമ്മദ് നബി (സ)യെന്ന വിളക്കില്‍ നിന്ന് പുറപ്പെടുന്ന വെളിച്ചമാണ് സുന്നത്ത്. ഇസ്‌ലാമിന്റെ രണ്ടാമത്തെ പ്രമാണമാണത്. മുഹമ്മദ് നബി (സ) സ്വയം വിളക്കായിത്തീര്‍ന്നതല്ല, പ്രത്യുത അല്ലാഹു അദ്ദേഹത്തെ വിളക്കാക്കിത്തീര്‍ത്തതാണ്. സ്വന്തം ജീവിതത്തിന്റെ പ്രകാശത്തിലൂടെ അവസാനനാളുവരെയുള്ള മുഴുവന്‍ മനുഷ്യര്‍ക്കും വഴികാട്ടിയായിത്തീരുവാനുള്ള വിളക്ക്. കെട്ടുപോയ വിളക്കല്ല അദ്ദേഹം; പ്രകാശം നല്‍കികൊണ്ടിരിക്കുന്ന സജീവമായ വിളക്കാണ്-സിറാജന്‍ മുനീറാ. എത്ര സുന്ദരമായ ഉപമാലങ്കാരം!

അല്ലാഹുവിനെ പരിചയപ്പെടുത്തുന്ന അതിസുന്ദരമായ ഖുര്‍ആന്‍ വചനങ്ങളിലൊന്നാണ് സൂറത്തുന്നൂറിലെ 35ാമത്തെ വചനം. ഇതും ഒരു ഉപമാലങ്കാരമാണ്. ആകാശഭൂമികളുടെ പ്രകാശമാണ് അല്ലാഹു. പ്രപഞ്ചത്തിന് മുഴുവന്‍ വെളിച്ചം നല്‍കുന്ന അവന്റെ പ്രകാശം മറ്റേതെങ്കിലും സ്രോതസ്സില്‍ നിന്ന് വരുന്നതല്ല. അവന്‍തന്നെയാണ് വിളക്കും വിളക്കുമാടവും അത് വെച്ചിരിക്കുന്ന സ്ഫടികക്കൂടുമെല്ലാം. പ്രകാശത്തിനു മേല്‍ പ്രകാശമാണവന്‍. അവന്റെ പ്രകാശത്തിലേക്ക് ആളുകളെ നയിക്കുന്നതും അവന്‍തന്നെ. ഇവിടെ അല്ലാഹുവിനെ കേവല പ്രകാശത്തോടല്ല ഉപമിച്ചിട്ടുള്ളതെന്ന കാര്യം പ്രത്യേകം ശ്രദ്ധേയമാണ്. അവന്‍തന്നെയാണ് വിളക്കും വിളക്കുമാടവും സ്ഫടികക്കൂടുമെല്ലാം എന്ന് വ്യക്തമാക്കുകയും അവന്റെ പ്രകാശത്തിലേക്ക് അവന്‍ തന്നെയാണ് ജനങ്ങളെ നയിക്കുന്നതെന്ന് പഠിപ്പിക്കുകയും ചെയ്യുന്നതാണ് ഈ വചനം. അല്ലാഹുവിനെ എത്ര സുന്ദരമായാണ് ഈ ഉപമയിലൂടെ ഖുര്‍ആന്‍ പരിചയപ്പെടുത്തിയിരിക്കുന്നത്!

സൂറത്തുല്‍ അഹ്‌സാബിലെ വചനം മുഹമ്മദ് നബി(സ)യെയും സൂറത്തുന്നൂറിലെ വചനം അല്ലാഹുവിനെയും സ്വതന്ത്രമായി ഉപമാലങ്കാരത്തിലൂടെ പരിചയപ്പെടുത്തുകയാണ് ചെയ്യുന്നത്. മുഹമ്മദ്‌നബി (സ) വിളക്കും അദ്ദേഹത്തില്‍നിന്നു പുറപ്പെടുന്ന പ്രകാശം അല്ലാഹുവുമാണെന്ന് ഈ വചനങ്ങള്‍ സൂചിപ്പിക്കുന്നുപോലുമില്ല. സ്വയം പ്രകാശിച്ചുകൊണ്ട് മനുഷ്യര്‍ക്ക് വെളിച്ചമാകുവാന്‍ അല്ലാഹു നിയോഗിച്ചതാണ് മുഹമ്മദ് നബി (സ)യെയെന്ന് ഒന്നാമത്തെ വചനവും പ്രപഞ്ചത്തിന്റെ വിളക്കും വെളിച്ചവുമാണ് അല്ലാഹുവെന്ന് രണ്ടാമത്തെ വചനവും വ്യക്തമാക്കുന്നു. സൂര്യനെ വിളക്കും ചന്ദ്രനെ പ്രകാശവുമായി പരിചയപ്പെടുത്തിയ വചനങ്ങളിലാകട്ടെ രണ്ടും ഒരേ വചനത്തില്‍തന്നെ പ്രതിപാദിക്കുകയും ഒന്ന് മറ്റേതിന് ഉപോല്‍ബലകമാണെന്ന് വ്യക്തമാക്കുകയും ചെയ്യുന്നുണ്ട്. സൂര്യനെ സിറാജും ചന്ദ്രനെ നൂറുമായി പരിചയപ്പെടുത്തിയതും അല്ലാഹുവിനെ നൂറും മിസ്വ്ബാഹുമായും മുഹമ്മദ് നബിയെ സിറാജന്‍ മുനീറയായും പരിചയപ്പെടുത്തിയതും തമ്മില്‍ താരതമ്യത്തിനുതന്നെ പറ്റാത്തത്ര വ്യത്യാസമുണ്ടെന്ന് സാരം.

സൂര്യനെയും ചന്ദ്രനെയും കുറിച്ച് നിരവധി പരാമർശങ്ങൾ ഖുര്ആനിലുണ്ട്. അവയിലെല്ലാം ഖുർആൻ സൂക്ഷ്മത പുലർത്തുന്നുണ്ട്. ബൈബിളിലെ ആകാശഗോളങ്ങളെക്കുറിച്ച പരാമർശങ്ങളിൽ അബദ്ധങ്ങളുണ്ടെന്ന് ചില ഗവേഷകന്മാർ അഭിപ്രായപ്പെട്ടിട്ടുണ്ട്. എങ്ങനെയായിരിക്കും ഖുർആനിലും എന്ന കരുതുന്ന ചില യുക്തിവാദികളാണ് ഈ വിമർശനം ഉന്നയിക്കാറുള്ളത്.

''ദൈവം മഹാദീപങ്ങള്‍ സൃഷ്ടിച്ചു; പകലിനെ നയിക്കാന്‍ വലുത്, രാത്രിയെ നയിക്കാന്‍ ചെറുത്'' (ഉല്‍പത്തി 1:6) എന്ന ബൈബിള്‍ വചനം സൂക്ഷ്മമായി അപഗ്രഥിച്ചാല്‍ ബൈബിള്‍ രചയിതാക്കള്‍ക്കിടയിലുണ്ടായിരുന്ന അബദ്ധധാരണകളുടെ സ്വാധീനമുള്‍ക്കൊള്ളുന്നതായി മനസ്സിലാക്കാനാവുമെന്ന് ബൈബിള്‍ ഗവേഷകരില്‍ ചിലര്‍ വ്യക്തമാക്കിയിട്ടുണ്ട്. ഇവിടെ 'മഹാദീപ'മെന്ന് പരിഭാഷപ്പെടുത്തിയിരിക്കുന്നത് 'ഗഡോള്‍ മ'ഓര്‍'(gadowl ma'owr) എന്ന ഹിബ്രു ശബ്ദത്തെയാണ്. വിളക്കിനാണ് മ'ഓര്‍ എന്ന് പറയുകയെന്ന് ബൈബിളിന്റെ ആധികാരിക ശബ്ദകോശമായ സ്‌ട്രോങ്ങ് ലക്‌സിക്കണ്‍ വ്യക്തമാക്കുന്നു. (Strongs Lexicon H -3974) പ്രകാശവുമായി ബന്ധപ്പെടുത്തി പറയുമ്പോള്‍ സൂര്യനെയും ചന്ദ്രനെയും കുറിക്കുവാന്‍ ഒരേപദം ഉപയോഗിച്ചിരിക്കുന്നത് സൂക്ഷ്മമായ അര്‍ഥത്തിലുള്ള ഒരു അബദ്ധമാണെന്നാണ് വാദം. സൂര്യചന്ദ്രന്മാര്‍ ആകാശത്ത് നിര്‍വഹിക്കുന്ന ദൗത്യം രണ്ടാണെന്നിരിക്കെ, രണ്ടിനെയും ദീപമായി ഉപമിച്ചിരിക്കുന്നത് ശരിയല്ലെന്നും പ്രകാശം പുറപ്പെടുവിക്കുന്ന സൂര്യന്‍ ദീപമാണെങ്കില്‍ അത് പ്രതിഫലിപ്പിക്കുന്ന ചന്ദ്രന്‍ പ്രകാശപ്രതിബിംബം മാത്രമാണെന്നും രണ്ടും പ്രകാശം പുറപ്പെടുവിക്കുന്ന ആകാശഗോളങ്ങളാണെന്ന അബദ്ധധാരണയില്‍ നിന്നാണ് ഈ ഉപമാപ്രയോഗമുണ്ടായിരിക്കുന്നത് എന്നതുകൊണ്ടു തന്നെ ഈ പ്രയോഗം സ്ഖലിതമാണെന്നുമുള്ള വിമര്‍ശനങ്ങളില്‍ കഴമ്പുണ്ടെന്ന് തന്നെയാണ് ബൈബിള്‍ പരാമര്‍ശങ്ങളെയും അതിന്റെ രചനാചരിത്രത്തെയും കുറിച്ച് പഠിച്ചാല്‍ നമുക്ക് മനസ്സിലാവുക.

ഇതിൽ നിന്ന് തികച്ചും വ്യത്യസ്തമാണ് ഖുർആനിലെ പരാമർശങ്ങൾ. സൂര്യചന്ദ്രന്‍മാരെക്കുറിച്ച ഖുര്‍ആന്‍ പരാമര്‍ശങ്ങള്‍ ശ്രദ്ധിക്കുക.

''ചന്ദ്രനെ അവിടെ ഒരു പ്രകാശമാക്കിയിരിക്കുന്നു. സൂര്യനെ ഒരു വിളക്കുമാക്കിയിരിക്കുന്നു.'' (71:16)

''സൂര്യനെ ഒരു പ്രകാശമാക്കിയത് അവനാകുന്നു. ചന്ദ്രനെ അവനൊരു ശോഭയാക്കുകയും, അതിന് ഘട്ടങ്ങള്‍ നിര്‍ണയിക്കുകയും ചെയ്തിരിക്കുന്നു. നിങ്ങള്‍ കൊല്ലങ്ങളുടെ എണ്ണവും കണക്കും അറിയുന്നതിന് വേണ്ടി. യഥാര്‍ഥ മുറപ്രകാരമല്ലാതെ അല്ലാഹു അതൊന്നും സൃഷ്ടിച്ചിട്ടില്ല. മനസ്സിലാക്കുന്ന ആളുകള്‍ക്കു വേണ്ടി അല്ലാഹു തെളിവുകള്‍ വിശദീകരിക്കുന്നു.'' (10:5)

''ആകാശത്ത് നക്ഷത്രമണ്ഡലങ്ങള്‍ ഉണ്ടാക്കിയവന്‍ അനുഗ്രഹപൂര്‍ണനാകുന്നു. അവിടെ അവന്‍ ഒരു വിളക്കും (സൂര്യന്‍) വെളിച്ചം നല്‍കുന്ന ചന്ദ്രനും ഉണ്ടാക്കിയിരിക്കുന്നു.'' (25:61)

ഈ വചനങ്ങളില്‍ സൂര്യനെ വിളിച്ചിരിക്കുന്നത് സിറാജ്, ദ്വിയാഅ് എന്നിങ്ങനെയാണ്. സിറാജ് എന്നാല്‍ 'വിളക്ക്' എന്നാണര്‍ഥം; ദ്വിയാഅ് എന്നാല്‍ ' തിളങ്ങുന്ന ശോഭ'യെന്നും. ചന്ദ്രനെ വിളിച്ചരിക്കുന്നതാകട്ടെ നൂര്‍ എന്നോ മുനീര്‍ എന്നോ ആണ്. നൂര്‍ എന്നാല്‍ 'പ്രകാശം' എന്നാണര്‍ഥം; മുനീര്‍ എന്നാല്‍ 'വെളിച്ചം നല്‍കുന്നത്' എന്നും. സിറാജ് പ്രകാശത്തിന്റെ സ്രോതസ്സാണ്. നൂര്‍ അത് നിര്‍മിക്കുന്ന പ്രകാശവും. സൂര്യനാണ് പ്രകാശത്തിന്റെ സ്രോതസ്സ് എന്നും ചന്ദ്രനില്‍ നിന്ന് ലഭിക്കുന്നത് സൂര്യനില്‍ നിര്‍മിക്കപ്പെടുന്ന പ്രകാശമാണെന്നും സ്വയം പ്രകാശിക്കാത്ത ചന്ദ്രനില്‍ സൂര്യപ്രകാശം പ്രതിചലിക്കുന്നതുകൊണ്ടാണ് അതില്‍നിന്ന് നമുക്ക് വെളിച്ചം ലഭിക്കുന്നത് എന്നും ഇന്നു നമുക്കറിയാം. ഖുര്‍ആന്‍ അവതരിപ്പിക്കപ്പെടുന്ന കാലത്ത് മനുഷ്യര്‍ക്ക് ഇല്ലാതിരുന്ന അറിവാണിത്. എത്ര കൃത്യമാണ് ഖുര്‍ആനിക പരാമര്‍ശങ്ങള്‍!

'സിറാജ്' എന്ന അറബി പദത്തിന്റെ നേര്‍ക്കുനേരെയുള്ള അര്‍ഥം 'വിളക്ക്' എന്നാണ്. രാത്രിയിലാണ് മനുഷ്യര്‍ക്ക് വിളക്ക് ആവശ്യമായി വരാറുള്ളത്. നല്ല നിലാവുള്ള രാത്രിയില്‍ ചന്ദ്രന്‍ നമുക്ക് വിളക്കിന് പകരമാവാറുണ്ട്. അതുകൊണ്ടുതന്നെ സാധാരണഗതിയില്‍ ചന്ദ്രനെയാണ് വിളക്കിനോട് ഉപമിക്കുവാന്‍ ഏറ്റവും അനുയോജ്യം. മനുഷ്യരുടെ ഉപമാലങ്കാരങ്ങളില്‍ അങ്ങനെയാണ് കാണപ്പെടുക. ഖുര്‍ആന്‍ ഇവിടെ കൃത്യത പുലര്‍ത്തുന്നു. സൂര്യനാണ് യഥാര്‍ഥത്തില്‍ വിളക്ക്; പ്രകാശത്തിന്റെ സ്രോതസ്സ്. ചന്ദ്രനില്‍ നാം കാണുന്നത് പ്രതിഫലിക്കപ്പെട്ട പ്രകാശം മാത്രമാണ്. ഖുര്‍ആന്‍ സൂര്യനെ സിറാജായും ചന്ദ്രനെ നൂറായും പരിചയപ്പെടുത്തുന്നു. പതിനാലു നൂറ്റാണ്ടു മുമ്പത്തെ അറിവിന്റെ അടിസ്ഥാനത്തില്‍ എഴുതപ്പെട്ടതായിരുന്നുവെങ്കില്‍ ഇത്ര കൃത്യമായ പരാമര്‍ശങ്ങള്‍ കാണുവാന്‍ നമുക്ക് കഴിയുകയില്ലായിരുന്നു. സര്‍വ്വേശ്വരനായ തമ്പുരാന്റെ വചനങ്ങളാണ് ഖുര്‍ആന്‍ എന്ന വസ്തുത വ്യക്തമാക്കുന്നതാണ് ഈ കൃത്യത.

രിക്കലും തെറ്റുപറ്റാത്തതെന്ന് വിശ്വസിക്കപ്പെടുന്ന ഖുര്‍ആന്‍ വചനങ്ങളെ തെറ്റാന്‍ സാധ്യതയുള്ള ശാസ്ത്ര സിദ്ധാന്തങ്ങളുടെ വെളിച്ചത്തില്‍ വ്യാഖ്യാനിക്കുന്നത് അപടകരവും ബാലിശവുമാണെന്ന് കരുതുന്നവരുണ്ട്. ഖുര്‍ആന്‍ ശാസ്ത്രപഠനങ്ങള്‍ പരിധിവിടുമ്പോള്‍ അവ അപകടകരമായിത്തീരാറുണ്ടെന്നത് ശരിയാണ്. എന്നാല്‍ ഖുര്‍ആനിന്റെ ദൈവികത വ്യക്തമാക്കുന്ന പഠനങ്ങളെ മുഴുവന്‍ നിഷേധിക്കുവാന്‍ അത് കാരണമായിക്കൂടാ. ഖുര്‍ആന്‍-ശാസ്ത്ര പഠനങ്ങളുടെ യഥാര്‍ഥധര്‍മം മനസ്സിലാവാത്തതുകൊണ്ടാണ് ഇത്തരം വിമര്‍ശനങ്ങളുണ്ടാവുന്നത്.

ശാശ്വത മൂല്യങ്ങളാണ് ഖുര്‍ആന്‍ പഠിപ്പിക്കുന്നത്. അതിലെ പ്രധാനപ്പെട്ട പ്രതിപാദ്യങ്ങളെല്ലാം ശാസ്ത്രീയമായ അപഗ്രഥനത്തിന് പുറത്തുള്ളവയാണ്. സ്രഷ്ടാവും സംരക്ഷകനുമായ തമ്പുരാനിലുള്ള വിശ്വാസവും അവനെ മാത്രം ആരാധിക്കേണ്ടതിന്റെ ആവശ്യകതയുമാണ് ഖുര്‍ആനിക പരാമര്‍ശങ്ങളുടെ കേന്ദ്രബിന്ദു. പ്രപഞ്ചത്തെ സൃഷ്ടിച്ച് പരിപാലിക്കുന്ന, പദാര്‍ഥം ലോകത്തിന് അതീതനായ അല്ലാഹുവിന്റെ അസ്തിത്വമോ അവന്റെ ആരാധ്യതയോ ശാസ്ത്രീയമായ അപഗ്രഥനത്തിന് കഴിയുന്നതല്ല. മരണാനന്തരജീവിതവും അതിലെ രക്ഷാ ശിക്ഷകളുമാണ് ഖുര്‍ആനില്‍ പ്രതിപാദിക്കപ്പെടുന്ന രണ്ടാമത്തെ പ്രധാനപ്പെട്ട കാര്യം. ഇവയും ശാസ്ത്രീയമായ നിരീക്ഷണങ്ങള്‍ക്ക് അതീതമായ വസ്തുതകളാണ്.

നന്‍മതിന്‍മകളെക്കുറിച്ച ഉല്‍ബോധനമാണ് പിന്നീട് ഖുര്‍ആനിലുള്ളത്. ധര്‍മാധര്‍മങ്ങളെ വ്യവഛേദിക്കുവാന്‍ ശാസ്ത്രത്തിന്റെ പക്കല്‍ മാനദണ്ഡങ്ങളൊന്നുമില്ല. ഖുര്‍ആനിന്റെ പ്രധാനപ്പെട്ട പ്രമേയങ്ങളൊന്നും തന്നെ ശാസ്ത്രീയമായ അപഗ്രഥനത്തിന് പറ്റുന്നതല്ല. അതുകൊണ്ടുതന്നെ 'ഖുര്‍ആനിനെ ശാസ്ത്രീയമായി വ്യാഖ്യാനിക്കുക'യെന്ന് പറയുന്നത് തന്നെ ശുദ്ധഭോഷ്‌ക്കാണ്. ശാസ്ത്രത്തിന്റെ അപഗ്രഥന വിശദീകരണ പരിധിയില്‍ വരാത്ത കാര്യങ്ങളെ എങ്ങനെയാണ് ശാസ്ത്രീയമായി വ്യാഖ്യാനിക്കുക?

പ്രകൃതി പ്രതിഭാസങ്ങളെക്കുറിച്ച ഖുര്‍ആനിക പരാമര്‍ശങ്ങള്‍ തെറ്റു പറ്റാത്തവയാണെന്നതിന് ശാസ്ത്രീയ നിരീക്ഷണങ്ങള്‍ നല്‍കുന്ന തെളിവുകളെക്കുറിച്ച പഠനം ഖുര്‍ആനിനെ ശാസ്ത്രീയമായി വ്യാഖ്യാനിക്കലല്ല; അങ്ങനെ ആയിക്കൂടാ. ഖുര്‍ആനിന്റെ വെളിച്ചത്തില്‍ ശാസ്ത്രഗവേഷണങ്ങളെ പരിശോധനാവിധേയമാക്കലാണ് അത്. ഖുര്‍ആനിക പരാമര്‍ശങ്ങള്‍ എത്രത്തോളം കൃത്യവും തെറ്റുപറ്റാത്തവയുമാണെന്ന് മനസ്സിലാക്കുവാന്‍ ശാസ്ത്രീയ ഗവേഷണങ്ങളെ ഉപയോഗപ്പെടുത്തുക മാത്രമാണ് ഇവിടെ ചെയ്യുന്നത്. അതല്ലാതെ, നിലവിലുള്ള ശാസ്ത്രജ്ഞാനത്തിന് അനുസൃതമായി ഖുര്‍ആന്‍ വചനങ്ങളെയോ പരാമര്‍ശങ്ങളെയോ വ്യാഖ്യാനിച്ച് വികലമാക്കലല്ല ഖുര്‍ആന്‍-ശാസ്ത്ര പഠനങ്ങള്‍ ചെയ്യേണ്ടത്. അങ്ങനെ വ്യാഖ്യാനിക്കുന്ന പഠനങ്ങള്‍ യാതൊരു ന്യായീകരണവുമര്‍ഹിക്കുന്നില്ല. തെറ്റു പറ്റാത്ത അല്ലാഹുവിന്റെ വചനങ്ങളെ വ്യാഖ്യാനിക്കുവാന്‍ അബദ്ധങ്ങള്‍ സംഭവിച്ചുകൊണ്ടിരിക്കുന്ന ശാസ്ത്രത്തെ ഉപയോഗപ്പെടുത്തുന്നത് നീതീകരിക്കുവാന്‍ കഴിയാത്ത കാര്യമാണ്.

ശാസ്ത്രത്തിന്റെ തെറ്റുപറ്റാനുള്ള സാധ്യതയെക്കുറിച്ചു പറയുമ്പോള്‍ ശാസ്ത്രീയ നിഗമനങ്ങള്‍, സിദ്ധാന്തങ്ങള്‍, വസ്തുതകള്‍ എന്നിവ തമ്മിലുള്ള വ്യത്യാസം നാം കൃത്യമായി മനസ്സിലാക്കേണ്ടതാണ്. തനിക്ക് ലഭിച്ച വിവരങ്ങളുടെ വെളിച്ചത്തില്‍ ശാസ്ത്രജ്ഞന്‍ ആദ്യമായി ഒരു 'നിഗമന'ത്തില്‍ (hypothesis) എത്തിച്ചേരുന്നു. പ്രസ്തുത നിഗമനത്തിന് ഉപോല്‍ബലകമായ തെളിവുകള്‍ ശേഖരിക്കുകയും പ്രസ്തുത തെളിവുകളുടെ വെളിച്ചത്തില്‍ ഒരു സിദ്ധാന്തത്തിന് (theory) അയാള്‍ രൂപം നല്‍കുകയും ചെയ്യുന്നു. പ്രസ്തുത സിദ്ധാന്തം ശരിയാണെങ്കില്‍ കണ്ടുപിടിക്കപ്പെടേണ്ട കാര്യങ്ങളെക്കുറിച്ച പ്രവചനങ്ങള്‍ ശരിയാണെന്ന് സ്ഥിരീകരിക്കപ്പെടുന്നതോടെ അത് ശാസ്ത്രലോകം അംഗീകരിക്കുന്ന സിദ്ധാന്തങ്ങളുടെ ഗണത്തിലെത്തിച്ചേരുന്നു. സ്വീകരിക്കപ്പെട്ട സിദ്ധാന്തവുമായി ബന്ധപ്പെട്ട സംശയങ്ങള്‍ ദൂരീകരിക്കപ്പെടുകയും ഉത്തരം കണ്ടെത്തേണ്ട പ്രശ്‌നങ്ങള്‍ക്ക് പരിഹാരമാവുകയും ചെയ്യുന്നതോടെ അത് ഒരു യാഥാര്‍ഥ്യമായി (fact) അംഗീകരിക്കപ്പെടുന്നു.

ശാസ്ത്രലോകത്ത് അംഗീകരിക്കപ്പെട്ട യാഥാര്‍ഥ്യങ്ങള്‍ തെറ്റുപറ്റാത്തവയാകാമെങ്കിലും അവ വീണ്ടും വികസിക്കുവാന്‍ സാധ്യതയുള്ളതാണ്. സൂര്യപ്രകാശത്തില്‍ അടങ്ങിയിരിക്കുന്നത് സപ്തവര്‍ണങ്ങളാണ് (vibgyor) എന്നത് ശാസ്ത്രീയമായി അംഗീകരിക്കപ്പെട്ട ഒരു യാഥാര്‍ഥ്യമാണ്. എന്നാല്‍ സപ്തവര്‍ണങ്ങളെ കൂടാതെ അള്‍ട്രാവയലറ്റ്, ഇന്‍ഫ്രാറെഡ് തുടങ്ങിയ കിരണങ്ങള്‍ കൂടി സൂര്യപ്രകാശത്തിലുണ്ട് എന്നത് പ്രസ്തുത യാഥാര്‍ഥ്യത്തിന്റെ വികാസമാണ്. ശാസ്ത്രീയമായി സ്ഥിരീകരിക്കപ്പെട്ട വസ്തുതകള്‍പോലും പൂര്‍ണമായിക്കൊള്ളണമെന്നില്ല എന്നര്‍ഥം.

വിശുദ്ധ ഖുര്‍ആനില്‍ അബദ്ധങ്ങളൊന്നുമില്ലെന്ന് ശാസ്ത്രീയമായ പഠനങ്ങള്‍ വ്യക്തമാക്കുന്നുവെന്ന് പറയുമ്പോള്‍ സ്ഥിരീകരിക്കപ്പെട്ട വസ്തുതകള്‍ ഖുര്‍ആനിന്റെ അപ്രമാദിത്വത്തെ അംഗീകരിക്കുന്നുവെന്ന് മാത്രമെ അര്‍ഥമാക്കുന്നുള്ളൂ. ശാസ്ത്രജ്ഞരുടെ നിഗമനങ്ങളും തെളിയിക്കപ്പെടാത്ത സിദ്ധാന്തങ്ങളും ചിലപ്പോള്‍ ഖുര്‍ആനിക പരാമര്‍ശങ്ങളോട് വൈരുധ്യം പുലര്‍ത്തുന്നുണ്ടാവാം. അവ ഖുര്‍ആനിന്റെ സാധുതയെ ഒരുവിധത്തിലും ബാധിക്കുന്നില്ല. പ്രസ്തുത സിദ്ധാന്തങ്ങള്‍ക്ക് അനുസൃതമായി ഖുര്‍ആനിനെ വ്യാഖ്യാനിക്കുവാന്‍ ശ്രമിക്കുന്നത് ദൈവികഗ്രന്ഥത്തോട് ചെയ്യുന്ന വലിയ പാതകമാണ്. ശാസ്ത്രലോകംതന്നെ അംഗീകരിച്ചു കഴിഞ്ഞിട്ടില്ലാത്ത സങ്കല്‍പ്പങ്ങള്‍ക്കു അനുസരിച്ച് സ്രഷ്ടാവിന്റെ വചനങ്ങളെ വ്യാഖ്യാനിച്ച് വികലമാക്കുന്നത് യാതൊരുവിധ ന്യായീകരണവുമര്‍ഹിക്കുന്നില്ല. തെളിയിക്കപ്പെട്ട വസ്തുകള്‍ക്ക് അനുസൃതമായി ഖുര്‍ആന്‍ വ്യാഖ്യാനിക്കേണ്ടതില്ല; വസ്തുതകള്‍ വ്യാഖ്യാനങ്ങളില്ലാതെത്തന്നെ ഖുര്‍ആന്‍ പരാമര്‍ശങ്ങളെ സത്യപ്പെടുത്തുന്നവയായിരിക്കും എന്നതാണ് യാഥാര്‍ഥ്യം. ഖുര്‍ആന്‍ ദൈവികമാണെന്നതിന് അംഗീകരിക്കപ്പെട്ട ശാസ്ത്രീയ വസ്തുതകള്‍ നല്‍കുന്ന തെളിവുകള്‍ വെളിപ്പെടുത്തുകയാണ്, ശാസ്ത്രത്തിനനുസരിച്ച് ഖുര്‍ആന്‍ വ്യാഖ്യാനിക്കുകയല്ല ഖുര്‍ആന്‍-ശാസ്ത്ര പഠനങ്ങളുടെ ലക്ഷ്യമെന്ന് ചുരുക്കം.

ഖുര്‍ആനില്‍ എല്ലാ ശാസ്ത്രീയ വസ്തുതകളും പരാമര്‍ശിക്കപ്പെട്ടിട്ടുണ്ടെന്നോ അതില്‍ പറഞ്ഞിട്ടുള്ള കാര്യങ്ങള്‍ മാത്രമാണ് ശാസ്ത്രീയ ഗവേഷണങ്ങള്‍ പുറത്തുകൊണ്ടുവരുന്നത് എന്നോ ഉള്ള അവകാശ വാദങ്ങളൊന്നും മുസ്‌ലിംകള്‍ക്കില്ല. ഖുര്‍ആന്‍ ശാസ്ത്രം പഠിപ്പിക്കുവാന്‍ വേണ്ടി അവതരിപ്പിക്കപ്പെട്ടതല്ല എന്നതുകൊണ്ടുതന്നെ അതില്‍ സകല ശാസ്ത്രവും ഉണ്ടെന്ന് ആര്‍ക്കും അവകാശപ്പെടാനാവില്ല; അങ്ങനെ ആരും അവകാശപ്പെടുന്നുമില്ല. പ്രകൃതി പ്രതിഭാസങ്ങളെക്കുറിച്ച് പരാമര്‍ശങ്ങളുള്‍ക്കൊള്ളുന്ന ഖുര്‍ആന്‍ വചനങ്ങളുടെ കൃത്യതയും അപ്രമാദിത്വവും ശാസ്ത്രീയ ഗവേഷണങ്ങള്‍ വ്യക്തമാക്കുകയാണ് ചെയ്യുന്നത് എന്നാണ് മുസ്‌ലിംകള്‍ അവകാശപ്പെടുന്നത്. ഇതൊരു കേവലമായ അവകാശവാദമല്ല. ആര്‍ക്കും പരിശോധിച്ച് സ്വയം തന്നെ ബോധ്യപ്പെടാന്‍ കഴിയുന്ന വസ്തുതയാണത്. തങ്ങളുടെ കൈവശമുള്ള പൗരാണികമോ ആധുനികമോ ആയ ഏത് മാനദണ്ഡമുപയോഗിച്ച് പരിശോധനാവിധേയമാക്കിയാലും ഖുര്‍ആന്‍ അബദ്ധങ്ങളില്‍ നിന്നു മുക്തമാണെന്ന് ആര്‍ക്കും മനസ്സിലാവും.

ശാസ്ത്രീയ വസ്തുതകള്‍ മുഴുവനുമോ ഗവേഷണങ്ങളിലൂടെ നിര്‍മ്മിച്ചെടുക്കുന്ന സാമഗ്രികളെക്കുറിച്ച വിവരങ്ങളോ ഖുര്‍ആനില്‍ മുമ്പേ പരാമര്‍ശിക്കപ്പെട്ടിട്ടുണ്ട് എന്നതല്ല നമ്മുടെ അവകാശവാദമെന്ന വസ്തുത ഖുര്‍ആനിന്റെ അനുകൂലികളും പ്രതികൂലികളും ഒരേപോലെ മനസ്സിലാക്കേണ്ടതുണ്ട്. ഖുര്‍ആനില്‍ അബദ്ധങ്ങളില്ലെന്ന യാഥാര്‍ഥ്യത്തിന് ശാസ്ത്രീയമായ ഗവേഷണങ്ങള്‍ തെളിവു നല്‍കുന്നുവെന്നാണ് മുസ്‌ലിംകളുടെ വാദം.

ഇത് വേണ്ട രൂപത്തില്‍ മനസ്സിലാക്കാത്തതിനാല്‍ ചിലപ്പോഴെല്ലാം ഖുര്‍ആന്‍ ശാസ്ത്ര പഠനങ്ങള്‍പരിധിവിട്ട അവകാശവാദങ്ങള്‍ ഉന്നയിക്കുന്ന അവസ്ഥയിലെത്തിച്ചേറാറുണ്ട്. ഖുര്‍ആനിലുള്ളതെല്ലാം ശാസ്ത്രമാണെന്നും ഖുര്‍ആനിന്റെ അംഗീകാരമില്ലാത്തതൊന്നും ശാസ്ത്രമല്ലെന്നുമുള്ള രീതിയിലുള്ള പരാമര്‍ശങ്ങളും ശാസ്ത്രത്തിന് വിശദീകരിക്കുവാന്‍ കഴിഞ്ഞിട്ടില്ലാത്ത മേഖലകളെക്കുറിച്ച ഖുര്‍ആന്‍ പരാമര്‍ശങ്ങളെ ശാസ്ത്രീയമായി വ്യാഖ്യാനിക്കുവാനുള്ള ത്വരയുമെല്ലാം പരിധിവിട്ടതും അംഗീകരിക്കുവാന്‍ കഴിയാത്തതുമാണ്. ഒരു കളങ്കവുമില്ലാത്ത വിശുദ്ധ നെയ്യാണ് ശാസ്ത്രമെന്ന ധാരണയുടെ വെളിച്ചത്തിലാണ് ഇത്തരം കസര്‍ത്തുകളെല്ലാം അരങ്ങേറാറുള്ളത്. ശാസ്ത്ര നിഗമനങ്ങളും സിദ്ധാന്തങ്ങളും വസ്തുതകളും തമ്മിലുള്ള ബന്ധത്തെക്കുറിച്ച ശാസ്ത്രദര്‍ശനത്തിന്റെ കാഴ്ചപ്പാട് എന്താണെന്നുപോലും ഇത്തരം വ്യാഖ്യാനവിശാരദന്‍മാര്‍ പരിഗണിക്കാറില്ല. ശാസ്ത്രത്തിന്റെ ലേബലില്‍ കാണപ്പെടുന്നതെല്ലാം സത്യമാണെന്ന ധാരണയുടെ വെളിച്ചത്തില്‍ നടക്കുന്ന ഇത്തരം ഖുര്‍ആന്‍-ശാസ്ത്ര പഠനങ്ങള്‍ക്ക് ഖുര്‍ആനിന്റെ അംഗീകാരമില്ല; അവയ്ക്ക് ശാസ്ത്രീയമായ അടിത്തറയുമുണ്ടാകാറില്ല. അങ്ങനെയുള്ള പഠനക്കസര്‍ത്തുകള്‍ മുന്നില്‍വെച്ച്, ഖുര്‍ആനില്‍ അബദ്ധങ്ങളൊന്നുമില്ലെന്ന വസ്തുതയ്ക്ക് ശാസ്ത്രം സാക്ഷ്യം വഹിക്കുന്നുവെന്ന വസ്തുത വ്യക്തമാക്കുന്ന ഗവേഷണങ്ങളെ പിന്തിരിപ്പിക്കാനായി അവതരിപ്പിക്കുന്നത് ന്യായീകരണമര്‍ഹിക്കുന്നില്ല.

വിഷയവുമായി ബന്ധപ്പെട്ട വീഡിയോ

ര്‍വ്വശക്തനായ സ്രഷ്ടാവില്‍നിന്ന് അവതരിപ്പിക്കപ്പെട്ടതാണെന്ന് സ്വയം പ്രഖ്യാപിക്കുന്ന ഒരേയൊരു വേദഗ്രന്ഥമാണ് ഖുര്‍ആന്‍.

''തീര്‍ച്ചയായും ഇത് (ഖുര്‍ആന്‍) ലോകരക്ഷിതാവ് അവതരിപ്പിച്ചത് തന്നെയാകുന്നു.'' (ഖുര്‍ആന്‍ :26:192)

''. ഈ ഗ്രന്ഥത്തിന്റെ അവതരണം സര്‍വ്വലോകരക്ഷിതാവിങ്കല്‍ നിന്നാകുന്നു. ഇതില്‍ യാതൊരു സംശയവുമില്ല.'' (ഖുര്‍ആന്‍ : 32: 2)

''പരമകാരുണികന്‍; ഈ ഖുര്‍ആന്‍ പഠിപ്പിച്ചു.'' (ഖുര്‍ആന്‍ : 55:1,2)

ദൈവികഗ്രന്ഥത്തില്‍ അബദ്ധങ്ങളൊന്നുമുണ്ടാകുവാന്‍ പാടില്ല; തെറ്റുപറ്റാത്തവനായ പടച്ചവനില്‍നിന്ന് അവതീര്‍ണമായതെന്ന് അവകാശപ്പെടുന്ന ഗ്രന്ഥത്തില്‍ തെറ്റുകളുണ്ടാകുവാന്‍ പാടില്ലെന്നത് സാമാന്യമായ ഒരു വസ്തുതയാണ്. എന്നാല്‍ മനുഷ്യരുടെ രചനകള്‍ അങ്ങനെയല്ല; എത്ര വലിയ ബുദ്ധിജീവിയുടെ രചനയാണെങ്കിലും അതില്‍ അബദ്ധങ്ങളുണ്ടാകാവുന്നതാണ്. അവ ചിലപ്പോള്‍ അയാളുടെ ജീവിതകാലത്ത്തന്നെ വെളിപ്പെടും. അതല്ലെങ്കില്‍ തലമുറകള്‍ കഴിഞ്ഞായിരിക്കും അത് ബോധ്യപ്പെടുക. മനുഷ്യരുടെ അറിവ് പരിമിതമായതിനാലും അത് വളര്‍ന്നുകൊണ്ടിരിക്കുന്നതിലുമാണ് ഇത്. ഇന്നലെയുള്ള അറിവിന്റെ അടിസ്ഥാനത്തില്‍ എഴുതിയതിലെ അബദ്ധങ്ങള്‍ ഇന്ന് കൂടുതല്‍ കൃത്യമായ അറിവ് ലഭിക്കുമ്പോള്‍ നാം തിരുത്തുന്നു. വിജ്ഞാനവര്‍ധനവിനനുസരിച്ച് സംഭവിക്കുന്ന സ്വാഭാവികമായ മാനവിക പ്രക്രിയയാണിത്. എല്ലാം അറിഞ്ഞു കഴിഞ്ഞുവെന്ന ഒരു അവസ്ഥ മാനവസമൂഹത്തിന് ഒരിക്കലും ഉണ്ടാകുവാന്‍ പോകുന്നില്ല എന്നിരിക്കെ ഈ തിരുത്തല്‍ പ്രക്രിയ മനുഷ്യാവസാനംവരെ തുടര്‍ന്നുകൊണ്ടിരിക്കും. ഒരു വിജ്ഞാനീയവും ഒരിക്കലും സ്വയം സമ്പൂര്‍ണമാകുന്നില്ല എന്നതുകൊണ്ടുതന്നെ അതിലുള്ള അറിവ് എപ്പോഴും വര്‍ധമാനമായിരിക്കുകയും പ്രസ്തുത വര്‍ധനവിനനുസരിച്ച് ഇന്നലെത്തെ വിവരങ്ങള്‍ തിരുത്തപ്പെട്ടുകൊണ്ടിരിക്കുകയും ചെയ്യും.

സ്രഷ്ടാവ് സര്‍വ്വജ്ഞനാണ്. അവന്റെ അറിവ് എന്നും സ്വയം സമ്പൂര്‍ണമാണ്. പ്രസ്തുത അറിവിലേക്ക് യാതൊന്നും കൂട്ടിച്ചേര്‍ക്കപ്പെടുകയോ അതില്‍നിന്ന് എന്തെങ്കിലും കുറയുകയോ ചെയ്യുന്നില്ല. അതുകൊണ്ടുതന്നെ അവന്റെ വചനങ്ങളില്‍ അബദ്ധങ്ങളുണ്ടായിക്കൂടാ. ദൈവികമെന്നവകാശപ്പെടുന്ന ഏതെങ്കിലുമൊരു ഗ്രന്ഥത്തില്‍ അബദ്ധങ്ങളുണ്ടെങ്കില്‍ പ്രസ്തുത അവകാശവാദം തെറ്റാണെന്നതിന് അതുതന്നെ മതിയായ തെളിവാണ്.

പ്രപഞ്ചത്തെയും പ്രകൃതിയെയും കുറിച്ച വസ്തുനിഷ്ഠമായ പഠനമാണ് ശാസ്ത്രം. ശാസ്ത്രീയമായ വിജ്ഞാനീയങ്ങളെല്ലാം വര്‍ധമാനമായ അറിവിന്റെ അടിസ്ഥാനത്തില്‍ ചിട്ടപ്പെടുത്തപ്പെട്ടവയാണ്. നമ്മുടെ അന്വേഷണം പുരോഗമിക്കുന്നതിനനുസരിച്ച് ഓരോ വിജ്ഞാനത്തിലുമുള്ള പൂര്‍വ്വകാല ധാരണകളില്‍ പലതും ചോദ്യം ചെയ്യപ്പെടുകയും തിരുത്തപ്പെടുകയും ചെയ്തിട്ടുണ്ട്. ഇന്നലെ ശരിയായിരുന്നുവെന്ന് കരുതിയവ തെറ്റാണെന്ന് മനസ്സിലാവുകയും പുതിയതും കൃത്യവുമായ ശരികളിലെത്തിച്ചേരുകയും ചെയ്യുകയെന്നത് ശാസ്ത്രലോകത്തെ സ്വാഭാവികമായ പ്രതിഭാസമാണ്. വിജ്ഞാനീയങ്ങളെക്കുറിച്ച പ്രതിപാദനങ്ങളുള്‍ക്കൊള്ളുന്ന ഒരു ഗ്രന്ഥത്തില്‍ തലമുറകള്‍ കഴിഞ്ഞിട്ടും തിരുത്തലുകളൊന്നും വേണ്ടിവരുന്നില്ലയെന്ന വര്‍ത്തമാനം ശാസ്ത്രലോകത്തിന് തീരെ ഉള്‍ക്കൊള്ളാനാവാത്തതാണ്. പ്രകൃതിയെയും പ്രാപഞ്ചിക പ്രതിഭാസങ്ങളെയും കുറിച്ച പരാമര്‍ശങ്ങളുണ്ടെങ്കില്‍ അതില്‍ തിരുത്തലുകള്‍ ആവശ്യമായിവരും എന്നതാണ് ശാസ്ത്രലോകത്തിന്റെ പൊതുവായ കാഴ്ചപ്പാട്.

ഖുര്‍ആന്‍ ദൈവികമാണെന്ന വസ്തുത ബോധ്യപ്പെടുന്ന പ്രധാനപ്പെട്ട മേഖലകളിലൊന്നാണിത്. പ്രപഞ്ചത്തെയും പ്രകൃതിയെയും കുറിച്ച നിരവധി പരാമര്‍ശങ്ങള്‍ പരിശുദ്ധ ഖുര്‍ആനിലുണ്ട്. മനുഷ്യരുടെ ദൈനംദിന ജീവിതവുമായി ബന്ധപ്പെടുന്ന വിജ്ഞാനീയങ്ങളിലുള്ള പരാമര്‍ശങ്ങളാല്‍ നിബിഢമാണ് ഖുര്‍ആന്‍ വചനങ്ങള്‍. പതിനാല് നൂറ്റാണ്ടുകള്‍ക്ക് മുമ്പ് നിരക്ഷരനായ ഒരു മനുഷ്യന്റെ (സ) നാവിലൂടെയാണ് ലോകം ഈ വചനങ്ങള്‍ ശ്രവിച്ചത്. ഇന്നു നാം ഉപയോഗിക്കുന്ന ശാസ്ത്രീയമായ മാനദണ്ഡങ്ങള്‍ വെച്ചു നോക്കുമ്പോള്‍ വിജ്ഞാനീയങ്ങള്‍ അവയുടെ ഭ്രൂണദശപോലും പ്രാപിച്ചിട്ടില്ലാത്ത സമയത്താണ് ഖുര്‍ആന്‍ വചനങ്ങള്‍ അവതരിപ്പിക്കപ്പെട്ടത്. മറ്റു ഗ്രന്ഥങ്ങളെ പരിശോധിക്കുവാന്‍ ഉപയോഗിക്കുന്ന മാനദണ്ഡങ്ങള്‍ വെച്ചു നോക്കുമ്പോള്‍ ഖുര്‍ആന്‍ അബദ്ധങ്ങളാല്‍ നിബിഢമാകേണ്ടതാണ്. എന്നാല്‍, അത്ഭുതം! ഖുര്‍ആനില്‍ അബദ്ധങ്ങളൊന്നും തന്നെ കണ്ടെത്തുവാന്‍ കഴിയുന്നില്ല. ഖുര്‍ആന്‍ പരാമര്‍ശിച്ച കാര്യങ്ങളുമായി ബന്ധപ്പെട്ട വിജ്ഞാനീയങ്ങളുടെ വളര്‍ച്ച അതില്‍ അബദ്ധങ്ങളുണ്ടെന്ന് സ്ഥാപിക്കുകയല്ല, പ്രത്യുത അതില്‍ സുബദ്ധങ്ങളേയുള്ളുവെന്ന് സ്ഥിരീകരിക്കുകയാണ് ചെയ്തിട്ടുള്ളത്.

പതിനാലു നൂറ്റാണ്ടുകള്‍ക്കു മുമ്പ് ലോകം ശ്രവിച്ച ഒരു ഗ്രന്ഥത്തില്‍, പ്രപഞ്ചത്തെയും അതിന്റെ നിലനില്‍പിനെയും, ഭൂമിയെയും അതിലെ ജീവജാലങ്ങളെയും, മനുഷ്യനെയും അവനെ നിലനിര്‍ത്തുന്ന പ്രകൃതി പ്രതിഭാസങ്ങളെയും, സൂര്യനെയും ചന്ദ്രനെയും സമുദ്രത്തെയും കാറ്റിനെയും മഴയെയും മനുഷ്യന്റെ ഭ്രൂണ പരിണാമത്തെയുമെല്ലാമുള്ള പരാമര്‍ശങ്ങളുണ്ടായിട്ട് അതിലൊന്നും യാതൊരു അബദ്ധങ്ങളുമില്ലെന്നത് അത്ഭുതകരം തന്നെയാണ്. വര്‍ധമാനമായ അറിവിന്റെ ഉടമയായ മനുഷ്യനില്‍ നിന്നുള്ളതല്ല ഈ ഗ്രന്ഥമെന്ന വസ്തുത വ്യക്തമാക്കുന്നതാണ് ഇത്. പൂര്‍ണമായ അറിവിന്റെ നാഥന് മാത്രമെ ഒരിക്കലും തെറ്റു പറ്റാത്ത ഒരു ഗ്രന്ഥം അവതരിപ്പിക്കാനാവൂ. പുതിയ പുതിയ ശാസ്ത്രീയ ഗവേഷണങ്ങള്‍ ഖുര്‍ആന്‍ വചനങ്ങളുടെ കൃത്യതയും അപ്രമാദിത്വവും വ്യക്തമാക്കുമ്പോള്‍ അത് ദൈവികമാണെന്ന വസ്തുത കൂടുതല്‍ തെളിഞ്ഞു വരികയാണ് ചെയ്യുന്നത്. ശാസ്ത്രീയമായ ഗവേഷണങ്ങള്‍ ഖുര്‍ആനിന്റെ ദൈവികതയ്ക്ക് തെളിവുകള്‍ നല്‍കിക്കൊണ്ടിരിക്കുകയാണ് ചെയ്യുന്നതെന്ന് പറയുന്നത് ഇതുകൊണ്ടാണ്.

ശാസ്ത്രം എത്ര തന്നെ വളർന്നാലും ഖുർആനിൽ തെറ്റുകളൊന്നും കണ്ടെത്താനാവില്ലെന്ന് ബോധ്യപ്പെടുത്തുന്നതിനു വേണ്ടിയുള്ളതാണ് ക്വുർആൻ-ശാസ്ത്രപഠനങ്ങൾ. പ്രകൃതിപ്രതിഭാസങ്ങളെക്കുറിച്ച ക്വുർആനിലെ പരാമർശങ്ങൾ കൃത്യവും അബദ്ധമുക്തവുമാണെന്ന വസ്തുത ശാസ്ത്രീയമായ പഠനങ്ങളുടെ വെളിച്ചത്തിൽ വ്യക്തമാക്കുന്നതിനു വേണ്ടിയുള്ളതാണത്. ക്വർആനിനെ ശാസ്ത്രീയമായി വ്യാഖ്യാനിക്കുകയോ ശാസ്തത്തെ ക്വുർആനിനനുസരിച്ച് വളച്ചോടിക്കുകയോ ചെയ്യുന്നതിന് വേണ്ടിയുള്ളതല്ല അത്.

വിഷയവുമായി ബന്ധപ്പെട്ട വീഡിയോ

ല്ല. കഞ്ഞിന്റെ സൃഷ്ടിയിൽ പുരുഷസ്രവത്തിനും സ്ത്രീസ്രവത്തിനും പങ്കുണ്ടെന്നും അവ കൂട്ടിച്ചെർന്നാണ് കുഞ്ഞുണ്ടാവുന്നത് എന്നും തന്നെയാണ് ഖുർആനും ഹദീഥുകളും വ്യക്തമാക്കുന്നത്. മനുഷ്യനെ ജലത്തില്‍നിന്നാണ് സൃഷ്ടിച്ചിരിക്കുന്നതെന്ന് പ്രസ്താവിക്കുന്ന ക്വുര്‍ആന്‍ വചനങ്ങള്‍ സൂചിപ്പിക്കുന്നത് പുരുഷസ്രവത്തില്‍നിന്നുള്ള മനുഷ്യ സൃഷ്ടിയാണെന്നാണ് പ്രമുഖരായ ക്വുര്‍ആന്‍ വ്യാഖ്യാതക്കളെല്ലാം അഭിപ്രായപ്പെട്ടിരിക്കുന്നതെന്നത് ശരിയാണ് . ജലത്തില്‍ നിന്ന് മനുഷ്യനെ സൃഷ്ടിച്ചതായി പരാമര്‍ശിക്കുന്ന സുറത്തുല്‍ ഫുര്‍ക്വാനിലെ 25ാം വചനത്തിന് വ്യാഖ്യാനമായി നിസ്സാരമായ ജലത്തില്‍നിന്നാണ് മനുഷ്യ സൃഷ്ടി നടന്നതെന്ന സൂറത്തുല്‍ മുര്‍സലാത്തിലെ 20ാം വചനവും 'നിസാരമായ ഒരു ജലത്തിന്റെ സത്തില്‍' നിന്നാണ് അത് നടന്നതെന്ന സൂറത്തുസ്സജദയിലെ എട്ടാം വചനവും നിലകൊള്ളുന്നുണ്ട്. ഈ വചനങ്ങള്‍ താരതമ്യം ചെയ്ത് പരിശോധിച്ചാല്‍ മനുഷ്യനെ സൃഷ്ടിച്ച ജലമായി ക്വുര്‍ആന്‍ പരിചയപ്പെടുത്തുന്നത് പുരുഷസ്രവമാണെന്നു തന്നെയാണ് മനസ്സിലാവുക.

സ്ത്രീയുടെ സ്രവത്തെക്കുറിച്ച് ക്വുര്‍ആനില്‍ നേര്‍ക്കുനേരെയുള്ള പരാമര്‍ശങ്ങളൊന്നുമില്ലെങ്കിലും സ്വുല്‍ബിന്റെയും തറാഇബിന്റെയും ഇടയില്‍നിന്ന് പുറപ്പെടുന്ന തെറിച്ചുവീഴുന്ന ദ്രാവകത്തില്‍നിന്നാണ് മനുഷ്യനെ സൃഷ്ടിച്ചതെന്ന് പറയുന്ന സൂറത്തുത്ത്വാരിഖിലെ ആറും ഏഴും വചനങ്ങളെ വ്യാഖ്യാനിച്ച പ്രവാചകാനുചരന്‍മാരില്‍ ക്വുര്‍ആന്‍ വ്യാഖ്യാനത്തിന് പ്രസിദ്ധനായ ഇബ്‌നു അബ്ബാസും(റ) മറ്റൊരു സ്വഹാബിയായ ഇക്‌രിമ(റ)യും പുരുഷന്റെ സ്വുല്‍ബില്‍നിന്ന് പുറപ്പെടുന്ന ദ്രാവകവും സ്ത്രീയുടെ തറാഇബില്‍നിന്ന് പുറപ്പെടുന്ന ദ്രാവകവും ഒരുമിച്ചു ചേര്‍ന്നാണ് കുഞ്ഞുണ്ടാകുന്നതെന്ന് വ്യാഖ്യാനിച്ചതായി ഇമാം ത്വബരി രേഖപ്പെടുത്തുന്നുണ്ട്.(തഫ്‌സീര്‍ അത്ത്വബ്‌രി) പ്രസിദ്ധ ക്വുര്‍ആന്‍ വ്യാഖ്യാതക്കളായ ത്വബ്‌രി, സമഖ്ശരി, ത്വബ്‌റാനി, റാസി, ക്വുര്‍തുബി, ഇബ്‌നുകഥീര്‍, ജലാലൈനി, ശൗക്വാനി തുടങ്ങിയവരെല്ലാം പുരുഷന്റെ സ്വുല്‍ബില്‍നിന്നും സ്ത്രീയുടെ തറാഇബില്‍നിന്നും പുറപ്പെടുന്ന ദ്രാവകങ്ങളുടെ മിശ്രണത്തില്‍നിന്നാണ്് കുഞ്ഞുണ്ടാവുന്നതെന്നാണ് ഈ ആയത്ത് അര്‍ത്ഥമാക്കുന്നതെന്നാണ് അഭിപ്രായപ്പെട്ടിരിക്കുന്നത്. സ്ത്രീസ്രവവും പുരുഷസ്രവവും കൂടിച്ചേര്‍ന്നാണ് കുഞ്ഞുണ്ടാകുന്നതെന്ന് പ്രവാചകാനുചരന്‍മാര്‍ പരിശുദ്ധ ക്വുര്‍ആനില്‍ നിന്നു മനസ്സിലാക്കിയിരുന്നുവെന്ന് ഇത് വ്യക്തമാക്കുന്നു.

ഹദീഥുകള്‍ ഇവ്വിഷയകമായ കൂടുതല്‍ വിശദീകരണങ്ങള്‍ നല്‍കുന്നുണ്ട്. 'സ്ത്രീകള്‍ക്ക് സ്രവമുണ്ടാകുമോ?'യെന്ന ഉമ്മുസുലൈമി (റ)ന്റെ ചോദ്യത്തിന് പ്രവാചകന്‍(സ) നല്‍കിയ മറുപടിയില്‍നിന്ന് അക്കാലത്തെ പൊതുവിശ്വാസവും അതിലെ കൃത്യമായ പ്രവാചകതിരുത്തലും നമുക്ക് ലഭിക്കുന്നു. സ്വഹീഹുല്‍ ബുഖാരിയില്‍ ഉമ്മുസലമ(റ)യില്‍നിന്ന് നിവേദനം ചെയ്യപ്പെട്ട ഈ ഹദീഥില്‍നിന്ന് സ്ത്രീയുടെ സ്രവത്തെക്കുറിച്ച് അക്കാലത്തെ സ്ത്രീകള്‍ക്കുതന്നെ അറിയില്ലായിരുന്നുവെന്ന് മനസ്സിലാക്കാം.

അത്ഭുതത്തോടുകൂടിയാണ് ഉമ്മുസുലൈം 'സ്ത്രീകള്‍ക്ക് സ്രവമുണ്ടാകുമോ?'യെന്ന് ചോദിക്കുന്നത്. സംശയം ചോദിക്കുകയെന്നതിലുപരി അങ്ങനെ ഉണ്ടാവില്ലല്ലോയെന്ന് ദ്യോതിപ്പിച്ചുകൊണ്ടുള്ള പ്രസ്തുത ചോദ്യത്തിന് 'അതെ! ഇതെന്തൊരു ചോദ്യം? പിന്നെയെങ്ങനെയാണ് കുട്ടിക്ക് അവളോട് സാദൃശ്യമുണ്ടാവുക?' എന്ന മറുചോദ്യമാണ് പ്രവാചകന്‍ (സ) മറുപടിയായി നല്‍കുന്നത്. സ്ത്രീകള്‍ക്ക് സ്രവമുണ്ടെന്ന് വ്യക്തമാക്കുക മാത്രമല്ല, അത് കുട്ടിയുടെ പാരമ്പര്യദാതാവുകൂടിയാണെന്ന് പഠിപ്പിക്കുകകൂടി ചെയ്യുന്നുണ്ട് ഈ പ്രവാചകവചനം. ഉമ്മുസുലൈമും(റ) പ്രവാചകനും(സ) തമ്മില്‍ നടന്ന ഈ സംഭാഷണം കൂറേക്കൂടി വിശദമായി ഇമാം മുസ്്‌ലിം(റ) അനസുബ്‌നു മാലിക്കില്‍ (റ) നിന്ന് നിവേദനം ചെയ്തിട്ടുണ്ട്. 'പുരുഷന്റെ സ്രവം വെളുത്തതും കട്ടിയുള്ളതുമാണ്; സ്ത്രീയുടെ സ്രവം മഞ്ഞ നിറത്തിലുള്ളതും നേര്‍മയുള്ളതുമാണ്. ഏത് സ്രവമാണോ മുന്‍കടക്കുന്നത് അതിനോടാണ് കുഞ്ഞിന് സാദൃശ്യമുണ്ടാവുക' എന്നുകൂടി ഉമ്മുസുലൈമിനോട്(റ) പ്രവാചകന്‍(സ) പറഞ്ഞതായി ഈ നിവേദനത്തിലുണ്ട്. വെളുത്ത, കട്ടിയായ പുരുഷസ്രവത്തോട് മഞ്ഞ, നേര്‍മയായ സ്ത്രീസ്രവം കൂടിച്ചേര്‍ന്നാണ് കുഞ്ഞുണ്ടാകുന്നതെന്നാണ് ഇവിടെ പ്രവാചകന്‍(സ) പഠിപ്പിക്കുന്നത്.

ഒരു ജൂത പണ്ഡിതന്റെ ചോദ്യങ്ങള്‍ക്കുള്ള പ്രവാചകന്റെ(സ) ഉത്തരത്തെപ്പറ്റി വിശദീകരിക്കുന്ന ഥൗബാന്‍(റ) നിവേദനം ചെയ്ത സ്വഹീഹ് മുസ്ലിമിലുള്ള ദീര്‍ഘമായ ഹദീഥിലും ശിശുവിന്റെ സൃഷ്ടിയെക്കുറിച്ച ചോദ്യത്തിനുള്ള വിശദമായ ഉത്തരം ആരംഭിക്കുന്നത് 'പുരുഷസ്രവം വെളുത്തനിറത്തിലുള്ളതും സ്ത്രീസ്രവം മഞ്ഞനിറത്തിലുള്ളതുമാണ്; അവ രണ്ടും കൂട്ടിച്ചെരുമ്പോൾ....' എന്നു പറഞ്ഞുകൊണ്ടാണ്. ജൂത ചോദ്യങ്ങള്‍ക്കെല്ലാം മറുപടി പറഞ്ഞശേഷം 'അയാള്‍ എന്നോട് ചോദിച്ച കാര്യങ്ങളെക്കുറിച്ചൊന്നും അല്ലാഹു അറിയിച്ചുതരുന്നതുവരെ എനിക്ക് യാതൊരു വിവരവുമുണ്ടായിരുന്നില്ല' എന്ന് പറഞ്ഞതായുള്ള ഥൗബാനി (റ)ന്റെ പരാമര്‍ശം ശ്രദ്ധേയമാണ്. സ്വന്തം സ്രവത്തെക്കുറിച്ച് അറിയാത്ത സ്ത്രീകള്‍ക്കടക്കം നിങ്ങളുടെ സ്രവം മഞ്ഞനിറത്തിലുള്ളതാണ് എന്ന് പ്രവാചകന്‍(സ) പറഞ്ഞുകൊടുത്തത് വ്യക്തമായ ദൈവബോധനത്തിന്റെ അടിസ്ഥാനത്തിലാണെന്ന് വ്യക്തമാക്കുന്നതാണീ പ്രവാചകപരാമര്‍ശം.

ഏതാണീ മഞ്ഞ ദ്രാവകം? കുഞ്ഞിന്റെ സൃഷ്ടിയില്‍ പങ്കെടുക്കുന്ന പുരുഷസ്രവത്തിന്റെ നിറം 'അബ്‌യദ്വ്' ആണെന്നു പറഞ്ഞതിനുശേഷമാണ് സ്ത്രീ സ്രവത്തിന്റെ നിറം 'അസ്വ്ഫര്‍' (മഞ്ഞ) ആണെന്ന് പ്രവാചകന്‍ (സ) പറഞ്ഞത്. രണ്ടും കൂടിച്ചേര്‍ന്നാണ് കുഞ്ഞുണ്ടാകുന്നതെന്നും അതിനുശേഷം അദ്ദേഹം വ്യക്തമാക്കി. വെള്ള നിറത്തിലുള്ള പുരുഷസ്രവത്തെപോലെതന്നെ ബീജ സങ്കലനത്തില്‍ പങ്കെടുക്കുന്ന സ്ത്രീസ്രവത്തിന്റെ നിറം മഞ്ഞയാണെന്നാണ് പ്രവാചകന്‍ (സ) ഇവിടെ പഠിപ്പിക്കുന്നതെന്നുറപ്പാണ്. സ്ത്രീശരീരത്തില്‍നിന്ന് നിര്‍ഗളിക്കുന്ന ഏതു സ്രവത്തിനാണ് മഞ്ഞനിറമുള്ളതെന്ന കാര്യത്തില്‍ കര്‍മശാസ്ത്ര പണ്ഡിതന്‍മാര്‍ ഏറെ ചര്‍ച്ച ചെയ്തതായി കാണാന്‍ കഴിയും. സ്ത്രീജനനേന്ദ്രിയത്തില്‍നിന്ന് നിര്‍ഗളിക്കുന്ന കാണാനാവുന്ന സ്രവങ്ങള്‍ക്കൊന്നും തന്നെ മഞ്ഞനിറമില്ലെന്ന വസ്തുതയാണ് വിശാലമായ ഇത്തരം ചര്‍ച്ചകളുടെ ഉല്‍ഭവത്തിന് നിമിത്തമായത്.

സ്ത്രീകളുടെ ജനനേന്ദ്രിയത്തില്‍നിന്ന് പുറത്തുവരുന്ന സ്രവങ്ങള്‍ മൂന്നെണ്ണമാണ്. തന്റെ ശരീരം ലൈംഗികബന്ധത്തിന് സജ്ജമായിയെന്ന് അറിയിച്ചുകൊണ്ട് സ്ത്രീജനനേന്ദ്രിയത്തില്‍നിന്ന് കിനിഞ്ഞിറങ്ങുന്ന ബര്‍ത്തോലിന്‍ സ്രവം(Bartholin fluid) ആണ് ഒന്നാമത്തേത്. യോനീമുഖത്തിനകത്തായി സ്ഥിതി ചെയ്യുന്ന പയര്‍വിത്തിന്റെ വലിപ്പത്തിലുള്ള രണ്ട് ബര്‍ത്തോലിന്‍ഗ്രന്ഥികള്‍ സ്ത്രീശരീരം ലൈംഗികമായി ഉത്തേജിപ്പിക്കപ്പെടുമ്പോള്‍ പുറപ്പെടുവിക്കുന്ന ഈ സ്രവത്തിന് നിറമില്ല. രതിമൂര്‍ച്ചയുടെ അവസരത്തില്‍ ചില സ്ത്രീകളുടെ ജനനേന്ദ്രിയത്തില്‍നിന്ന് പുറത്തുവരുന്ന പാരായുറിത്രല്‍ സ്രവമാണ്(Para urethral fluid) രണ്ടാമത്തെ യോനീ സ്രവം. യോനിയുടെ ആന്തരികഭിത്തിയില്‍ സ്ഥിതി ചെയ്യുന്ന പാരായുറിത്രല്‍ ഗ്രന്ഥികളില്‍നിന്നു വളരെ ചെറിയ അളവില്‍മാത്രം പുറത്തുവരുന്ന ഈ സ്രവം താരതമ്യേന കട്ടിയുള്ളതും വെള്ള നിറത്തിലുള്ളതുമായിരിക്കും. സ്ത്രീ ജനനേന്ദ്രിയത്തെ എല്ലായ്‌പ്പോഴും വരളാതെ സൂക്ഷിക്കുന്ന സെര്‍വിക്കല്‍ ശ്ലേഷ്മ (Cervical mucus) ആണ് മൂന്നാമത്തെ യോനീ സ്രവം. അണ്ഡോല്‍സര്‍ജനസമയമല്ലെങ്കില്‍ ഈ സ്രവം വഴുവഴുപ്പുള്ളതും നല്ല വെളുത്ത ക്രീം നിറത്തിലുള്ളതുമായിരിക്കും. അണ്ഡോല്‍സര്‍ജനത്തോടടുക്കുമ്പോള്‍ വെള്ളനിറം മങ്ങുകയും വഴുവഴുപ്പ് കുറയുകയും ചെയ്യുന്ന ഈ സ്രവം ഉല്‍സര്‍ജനസമയമാകുമ്പോഴേക്ക് ജലത്തെപ്പോലെ വര്‍ണരഹിതമാവുകയും മുട്ടയുടെ വെള്ളക്കരുവിനെപ്പോലെയായിത്തീരുകയും ചെയ്യും. അണുബാധയുണ്ടാകുമ്പോള്‍ മാത്രമാണ് സെല്‍വിക്കല്‍ ശ്ലേഷ്മത്തിന് മങ്ങിയ മഞ്ഞനിറമുണ്ടാകുന്നത്. സ്ത്രീജനനേന്ദ്രിയത്തില്‍നിന്ന് സാധാരണഗതിയില്‍ നിര്‍ഗളിക്കപ്പെടുന്ന മൂന്ന് സ്രവങ്ങളും വെളുത്തതോ നിറില്ലാത്തതോ ആണെന്നും ഹദീഥുകളില്‍ പറഞ്ഞ മഞ്ഞസ്രവമല്ല ഇവയെന്നും വ്യക്തമാണ്. ഇവയ്‌ക്കൊന്നുംതന്നെ കുഞ്ഞിന്റെ രൂപീകരണത്തില്‍ നേരിട്ട് പങ്കൊന്നുമില്ലതാനും.

കുഞ്ഞിന്റെ രൂപീകരണത്തിന് നിമിത്തമാകുന്ന സ്രവമെന്താണ് എന്ന ചോദ്യത്തിന് ഉത്തരം കാണാന്‍ ശ്രമിക്കുമ്പോഴാണ് ഹദീഥുകളില്‍ പറഞ്ഞ മഞ്ഞ സ്രവമേതാണെന്ന് നമുക്ക് മനസ്സിലാവുക. ആര്‍ത്തവചക്രത്തിന്റെ പതിനാലാം ദിവസം അണ്ഡാശയത്തിനകത്തെ പൂര്‍ണ വളര്‍ച്ചയെത്തിയ ഫോളിക്കിളില്‍ പ്രത്യക്ഷപ്പെടുന്ന ദ്വാരത്തിലൂടെ പ്രായപൂര്‍ത്തിയെത്തിയ അണ്ഡത്തെവഹിച്ചുകൊണ്ട് ഫോളിക്കുളാര്‍ ദ്രവവും ക്യൂമുലസ് കോശങ്ങളും പുറത്തേക്ക് തെറിച്ച് ഫലോപ്പിയന്‍ നാളിയുടെ അറ്റത്തുള്ള ഫിംബ്രയകളില്‍ പതിക്കുന്നതിനാണ് അണ്ഡോല്‍സര്‍ജനം (Ovulation) എന്നു പറയുന്നത്. രതിമൂര്‍ച്ചയോടനുബന്ധിച്ച് പുരുഷശരീരത്തില്‍ നടക്കുന്ന ശുക്ലസ്ഖലന(Ejaculation) ത്തിന് തുല്യമായി സ്ത്രീശരീരത്തില്‍ നടക്കുന്ന പ്രക്രിയയാണ് ഇതെങ്കിലും ഒരു ആര്‍ത്തവചക്രത്തില്‍ ഒരു തവണ മാത്രമാണ് ഇത് സംഭവിക്കുന്നത്. ശുക്ല സ്ഖലനവും അണ്ഡോല്‍സര്‍ജനവുമാണ് കുഞ്ഞിന്റെ സൃഷ്ടിക്ക് നിദാനമായി പുരുഷശരീരത്തിലും സ്ത്രീശരീരത്തിലും യഥാക്രമം സംഭവിക്കുന്ന രണ്ട് പ്രക്രിയകള്‍. പുരുഷബീജങ്ങളെ വഹിക്കുന്ന ശുക്ലദ്രാവകത്തെപ്പോലെ സ്ത്രീയുടെ അണ്ഡത്തെ വഹിക്കുന്ന ഫോളിക്കുളാര്‍ ദ്രവവും കുഞ്ഞിന്റെ നിര്‍മാണത്തിന് നിമിത്തമാകുന്ന ദ്രാവകമാണ്. ഹദീഥുകളില്‍ പറഞ്ഞ കുഞ്ഞിന്റെ സൃഷ്ടിക്ക് കാരണമായ സ്ത്രീസ്രവം അണ്ഡത്തെ വഹിക്കുന്ന ഫോളിക്കുളാര്‍ ദ്രാവകമാണെന്നാണ് ഇത് വ്യക്തമാക്കുന്നത്. അങ്ങനെയാണെങ്കില്‍ പുരുഷദ്രാവകം വെളുത്തതും സ്ത്രീദ്രാവകം മഞ്ഞയുമെന്ന് പരാമര്‍ശത്തിന്റെ വെളിച്ചത്തില്‍ പരിശോധിക്കുമ്പോള്‍ ഫോളിക്കുളാര്‍ ദ്രാവകത്തിന്റെ നിറം മഞ്ഞയായിരിക്കണം. എന്നാല്‍ എന്താണ് വസ്തുത?

പ്രായപൂര്‍ത്തിയെത്തുന്നതിനുമുമ്പുള്ള അണ്ഡാവസ്ഥയായ അണ്ഡത്തെ(Oocyte) സംരക്ഷിക്കുകയും വളര്‍ത്തിക്കൊണ്ടുവന്ന് ബീജസങ്കലനത്തിന് പറ്റിയ അണ്ഡമാക്കിത്തീര്‍ക്കുകയും ചെയ്യുകയാണ് ഫോളിക്കിളിന്റെ ധര്‍മം. പെണ്‍കുഞ്ഞ് ജനിക്കുമ്പോള്‍ തന്നെ അവളുടെ അണ്ഡാശയത്തിലുള്ള പ്രായപൂര്‍ത്തിയെത്താത്ത അണ്ഡകങ്ങളെ പൊതിഞ്ഞ് ആദിമ ഫോളിക്കിളുകളുണ്ടാവും (Primordial follicles).  അവള്‍ പ്രായപൂര്‍ത്തിയാകുന്നതോടെ ഇതില്‍ ചില ഫോളിക്കിളുകള്‍ വളര്‍ന്നുവരികയും ഓരോ ആര്‍ത്തവചക്രത്തിന്റെയും ശരാശരി 14-16 ദിവസങ്ങള്‍ കഴിഞ്ഞ് പൊട്ടി പൂര്‍ണവളര്‍ച്ചയെത്തിയ അണ്ഡത്തെ (Ovum) പുറത്തുവിടുന്നതോടെ അവയുടെ ധര്‍മം അവസാനിക്കുകയും ചെയ്യുന്നു. ജനനസമയത്തുള്ള ഏകദേശം 1,80,000 ഫോളിക്കിളുകളില്‍ നാനൂറെണ്ണത്തോളം മാത്രമാണ് അണ്ഡോല്‍സര്‍ജനത്തിനുമുമ്പത്തെ വളര്‍ച്ചയെത്തുവാനുള്ള ഭാഗ്യമുണ്ടാകുന്നത്. പ്രസ്തുത വളര്‍ച്ചയ്ക്ക് വ്യത്യസ്തങ്ങളായ ഘട്ടങ്ങളുണ്ട്. ഇതിലെ ഓരോ ഘട്ടങ്ങളിലും അതു കടന്നുപോകാന്‍ കഴിയാത്ത ഫോളിക്കിളുകള്‍ മരിച്ചുപോകുന്നുണ്ട്. ഓരോ ആര്‍ത്തവചക്രത്തിലും ഇരുപതോളം ഫോളിക്കിളുകള്‍ വളര്‍ച്ചയെത്തുന്നുവെങ്കിലും ഒരെണ്ണത്തിന് മാത്രമാണ് ഫോളിക്കിള്‍ മരണമായ അട്രീഷ്യ(atresia)യില്‍നിന്ന് രക്ഷപ്പെട്ട് അണ്ഡോല്‍സര്‍ജനത്തിന് കഴിയുന്നത്. അട്രീഷ്യയില്‍ നിന്ന് രക്ഷപ്പെട്ട് അണ്ഡോല്‍സര്‍ജനത്തിന് കഴിയുന്ന ഫോളിക്കിളുകള്‍ രണ്ട് ദശകളിലൂടെയാണ് കടന്നുപോകുന്നത്. അണ്ഡോല്‍സര്‍ജനത്തിലൂടെ അവസാനിക്കുന്ന ഒന്നാമത്തെ ദശയെ ഫോളിക്കുളാര്‍ ദശfollicular phase) എന്നും അതിനുശേഷമുള്ള ദശയെ ലൂടിയല്‍ ദശ (luteal phase) എന്നുമാണ് വിളിക്കുക. ആര്‍ത്തവം മുതല്‍ അണ്ഡോല്‍സര്‍ജനം വരെയുള്ള ഫോളിക്കുളാര്‍ ദശയില്‍ അണ്ഡകം പൂര്‍ണവളര്‍ച്ചയെത്തിയ അണ്ഡമായിത്തീരുന്നതിനും യഥാരൂപത്തിലുള്ള അണ്ഡോല്‍സര്‍ജനം നടക്കുന്നതിനും വേണ്ടി വ്യത്യസ്തങ്ങളായ പ്രക്രിയകള്‍ നടക്കേണ്ടതുണ്ട്. ഈ പ്രക്രിയകളുടെ അവസാനമായി ശരീരത്തിലെ ഈസ്ട്രജന്‍ നില പരമാവധി ഉയരുകയും ലൂറ്റിനൈസിംഗ് ഹോര്‍മോണ്‍ (LH), ഫോളിക്കിള്‍ സ്റ്റിമുലേറ്റിംഗ് ഹോര്‍മോണ്‍ (FSH) എന്നീ ഹോര്‍മോണുകളെ ഇതിന്റെ ഫലമായി ഉത്പാദിപ്പിക്കുകയും ചെയ്യുന്നു. 24 മുതല്‍ 36 വരെ മണിക്കൂറുകള്‍ നീണ്ടുനില്‍ക്കുന്ന ഈ പ്രക്രിയയുടെ അന്ത്യം കുറിച്ചുകൊണ്ടാണ് അണ്ഡം വഹിക്കുന്ന പൂര്‍ണവളര്‍ച്ചയെത്തിയ ഫോളിക്കിളില്‍(Ovarian follicle) സ്റ്റിഗ്മയെന്ന് പേരുള്ള ദ്വാരമുണ്ടാവുകയും അത് പൊട്ടി അണ്ഡത്തെ വഹിച്ചുകൊണ്ട് ഫോളിക്കുളാര്‍ ദ്രവം പുറത്തേക്ക് തെറിക്കുകയും ചെയ്യുന്നത്. ഈ പുറത്തേക്കു തെറിക്കല്‍ പ്രക്രിയക്കാണ് അണ്ഡോല്‍സര്‍ജനം (Ovulation)എന്നു പറയുക.

ഫോളിക്കുളാര്‍ ദശയിലുടനീളം നടക്കുന്ന അണ്ഡവളര്‍ച്ചയ്ക്കും അതിന് ഉല്‍സര്‍ജിക്കാനാവശ്യമായസംവിധാനങ്ങളൊരുക്കുന്നതിനും നിമിത്തമാകുന്നത് FSHപ്രവര്‍ത്തനങ്ങളാണ്. പ്രസ്തുത ഉത്പാദനത്തോടനുബന്ധിച്ചാണ് ഹൈപ്പോതലാമസില്‍നിന്നുള്ള ഗൊണാടോട്രോപിന്‍ റിലീസിംഗ് ഹോര്‍മോണിന്റെ(GnRH) പ്രേരണയാല്‍ പിറ്റിയൂട്ടറിയില്‍നിന്ന് LHന്റെ ഉത്പാദനം നടക്കുന്നത്. ഈ ഹോര്‍മോണ്‍ ഉത്പാദിപ്പിക്കുന്ന പ്രോട്ടീന്‍ വിഘാടക രസങ്ങളായ പ്രോട്ടിയോലിറ്റിക് എന്‍സൈമുകളാണ്ഫോ(Proteolytic enzymes) ളിക്കിളിലുണ്ടാവുന്ന ദ്വാരമായ സ്റ്റിഗ്മക്ക് കാരണമാകുന്നത്. അണ്ഡോല്‍സര്‍ജനത്തിനുശേഷമുള്ള ഫോളിക്കിള്‍ അവശിഷ്ടങ്ങളെ നിയന്ത്രിക്കുന്നതും പ്രധാനമായി ഈ ഹോര്‍മോണാണ്. ലൂട്ടിയല്‍ ദശയില്‍ അണ്ഡം നഷ്ടപ്പെട്ട ഫോളിക്കിള്‍ അവശിഷ്ടങ്ങള്‍ കോര്‍പസ് ലൂടിയം(Lorpus Luteum) ആയിത്തീരുകയും മാതൃസ്വഭാവങ്ങളെ ഉദ്ദീപിക്കുന്ന പ്രോജസ്റ്ററോണ്‍ (Progesterone) ഹോര്‍മോണിന്റെ വര്‍ധിതമായ ഉത്പാദനത്തിന് നിമിത്തമാവുകയും ചെയ്യുന്നു.

എന്താണീ ലൂറ്റിനൈസിംഗ് ഹോര്‍മോണ്‍? മഞ്ഞയെന്ന് അര്‍ത്ഥം വരുന്ന ലൂറ്റിയസ് (Luteus) എന്ന ലാറ്റിന്‍ പദത്തിന്റെ നപുംസകരൂപമായ ലൂറ്റിയത്തില്‍നിന്നാണ് (Luteum) ലൂറ്റിനൈസ് (Luteinize)എന്ന ക്രിയയുണ്ടായിരിക്കുന്നത്. കോര്‍പ്പസ് ലൂടിയത്തിന്റെ നിര്‍മിതിക്ക് നിമിത്തമായ പ്രവര്‍ത്തനങ്ങള്‍ക്കാണ് സാങ്കേതികമായി ലൂറ്റിനൈസ് എന്ന് പറയുന്നതെങ്കിലും പദപരമായി അതിനര്‍ത്ഥം 'മഞ്ഞയാക്കുന്നത്' എന്നാണ്. ലൂറ്റിനൈസിംഗ് ഹോര്‍മോണിന്റെ പ്രവര്‍ത്തനഫലമായാണ് ഫോളിക്കുളാര്‍ ദശ പിന്നിട്ട ഫോളിക്കിള്‍ അവശിഷ്ടങ്ങള്‍ കോര്‍പസ് ലൂടിയം ആയിത്തീരുന്നത്. കോര്‍പസ് ലൂടിയം എന്ന പദദ്വയത്തിനര്‍ത്ഥം മഞ്ഞ വസ്തുവെന്നാണ് (Yellow body). ലൂടിയല്‍ ദശയിലേക്ക് കടന്ന അണ്ഡം നഷ്ടപ്പെട്ട ഫോളിക്കിള്‍ അവശിഷ്ടങ്ങളെല്ലാം കൂടി രണ്ടു മുതല്‍ അഞ്ചു സെന്റീമീറ്റര്‍ വരെ വ്യാസത്തില്‍ ശരീരത്തില്‍ ഏതാനും ദിവസങ്ങള്‍ കൂടി അവശേഷിക്കും. മനുഷ്യരില്‍ ഇത് ഓറഞ്ചു നിറത്തിലാണ് കാണപ്പെടുന്നത്. അണ്ഡോല്‍സര്‍ജനത്തിന്റെ അവസാനഘട്ടത്തില്‍ ഉത്പാദിപ്പിക്കപ്പെടുന്ന LH അതിന്റെ പ്രവര്‍ത്തനമാരംഭിക്കുകയും ഫോളിക്കുളാര്‍ ദ്രവത്തെ മഞ്ഞവല്‍ക്കരിക്കുകയും ചെയ്യും. ഫോളിക്കിളിലെ സ്റ്റിഗ്മ പൊട്ടി അണ്ഡത്തോടെ പുറത്തേക്ക് തെറിക്കുന്ന ഫോളിക്കുളാര്‍ ദ്രാവകത്തിന്റെ നിറം മഞ്ഞയായിരിക്കും. പുരുഷ ശുക്ലവുമായി താരതമ്യം ചെയ്യുമ്പോള്‍ കട്ടിയില്ലാത്തതും മഞ്ഞ നിറത്തിലുള്ളതുമായ ദ്രാവകമാണ് ഫോളിക്കിള്‍ പൊട്ടി പുറത്തേക്കൊഴുകുന്ന കുഞ്ഞിന്റെ നിര്‍മാണത്തിന് നിമിത്തമാകുന്ന സ്ത്രീസ്രവം എന്നര്‍ത്ഥം.

കുഞ്ഞിന്റെ സൃഷ്ടിക്ക് നിമിത്തമാകുന്ന സ്ത്രീസ്രവം മഞ്ഞനിറത്തിലുള്ളതും കട്ടി കുറഞ്ഞതുമാണെന്ന പ്രവാചകവചനം എത്രമാത്രം കൃത്യമാണെന്ന് നമുക്ക് ബോധ്യപ്പെടുന്നത് ഫോളിക്കിള്‍ രൂപാന്തീകരണത്തെക്കുറിച്ച (folliculogenesis) പുതിയ പഠനങ്ങളുടെ വെളിച്ചത്തിലാണ്. കോര്‍പ്പസ് ലൂടിയത്തെയും ലൂറ്റിനൈസിംഗ് ഹോര്‍മോണിന്റെ ധര്‍മത്തെയുമെല്ലാം കുറിച്ച് കൃത്യമായി മനസ്സിലാക്കാന്‍ കഴിഞ്ഞത് കഴിഞ്ഞ നൂറ്റാണ്ടിന്റെ അന്ത്യത്തിലും ഈ നൂറ്റാണ്ടിന്റെ തുടക്കത്തിലുമായി നടന്ന സാങ്കേതിക വിപ്ലവങ്ങളുടെ ഫലമായി ഉണ്ടായിവന്ന സൂക്ഷ്മദര്‍ശിനികളുപയോഗിച്ചുള്ള പഠനങ്ങള്‍ വഴിയാണ്. ഇപ്പോള്‍ മാത്രം നമുക്ക് മനസ്സിലായ ഇക്കാര്യം എങ്ങനെ പ്രവാചകന്‍(സ) അറിഞ്ഞുവെന്നതിന് അദ്ദേഹം തന്നെ മറുപടി പറഞ്ഞിട്ടുണ്ട്. 'അയാള്‍ എന്നോട് ചോദിച്ച കാര്യങ്ങളെക്കുറിച്ചൊന്നും അല്ലാഹു അറിയിച്ചുതരുന്നതുവരെ എനിക്ക് യാതൊരു വിവരവുമുണ്ടായിരുന്നില്ല' എന്ന പ്രവാചക പ്രസ്താവനയില്‍നിന്ന് നബിവചനങ്ങളുടെ സ്രോതസ് എന്താണെന്ന് മനസ്സിലാക്കാനാവും. തന്റെ ചോദ്യങ്ങള്‍ക്ക് കൃത്യമായി മറുപടി നല്‍കിയ നബി (സ)യോട് 'താങ്കള്‍ പറഞ്ഞത് സത്യമാണ്; താങ്കളൊരു ദൈവദൂതന്‍ തന്നെയാണ്'(സ്വഹീഹ്മുസ്‌ലിം) എന്ന് സാക്ഷ്യപ്പെടുത്തിക്കൊണ്ടാണ് ജൂതപണ്ഡിതന്‍ തിരിച്ചുപോയതെന്ന വസ്തുത ശ്രദ്ധേയമാണ്. പൂര്‍വവേദങ്ങളെക്കുറിച്ച് അറിയാവുന്നവര്‍ക്ക് മുഹമ്മദ് നബി(സ)യെപ്പറ്റി സ്വന്തം മക്കളെ അറിയുന്നതുപോലെ അറിയാന്‍ കഴിഞ്ഞിരുന്നുവെന്ന ക്വുര്‍ആന്‍ പ്രസ്താവനയുടെ സത്യത കൂടി ഇവിടെ വെളിപ്പെടുന്നുണ്ട്: ''നാം വേദം നല്‍കിയിട്ടുള്ളവര്‍ക്ക് സ്വന്തം മക്കളെ അറിയാവുന്നത് പോലെ അദ്ദേഹത്തെ (റസൂലിനെ) അറിയാവുന്നതാണ്. തീര്‍ച്ചയായും അവരില്‍ ഒരു വിഭാഗം അറിഞ്ഞുകൊണ്ടുതന്നെ സത്യം മറച്ചുവെക്കുകയാകുന്നു.'' (ക്വുര്‍ആന്‍ 2:146)

വിഷയവുമായി ബന്ധപ്പെട്ട വീഡിയോ
പ്രപഞ്ചത്തിന്റെ വികാസത്തെക്കുറിച്ച് ഖുര്‍ആനിലുണ്ടെന്ന് പല മുസ്‌ലിം ഗ്രന്ഥകാരന്മാരും എഴുതിയിട്ടുണ്ട്. ഖുര്‍ആന്‍ 51:47 ആണ് അതിന് തെളിവായി കൊടുക്കാറുള്ളത്. എന്നാല്‍ പ്രസ്തുത സൂക്തത്തിലെ 'വ ഇന്നാ ലമൂസ്ഊന്‍' എന്നതിന്റെ അര്‍ഥം 'ദൈവം വിപുലമായ കഴിവുള്ളവനാണ്' എന്നാണെന്ന് ഒരു മുസ്‌ലിം നാമധാരിയായ യുക്തിവാദി എഴുതിയതായി കണ്ടു. വിശദീകരണം പ്രതീക്ഷിക്കുന്നു.
  • 'ഔസ'അ' എന്ന ക്രിയയില്‍നിന്നുള്ള കര്‍തൃനാമമാണ് 'മൂസിഅ്': ഈ ക്രിയ അകര്‍മകമായും സകര്‍മകമായും പ്രയോഗിക്കാറുണ്ട്. അകര്‍മകമാകുമ്പോള്‍ അതിന്റെ അര്‍ത്ഥം ജീവിത സൗകര്യവും ഐശ്വര്യവും ഉള്ളവനായി തീര്‍ന്നു എന്നാണ്. സകര്‍മകമാകുമ്പോള്‍ അതിന്റെ അര്‍ഥം വിശാലമാക്കി എന്നും വികസിപ്പിച്ചു എന്നുമാണ്. ബൈറൂത്തിലെ കാത്തലിക് പ്രസ് പ്രസിദ്ധീകരിച്ച 'അല്‍മുന്‍ജിദ്' എന്ന നിഘണ്ടുവില്‍ 'ദ്വയ്യക്വ' (ഇടുങ്ങിയതാക്കി) എന്നതിന്റെ വിപരീതമാണ് 'ഔസഅ' എന്ന് രേഖപ്പെടുത്തിയിരിക്കുന്നു. ഇതനുസരിച്ച് 'മൂസിഅ്' എന്നതിന്റെ അര്‍ത്ഥം വിശാലമാക്കുന്നവന്‍ അഥവാ വികസിപ്പിക്കുന്നവന്‍ എന്നാകുന്നു. 'മൂസിഊന്‍' എന്നത് പൂജക ബഹുവചനമാകുന്നു.
വിഷയവുമായി ബന്ധപ്പെട്ട വീഡിയോ

ക്ഷത്രങ്ങളും ഗ്രഹങ്ങളുമടങ്ങുന്ന ആകാശഗോളങ്ങളെല്ലാം പൊതുവെ ഗോളാകൃതിയുള്ളവയാണ്. അത് കൊണ്ട് തന്നെ പ്രപഞ്ചത്തിലെ ദൈവാനുഗ്രഹങ്ങളെക്കുറിച്ച പരാമര്ശങ്ങൾക്കിടയിൽ ഭൂമിയുടെ ഗോളാകൃതി വലിയൊരു ചർച്ചാവിഷയമല്ല. ഈ ആകാശഗോളങ്ങളില്‍ നിന്ന് ഭൂമിക്കുള്ള സവിശേഷത അതില്‍ ജീവന്‍ നിലനില്‍ക്കുന്നുവെന്നതാണ്. ജീവന്‍ നിലനില്‍ക്കുവാന്‍ പാകത്തില്‍ ഭൂമിയെ സംവിധാനിച്ചതിനെക്കുറിച്ചാണ് അല്ലാഹു നല്‍കിയ അനുഗ്രഹങ്ങളെക്കുറിച്ചു പറയുമ്പോള്‍ ക്വുര്‍ആന്‍ ഊന്നുന്നത്.

''നിങ്ങള്‍ക്ക് വേണ്ടി ഭൂമിയെ മെത്തയും ആകാശത്തെ മേല്‍പുരയുമാക്കിത്തരികയും ആകാശത്ത് നിന്ന് വെള്ളം ചൊരിഞ്ഞുതന്നിട്ട് അത് മുഖേന നിങ്ങള്‍ക്ക് ഭക്ഷിക്കുവാനുള്ള കായ്കനികള്‍ ഉല്‍പാദിപ്പിച്ചു തരികയും ചെയ്ത (നാഥനെ നിങ്ങൾ ആരാധിക്കുക ). ഇതെല്ലാം അറിയുന്ന നിങ്ങള്‍ അല്ലാഹുവിന് സമന്‍മാരെ ഉണ്ടാക്കുകയും അരുത്.'' (2:22)

പ്രപഞ്ചത്തിലുള്ള എന്തിനെക്കുറിച്ചും നാം പറയുമ്പോള്‍ ആപേക്ഷികമായാണ്, കേവലമായല്ല പരാമര്‍ശിക്കേണ്ടത് എന്നാണ് ആധുനികഭൗതികം പഠിപ്പിക്കുന്നത്. സൗരയൂഥത്തിലെ ഭൂമിക്കുപുറത്തുള്ള ഏതെങ്കിലും ഒരു കേന്ദ്രത്തിന് അപേക്ഷികമായി ഭൂമി ഉരുണ്ടതാണെന്നു പറയാം. നമ്മുടെ ഗ്യാലക്‌സിക്കു പുറത്തുള്ള ഒരു നിരീക്ഷകന് ആപേക്ഷികമായി ഭൂമിയുടെ ആകൃതിയെക്കുറിച്ച് പരാമര്‍ശിക്കുമ്പോള്‍ അതിന്റെ സ്വയംഭ്രമണവും പരിക്രമണവും ഗാലക്തികകേന്ദ്രത്തെ ചുറ്റിയുള്ള സൂര്യനോടൊപ്പമുള്ള ചലനവുമെല്ലാം പരിഗണിക്കേണ്ടിവരും. ഭൂമിക്കുപുറത്തെ നിരീക്ഷകന് ആപേക്ഷികമായി ഭൂമി ഗോളാകൃതിയിലായിരിക്കുന്നതുപോലെ ഭൂമിയില്‍ ജീവിക്കുന്ന നിരീക്ഷകന് ആപേക്ഷികമായി ഭൂമി പരന്നാണുള്ളത്. പരന്ന ഭൂമിയെ പരിഗണിച്ചുകൊണ്ടാണ് ഭൂമിയിലെ നമ്മുടെ പ്രവര്‍ത്തനങ്ങളെല്ലാം നാം ആസൂത്രണം ചെയ്യുന്നത്. പരന്നഭൂമിയെ പരിഗണിച്ചുകൊണ്ട് ക്രിസ്തുവിന് മൂന്നുനൂറ്റാണ്ടുകള്‍ക്ക് മുമ്പ് ഗ്രീക്ക് ഗണിതജ്ഞനായ യൂക്ലിഡ് നിര്‍ധരിച്ചെടുത്ത തത്വങ്ങളില്‍ തന്നെ രണ്ടായിരത്തിലധികം വര്‍ഷങ്ങള്‍ കഴിഞ്ഞിട്ടും ആധുനിക ജ്യാമിതി നിലനില്‍ക്കുന്നതും അതുപ്രകാരം ഭൂമിയിലെ നമ്മുടെ നിര്‍മാണങ്ങളെല്ലാം നാം ആസൂത്രണം ചെയ്യുന്നതും മനുഷ്യര്‍ക്ക് ആപേക്ഷികമായി ഭൂമി പരന്നതായതുകൊണ്ടാണ്. അല്ലാഹു നല്‍കിയ അനുഗ്രഹങ്ങളെക്കുറിച്ചു പറയുമ്പോള്‍ ഭൂമിയെ അല്ലാഹു പരത്തിയതായി ക്വുര്‍ആന്‍ പരാമര്‍ശിക്കുന്നുണ്ട്.

''ഭൂമിയിലേക്ക് (അവര്‍ നോക്കുന്നില്ലേ?) അത് എങ്ങനെ പരത്തപ്പെട്ടിരിക്കുന്നുവെന്ന്'' (88:20)

''ഭൂമിയാകട്ടെ നാം അതിനെ വികസിപ്പിക്കുകയും, അതില്‍ ഉറച്ചുനില്‍ക്കുന്ന പര്‍വ്വതങ്ങള്‍ നാം സ്ഥാപിക്കുകയും കൗതുകമുള്ള എല്ലാ സസ്യവര്‍ഗങ്ങളും നാം അതില്‍ മുളപ്പിക്കുകയും ചെയ്തിരിക്കുന്നു.'' (50:7)

''അതെ, നിങ്ങള്‍ക്ക് വേണ്ടി ഭൂമിയെ ഒരു തൊട്ടിലാക്കുകയും നിങ്ങള്‍ നേരായ മാര്‍ഗം കണ്ടെത്താന്‍ വേണ്ടി നിങ്ങള്‍ക്കവിടെ പാതകളുണ്ടാക്കിത്തരികയും ചെയ്തവന്‍.'' (43:10)

''നിങ്ങള്‍ക്ക് വേണ്ടി ഭൂമിയെ തൊട്ടിലാക്കുകയും, നിങ്ങള്‍ക്ക് അതില്‍ വഴികള്‍ ഏര്‍പെടുത്തിത്തരികയും, ആകാശത്ത് നിന്ന് വെള്ളം ഇറക്കിത്തരികയും ചെയ്തവനത്രെ അവന്‍. അങ്ങനെ അത് (വെള്ളം) മൂലം വ്യത്യസ്ത തരത്തിലുള്ള സസ്യങ്ങളുടെ ജോടികള്‍ നാം (അല്ലാഹു) ഉല്‍പാദിപ്പിക്കുകയും ചെയ്തിരിക്കുന്നു.'' (20:53)

''ഭൂമിയാകട്ടെ നാം അതിനെ ഒരു വിരിപ്പാക്കിയിരിക്കുന്നു. എന്നാല്‍ അത് വിതാനിച്ചവന്‍ എത്ര നല്ലവന്‍!'' (51:48)

''അല്ലാഹു നിങ്ങള്‍ക്കു വേണ്ടി ഭൂമിയെ ഒരു വിരിപ്പാക്കുകയും ചെയ്തിരിക്കുന്നു.'' (71:19)

''ഭൂമിയെ നാം ഒരു വിരിപ്പാക്കിയില്ലേ? പര്‍വ്വതങ്ങളെ ആണികളാക്കുകയും (ചെയ്തില്ലേ?).'' (78:6,7)

മനുഷ്യന് ആപേക്ഷികമായി, അവനും മറ്റുജീവജാലങ്ങള്‍ക്കും ജീവിക്കുവാന്‍ തക്കരൂപത്തില്‍ ഭൂമിയെ വിശാലമാക്കുകയും വികസിപ്പിക്കുകയും വിതാനിക്കുകയും പരത്തുകയും ചെയ്തതിനെക്കുറിച്ചു പറയുമ്പോള്‍ തന്നെ ഭൂമിക്കു പുറത്തെ നിരീക്ഷകന് ആപേക്ഷികമായുള്ള ഭൂമിയുടെ ഗോളാകൃതിയെപ്പറ്റിയും ക്വുര്‍ആന്‍ സൂചന നല്‍കുന്നുണ്ട്.

''ആകാശങ്ങളും ഭൂമിയും അവന്‍ യാഥാര്‍ത്ഥ്യപൂര്‍വ്വം സൃഷ്ടിച്ചിരിക്കുന്നു. രാത്രിയെക്കൊണ്ട് അവന്‍ പകലിന്‍മേല്‍ ചുറ്റിപ്പൊതിയുന്നു. പകലിനെക്കൊണ്ട് അവന്‍ രാത്രിമേലും ചുറ്റിപ്പൊതിയുന്നു. സൂര്യനെയും ചന്ദ്രനെയും അവന്‍ നിയന്ത്രണവിധേയമാക്കുകയും ചെയ്തിരിക്കുന്നു. എല്ലാം നിശ്ചിതമായ പരിധിവരെ സഞ്ചരിക്കുന്നു. അറിയുക: അവനത്രെ പ്രതാപിയും ഏറെ പൊറുക്കുന്നവനും.'' (39:5)

രാത്രിയും പകലും പരസ്പരം ചുറ്റുന്നതിനെക്കുറിച്ച് പരാമര്‍ശിക്കുമ്പോള്‍ 'യുകവ്വിറു' എന്നാണ് ക്വുര്‍ആന്‍ ഇവിടെ പറഞ്ഞിരിക്കുന്നത്. ഒരു ഗോളത്തിന്‍മേല്‍ ചുറ്റുന്നതിനെപ്പറ്റിയാണ് ഇങ്ങനെ പരാമര്‍ശിക്കുക. ഉദയസ്ഥാനങ്ങളെയും അസ്തമയസ്ഥാനങ്ങളെയും കുറിച്ചു പരാമര്‍ശിക്കുമ്പോഴും ഭൂമിയുടെ ഗോളാകൃതിയെക്കുറിച്ച് ക്വുര്‍ആന്‍ സൂചന നല്‍കുന്നുണ്ട്.

'' ഉദയസ്ഥാനങ്ങളുടെ രക്ഷിതാവും ആയിട്ടുള്ളവന്‍''. (37:5)

''രണ്ട് ഉദയസ്ഥാനങ്ങളുടെ രക്ഷിതാവും രണ്ട് അസ്തമന സ്ഥാനങ്ങളുടെ രക്ഷിതാവുമാകുന്നു അവന്‍.'' (55:17)

ഒന്നിലധികം ഉദയസ്ഥാനങ്ങളും അസ്തമയസ്ഥാനങ്ങളുമുണ്ടാവുക ഭൂമി ഗോളമായിരിക്കുമ്പോള്‍ മാത്രമാണല്ലോ?

ആകാശഗോളങ്ങളും ഭൂമിയുമെല്ലാം ഗോളാകൃതിയിലുള്ളവയാണെന്ന് ആദ്യകാല മുസ്‌ലിംകള്‍ മനസ്സിലാക്കിയിരുന്നതായി ശൈഖുല്‍ ഇസ്‌ലാം ഇബ്‌നുതീമിയ തന്റെ ഫതാവയില്‍ പരാമര്‍ശിക്കുന്നുണ്ട് (6/586-587). ഇമാം അഹ്മദിന്റ (റ) ശിഷ്യനായിരുന്ന അബുല്‍ഹുസൈന്‍ അഹ്മദ്ബ്‌നു ജഅ്ഫര്‍ ബ്‌നു മുനാദീ(റ)യും ഇമാം അബൂ മുഹമ്മദ്ബ്‌നു ഹസമും (റ) അബൂ ഫറാജ്ബ്‌നുല്‍ ജൗസി(റ)യുമെല്ലാം ഇക്കാര്യം വ്യക്തമാക്കുന്നുണ്ടെന്ന് അദ്ദേഹം സമര്‍ത്ഥിക്കുന്നു. പ്രവാചകശിഷ്യനായിരുന്ന ഇബ്‌നു അബ്ബാസില്‍ (റ) നിന്നുള്ള ചില പരാമര്‍ശങ്ങളും ഭൂമിയും മറ്റും ഉരുïതാണെന്നാണ് അദ്ദേഹം മനസ്സിലാക്കിയിരുന്നതെന്ന് വ്യക്തമാക്കുന്നുണ്ടെന്നും അദ്ദേഹം പറയുന്നുണ്ട്.

ഭൂമി ഒരു ഗോളമാണെന്ന് ആദ്യകാല മുസ്‌ലിം പണ്ഡിതന്‍മാര്‍ മനസ്സിലാക്കുകയും അതിന്റെ വെളിച്ചത്തില്‍ അവര്‍ ഗോളീയത്രികോണമിതി(Spherical trigonometry)രൂപീകരിക്കുകയും ചെയ്തതായും അതുപയോഗിച്ചാണ് ലോകത്തിലെ വ്യത്യസ്ത കോണുകളില്‍നിന്ന് മക്കയിലെ ഖിബ്‌ലയിലേക്കുള്ള ദിശ നിര്‍ണയിച്ചതെന്നും ചരിത്രകാരനായ ഡേവിഡ് എ. കിംഗ് തന്റെ അസ്‌ട്രോണമി ഇന്‍ദി സര്‍വീസ് ഓഫ് ഇസ്‌ലാം(Astronomy in the Service of Islam) എന്ന ഗ്രന്ഥത്തില്‍ പറയുന്നത്. ഭൂഗോളത്തിന്റെ ചുറ്റളവ് കണ്ടു പിടിക്കാനായി ഒരുപറ്റം മുസ്‌ലിം ഗോളശാസ്ത്രജ്ഞരെയും ഭൂമിശാസ്ത്രജ്ഞരെയും ഖലീഫ മഅ്മൂന്‍ ഉത്തരവാദിത്തപ്പെടുത്തിയതായും സിറിയയിലെ തദ്മൂറും റാഖ്ബയും തമ്മിലുള്ള ദൂരം അളന്ന് അവതമ്മില്‍ ഒരു ഡിഗ്രി അക്ഷാംശവ്യത്യാസമുണ്ടെന്ന് മനസ്സിലാക്കുകയും ഭൂമിയുടെ ചുറ്റളവ് 24000 മൈലുകളാണെന്ന് കണക്കാക്കുകയും ചെയ്തതായും മറ്റൊരുകൂട്ടം മുസ്‌ലിം ശാസ്ത്രജ്ഞരുടെ കണക്കുകള്‍പ്രകാരം ഭൂമിയുടെ ചുറ്റളവ് 40,284 കിലോമീറ്ററാണെന്നും ആധുനികയന്ത്രങ്ങളുപയോഗിച്ച് നാം ഇന്നുകണക്കാക്കുന്ന 40,068 കിലോമീറ്ററുമായി വളരെ അടുത്തുനില്‍ക്കുന്നതാണ് അവരുടെ കണക്കാക്കലെന്നത് അത്ഭുതകരമാണെന്നും ശാസ്ത്രചരിത്രകാരനായ അഡ്വേര്‍ഡ് എസ്.കെന്നഡി തന്റെ മാത്തമാറ്റിക്കല്‍ ജിയോഗ്രഫി എന്ന ഗ്രന്ഥത്തില്‍ (പുറം 185-201) നിരീക്ഷിക്കുന്നുണ്ട്.

 

ഭൂമിയെ പരത്തിയതായുള്ള ക്വുര്‍ആന്‍ പരാമര്‍ശത്തില്‍നിന്ന് അതൊരു ഗോളമല്ലെന്ന് ആദ്യകാല മുസ്‌ലിംകളൊന്നും മനസ്സിലാക്കിയിരുന്നില്ലെന്നുസാരം.

 

മനുഷ്യന് ജീവിക്കുവാൻ തക്ക രൂപത്തിൽ ഭൂമിയിൽ അല്ലാഹു ഏർപ്പെടുത്തിയ സംവിധാനങ്ങളെക്കുറിച്ച് പറയുമ്പോൾ അതിനെ വിരിപ്പും തൊട്ടിലുമാക്കിയാതായും പരത്തിയതായും വിതാനിച്ചതായുമെല്ലാം ക്വുർആൻ പറയുന്നുണ്ട്. ജീവന്‍ നിലനില്‍ക്കുവാനായി പ്രത്യേകം പടക്കപ്പെട്ട ഭൂമിയിലെ സംവിധാനങ്ങളെക്കുറിച്ചാണ് ഈ സൂക്തങ്ങൾ പരാമര്ശിക്കുന്നതെന്ന് അവ മനസ്സിരുത്തി വായിച്ചാൽ ആർക്കും ബോധ്യപ്പെടും. ചലനാത്മകമായ ഈ പ്രപഞ്ചത്തിൽ എത്രയെത്ര ചലനങ്ങള്‍ക്കാണ് ഭൂമി വിധേയമായിക്കൊണ്ടിരിക്കുന്നത്! ഭൂമിയുടെ സ്വയംഭ്രമണം. സുര്യനു ചുറ്റുമുള്ള പരിക്രമണം. ഗ്യാലക്‌സിക്കുചുറ്റും സൂര്യനോടൊപ്പം നടത്തുന്ന ചലനം. പ്രപഞ്ചകേന്ദ്രത്തെ ആസ്പദമാക്കി ഗ്യാലക്‌സി നടത്തുന്ന ഭ്രമണത്തോടൊപ്പമുള്ള ചലനം. പ്രപഞ്ചവികാസത്തിനനുപൂരകമായി നടത്തുന്ന ചലനം. ഇങ്ങനെ ചുരുങ്ങിയത് അഞ്ചുവിധം ചലനങ്ങള്‍ക്കെങ്കിലും ഭൂമി ഒരേസമയം വിധേയമാണ്.

ഇങ്ങനെ ചലനാത്മകമായ ഭൂമിയില്‍ തങ്ങള്‍ നിശ്ചലരും സുരക്ഷിതരുമാണെന്ന ബോധമുള്‍ക്കൊണ്ടുകൊണ്ടു ജീവിക്കുകയാണ് മനുഷ്യനടക്കമുള്ള ജീവജാലങ്ങള്‍. അവരെ സംബന്ധിച്ചിടത്തോളം ഭൂമി ഒരു മെത്ത തന്നെയാണ്. ഊഷരമായ മരുഭൂമിക്കുനടുവില്‍ ഉയര്‍പ്പെട്ട സകലവിധ സംവിധാനങ്ങളോടും കൂടിയ ഒരു ഗൃഹം പോലെയാണ് അവരെ സംബന്ധിച്ചിടത്തോളം കത്തുന്ന പ്രപഞ്ചത്തിന് നടുവിലുള്ള ഭൂമി. അതിശീഘ്രമായ ചലനങ്ങള്‍ തങ്ങളുടെ ജീവിതത്തിനാവശ്യമായ പ്രതിഭാസങ്ങള്‍ സൃഷ്ടിക്കുന്നുവെന്നതിനാല്‍ ജീവികള്‍ക്ക് ഭൂമിയൊരു തൊട്ടിലാണ്. നമുക്ക് ജീവിക്കാനനുകൂലമായ കാലാവസ്ഥയുള്‍ക്കൊള്ളുന്ന മറ്റൊരു ആകാശഗോളവും ഇതേവരെ കണ്ടെ ത്തിയിട്ടില്ല. ജീവജാലങ്ങള്‍ക്കു വസിക്കാന്‍ പറ്റിയ ഒരേയൊരു വീട് തന്നെയാണ് ഭൂമി. അള്‍ട്രാ വയലറ്റ് രശ്മികളേല്‍ക്കാതെ, ഉല്‍ക്കാ നിപാതങ്ങളേല്‍ക്കാതെ, ഓക്‌സിജന്‍ യഥേഷ്ടം ശ്വസിച്ചും വെള്ളം യഥേഷ്ടം കുടിച്ചും ഭക്ഷണപാനീയങ്ങളുയോഗിച്ചും വിശ്രമിക്കാന്‍ കഴിയുന്ന പ്രപഞ്ചത്തിലെ ഏക മെത്തയാണിത്. ഈ സംവിധാനങ്ങള്‍ക്കുപിന്നില്‍ സര്‍വജ്ഞനും സര്‍വശക്തനുമായ ഒരു സംവിധായകന്റെ കരവിരുതുകളല്ലാതെ മറ്റെന്താണ് ചിന്തിക്കുന്നവര്‍ക്ക് കാണാന്‍ കഴിയുന്നത്? ഇതു സൂചിപ്പിക്കുന്ന ക്വുര്‍ആന്‍ സൂക്തങ്ങള്‍ നോക്കുക:

''നിങ്ങള്‍ക്ക് വേണ്ടി ഭൂമിയെ മെത്തയും ആകാശത്തെ മേല്‍പുരയുമാക്കിത്തന്ന (നാഥന്‍)'' (2:22)

''അവനാണ് ഭൂമിയെ വിശാലമാക്കുകയും, അതില്‍ ഉറച്ചുനില്‍ക്കുന്ന പര്‍വ്വതങ്ങളും നദികളും ഉണ്ടാക്കുകയും ചെയ്തവന്‍.'' (13:3)

''ഭൂമിയെ നാം വിശാലമാക്കുകയും അതില്‍ ഉറച്ചുനില്‍ക്കുന്ന പര്‍വ്വതങ്ങള്‍ സ്ഥാപിക്കുകയും, അളവ് നിര്‍ണയിക്കപ്പെട്ട എല്ലാ വസ്തുക്കളും അതില്‍ നാം മുളപ്പിക്കുകയും ചെയ്തിരിക്കുന്നു.'' (15:19)

''നിങ്ങള്‍ക്ക് വേണ്ടി ഭൂമിയെ തൊട്ടിലാക്കുകയും, നിങ്ങള്‍ക്ക് അതില്‍ വഴികള്‍ ഏര്‍പെടുത്തിത്തരികയും, ആകാശത്ത് നിന്ന് വെള്ളം ഇറക്കിത്തരികയും ചെയ്തവനത്രെ അവന്‍. അങ്ങനെ അത് (വെള്ളം) മൂലം വ്യത്യസ്ത തരത്തിലുള്ള സസ്യങ്ങളുടെ ജോടികള്‍ നാം (അല്ലാഹു) ഉല്‍പാദിപ്പിക്കുകയും ചെയ്തിരിക്കുന്നു.'' (20:53)

''അഥവാ, ഭൂമിയെ നിവാസയോഗ്യമാക്കുകയും, അതിനിടയില്‍ നദികളുണ്ടാക്കുകയും, അതിന് ഉറപ്പ് നല്‍കുന്ന പര്‍വ്വതങ്ങള്‍ ഉണ്ടാക്കുകയും, രണ്ടുതരം ജലാശയങ്ങള്‍ക്കിടയില്‍ ഒരു തടസ്സം ഉണ്ടാക്കുകയും ചെയ്തവനോ? (അതോ അവരുടെ ദൈവങ്ങളോ?) അല്ലാഹുവോടൊപ്പം മറ്റു വല്ല ദൈവവുമുണ്ടോ? അല്ല, അവരില്‍ അധികപേരും അറിയുന്നില്ല.'' (27:61)

''അവനാകുന്നു നിങ്ങള്‍ക്ക് വേണ്ടി ഭൂമിയെ വിധേയമാക്കി തന്നവന്‍. അതിനാല്‍ അതിന്റെ ചുമലുകളിലൂടെ നിങ്ങള്‍ നടക്കുകയും അവന്റെ ഉപജീവനത്തില്‍ നിന്ന് ഭക്ഷിക്കുകയും ചെയ്തു കൊള്ളുക. അവങ്കലേക്ക് തന്നെയാണ് ഉയിര്‍ത്തെഴുന്നേല്‍പ്.'' (67:15)

''അല്ലാഹു നിങ്ങള്‍ക്കു വേണ്ടി ഭൂമിയെ ഒരു വിരിപ്പാക്കുകയും ചെയ്തിരിക്കുന്നു. അതിലെ വിസ്താരമുള്ള പാതകളില്‍ നിങ്ങള്‍ പ്രവേശിക്കുവാന്‍ വേണ്ടി.'' (71:19,20)

''ഭൂമിയാകട്ടെ, നാം അതിനെ ഒരു വിരിപ്പാക്കിയിരിക്കുന്നു. എന്നാല്‍ അത് വിതാനിച്ചവന്‍ എത്ര നല്ലവന്‍!'' (51:48)

ല്ല. ആർത്തവരക്തത്തിന് കുഞ്ഞിന്റെ രൂപീകരണത്തിൽ എന്തെങ്കിലും പങ്കുള്ളതായി ഖുർആൻ പഠിപ്പിക്കുന്നില്ല. ആര്‍ത്തവരക്തത്തെക്കുറിച്ച് ക്വുര്‍ആന്‍ പരാമര്‍ശിക്കുന്നത് രണ്ടു തവണയാണ്.

അവ ഇങ്ങനെയാണ്:

''ആര്‍ത്തവത്തെപ്പറ്റി അവര്‍ നിന്നോട് ചോദിക്കുന്നു. പറയുക; അതൊരു മാലിന്യമാകുന്നു. അതിനാല്‍ ആര്‍ത്തവഘട്ടത്തില്‍ നിങ്ങള്‍ സ്ത്രീകളില്‍ നിന്ന് അകന്നു നില്‍ക്കേണ്ടതാണ്. അവര്‍ ശുദ്ധിയാകുന്നത് വരെ അവരെ സമീപിക്കുവാന്‍ പാടില്ല. എന്നാല്‍ അവര്‍ ശുചീകരിച്ചു കഴിഞ്ഞാല്‍ അല്ലാഹു നിങ്ങളോട് കല്‍പിച്ച വിധത്തില്‍ നിങ്ങള്‍ അവരുടെ അടുത്ത് ചെന്നുകൊള്ളുക. തീര്‍ച്ചയായും അല്ലാഹു പശ്ചാതപിക്കുന്നവരെ ഇഷ്ടപ്പെടുന്നു. ശുചിത്വം പാലിക്കുന്നവരെയും ഇഷ്ടപ്പെടുന്നു.''(ക്വുര്‍ആന്‍ 2:222)

''നിങ്ങളുടെ സ്ത്രീകളില്‍ നിന്നും ആര്‍ത്തവത്തെ സംബന്ധിച്ച് നിരാശപ്പെട്ടിട്ടുള്ളവരെ സംബന്ധിച്ചിടത്തോളം നിങ്ങള്‍ അവരുടെ ഇദ്ദയുടെ കാര്യത്തില്‍ സംശയത്തിലാണെങ്കില്‍ അത് മൂന്ന് മാസമാകുന്നു. ആര്‍ത്തവമുണ്ടായിട്ടില്ലാത്തവരുടേതും അങ്ങനെ തന്നെ. ഗര്‍ഭവതികളായ സ്ത്രീകളാകട്ടെ, അവരുടെ അവധി അവര്‍ തങ്ങളുടെ ഗര്‍ഭം പ്രസവിക്കലാകുന്നു. വല്ലവനും അല്ലാഹുവെ സൂക്ഷിക്കുന്ന പക്ഷം അവന്ന് അവന്റെ കാര്യത്തില്‍ അല്ലാഹു എളുപ്പമുണ്ടാക്കികൊടുക്കുന്നതാണ്.''(ക്വുര്‍ആന്‍ 65:4)

ആര്‍ത്തവത്തെക്കുറിച്ച സംശയത്തിന് മറുപടി പറയുമ്പോള്‍ സൂറത്തുല്‍ ബക്വറയിലെ സൂക്തത്തില്‍ അതൊരു മാലിന്യമാണെന്നും അത് പുറപ്പെടുന്ന സന്ദര്‍ഭത്തില്‍ സ്ത്രീകളുമായി ശാരീരികബന്ധം പാടില്ലെന്നും മാത്രമാണ് പറയുന്നതെന്ന കാര്യം ശ്രദ്ധേയമാണ്. ഇവിടെ കുഞ്ഞിന്റെ രൂപീകരണവുമായി അതിന് ഏതെങ്കിലും തരത്തിലുള്ള ബന്ധമുണ്ടെന്ന് സൂചിപ്പിക്കുന്ന യാതൊരു പരാമര്‍ശവുമില്ല. സൂറത്തുത്ത്വലാക്വിലെ വചനമാകട്ടെ, ആര്‍ത്തവവിരാമക്കാരുടെയും ആര്‍ത്തവമുണ്ടായിട്ടില്ലാത്തവരുടെയും ഇദ്ദ കാലത്തെക്കുറിച്ചുള്ളതാണ്. അവിടെയും ഗര്‍ഭധാരണത്തെയോ കുഞ്ഞിന്റെ രൂപീകരണത്തെയോ കുറിക്കുന്ന യാതൊന്നും തന്നെ പറഞ്ഞിട്ടില്ല. ആര്‍ത്തവകാലത്തെയും ആര്‍ത്തവരക്തത്തെയും കുറിച്ച നിരവധി പരാമര്‍ശങ്ങള്‍ ഹദീഥുകളിലുണ്ട്. സ്വഹീഹുല്‍ ബുഖാരിയിലെ ആറാമത്തെ അധ്യായവും സ്വഹീഹു മുസ്്‌ലിമിലെ മൂന്നാം അധ്യായവും 'കിതാബുല്‍ ഹൈദ്വ്' അഥവാ ആര്‍ത്തവത്തെക്കുറിച്ച അധ്യായങ്ങളാണ്. ബുഖാരി 37 ഹദീഥുകളും മുസ്്‌ലിം 158 ഹദീഥുകളും ഈ അധ്യായത്തില്‍ നല്‍കിയിട്ടുണ്ട്. ഇവയില്‍ മിക്കതും കര്‍മശാസ്ത്ര സംബന്ധിയായ വിഷയങ്ങളാണ് കൈകാര്യം ചെയ്യുന്നത്. സുനനുന്നസാഇയിലെ മൂന്നാം അധ്യായമായ 'കിതാബുല്‍ ഹൈദ്വു വല്‍ ഇസ്തിഹാദ്വ', സുനനു അബൂദാവൂദിലെ ഒന്നാം അധ്യായമായ 'കിതാബുത്ത്വഹാറ', ജാമിഉത്തിര്‍മിദിയിലെ ഒന്നാം അധ്യായമായ 'കിതാബുത്ത്വഹാറത്തു അന്‍ റസൂലുല്ലാഹി സ്വല്ലല്ലാഹു അലൈഹിവസല്ലം', സുനനു ഇബ്‌നുമാജയിലെ ഒന്നാം അധ്യായമായ 'കിതാബുത്ത്വഹാറത്തു വസുനനുഹാ', മുവത്വാ മാലിക്കിലെ രണ്ടാം അധ്യായമായ 'കിതാബുത്ത്വഹാറ' എന്നിവയില്‍ ഉദ്ധരിച്ചിരിക്കുന്ന ആര്‍ത്തവ സംബന്ധിയായ ഹദീഥുകളിലും പ്രധാനമായി പരാമര്‍ശിച്ചിരിക്കുന്നത് കര്‍മപരമായ കാര്യങ്ങളെക്കുറിച്ചാണ്. ആര്‍ത്തവരക്തത്തെക്കുറിച്ചുള്ള നൂറിലധികം വരുന്ന ഹദീഥുകള്‍ക്കിടയിലെവിടെയും അതിന് കുഞ്ഞിന്റെ രൂപീകരണത്തില്‍ എന്തെങ്കിലും വിധത്തിലുള്ള പങ്കുണ്ടെന്ന് സൂചിപ്പിക്കുന്ന ഒരു പരാമര്‍ശം പോലുമില്ല.

വിഷയവുമായി ബന്ധപ്പെട്ട വീഡിയോ
 

കകോശജീവിയിൽ നിന്ന് പ്രകൃതിനിർധാരണത്തിലൂടെ മാത്രമായി പരിണമിച്ചാണ് ജീവികളെല്ലാം ഉണ്ടായത് എന്നും ജൈവപ്രപഞ്ചത്തിന്റെ നിലനിൽപിന് പിന്നിൽ പ്രകൃത്യാതീതമായ യാതൊന്നുമില്ലെന്നും പറയുന്നതാണ് പരിണാമവാദമെങ്കിൽ അതിനെ ഖുർആൻ അംഗീകരിക്കുന്നില്ല. ജീവികൾക്കിടയിൽ മാറ്റങ്ങളൊന്നുമുണ്ടായിട്ടില്ലെന്നാണ് ഖുർആൻ പറയുന്നത് എന്നല്ല ഇതിനർത്ഥം. ജീവജാതികൾക്കകത്ത് നിരവധി മാറ്റങ്ങളുണ്ടായിട്ടുണ്ടായിരിക്കാം; പുതിയ ജീവജാതികളുടെ ഉല്പത്തിക്ക് വരെ പ്രസ്തുത മാറ്റങ്ങൾ കാരണമായിരുന്നിരിക്കാം. ഇതൊന്നും തന്നെ ഖുർആൻ നിഷേധിക്കുന്നില്ല. എന്നാൽ അന്ധമായ പ്രകൃതിനിര്ധാരണം വഴി മാത്രമാണ് പ്രസ്തുത മാറ്റങ്ങളുണ്ടാവുന്നതെന്ന വാദത്തെ ക്വുർആൻ നിരാകരിക്കുന്നു.ജീവവര്ഗങ്ങളുടെ സൃഷ്ടിക്കും നിലനിൽപ്പിനും പിന്നിൽ വ്യക്തമായ ആസൂത്രണമുണ്ടെന്നാണ് ക്വുർആനും ഹദീഥുകളും പഠിപ്പിക്കുന്നത്. "എല്ലാ ജന്തുക്കളെയും അല്ലാഹു വെള്ളത്തില്‍ നിന്ന്‌ സൃഷ്ടിച്ചിരിക്കുന്നു. അവരുടെ കൂട്ടത്തില്‍ ഉദരത്തില്‍മേല്‍ ഇഴഞ്ഞ്‌ നടക്കുന്നവരുണ്ട്‌. രണ്ട്‌ കാലില്‍ നടക്കുന്നവരും അവരിലുണ്ട്‌. നാലുകാലില്‍ നടക്കുന്നവരും അവരിലുണ്ട്‌. അല്ലാഹു താന്‍ ഉദ്ദേശിക്കുന്നത്‌ സൃഷ്ടിക്കുന്നു. തീര്‍ച്ചയായും അല്ലാഹു എല്ലാകാര്യത്തിനും കഴിവുള്ളവനാകുന്നു." (ക്വുർആൻ 24 :45)

ജൈവവൈവിധ്യത്തെ വിശദീകരിക്കുവാനുള്ള സര്‍ഗാത്മകവും സുന്ദരവുമായ ഒരു പരിശ്രമമാണ് പരിണാമസിദ്ധാന്തം. ചാൾസ് ഡാർവിൻ 1859 ൽ എഴുതിയ ദി ഒറിജിൻ ഓഫ് സ്പെസിസ് എന്ന ഗ്രന്ഥത്തിലൂടെയാണ് പ്രകൃതി നിർധാരണത്തിലൂടെയുള്ള ജീവജാതികളുടെ പരിണാമമെന്ന സിദ്ധാന്തം ആദ്യമായി മുന്നോട്ടു വെക്കപ്പെടുന്നത്. പിന്നീട് നിയോഡാർവിനിസ്റ്റുകൾ ജനിതകത്തിന്റെ വെളിച്ചത്തിൽ പരിണാമത്തെ വിശദീകരിക്കാൻ ശ്രമിച്ചു. പ്രസ്തുത ശ്രമം ഇന്നും തുടർന്നുകൊണ്ടിരിക്കുന്നു. ജീവികളുടെ വൈവിധ്യത്തെ വിശദീകരിക്കാൻ ശ്രമിക്കുന്ന ഒരു സങ്കല്പം എന്നതിലുപരിയായി ഒരു ശാസ്ത്രസിദ്ധാന്തത്തിന്റെ രീതിയോ രൂപമോ നല്‍കാനാവുന്ന തത്ത്വമല്ല ജീവപരിണാമമെന്ന വസ്തുത ശക്തരായ പരിണാമവാദികള്‍ക്കുപോലും ബോധ്യമുള്ളതാണ്. പരിണാമവാദം ഒരു കേവല സാങ്കല്‍പിക തത്ത്വം മാത്രമാണെന്നും ഊഹങ്ങളല്ലാതെ ഒരു ശാസ്ത്രസിദ്ധാന്തത്തിനുണ്ടാവേണ്ട വസ്തുനിഷ്ഠ തെളിവുകളുടെ പിന്‍ബലം അതിനില്ലെന്നും പറയുന്നത് ഇവ്വിഷയകമായി ഇതഃപര്യന്തം നടന്ന ഗവേഷണങ്ങളെയൊന്നും അവഗണിച്ചുകൊണ്ടല്ല; പ്രത്യുത, പ്രസ്തുത ഗവേഷണങ്ങളൊന്നും തന്നെ ഉദ്ദേശിച്ച  ഫലം നല്‍കിയിട്ടില്ലെന്ന അതിന്റെ വക്താക്കളുടെ തന്നെ വെളിപ്പെടുത്തലുകളുടെ വെളിച്ചത്തിലാണ്. പരിണാമവാദത്തിനെതിരെ ശാസ്ത്രലോകത്തുനിന്ന് ഉന്നയിക്കപ്പെടുന്ന വിമര്‍ശനങ്ങള്‍ താഴെ പറയുന്നവയാണ്:

1. നാം ജീവിക്കുന്നത് ലക്ഷക്കണക്കിന് ജീവിവര്‍ഗങ്ങള്‍ക്കിടയിലാണ്. ഇവയിലൊന്നും തന്നെ മറ്റൊരു ജീവിവര്‍ഗമായി പരിണമിക്കുന്നത് നാം കാണുന്നില്ല. രേഖപ്പെട്ടിടത്തോളമുള്ള ചരിത്രത്തിലെവിടെയും ആരെങ്കിലും അത്തരമൊരു പരിണാമം നിരീക്ഷിച്ചതായി രേഖപ്പെടുത്തിയിട്ടില്ല.

2. ലഘുജീവികളില്‍ നിന്ന് സങ്കീര്‍ണമായവ പരിണമിച്ചുണ്ടായിയെന്ന തന്റെ വാദത്തെ സത്യപ്പെടുത്തുന്ന തെളിവുകള്‍ പുരാവസ്തുപഠനങ്ങള്‍ നല്‍കുമെന്ന ചാള്‍സ് ഡാര്‍വിന്റെ പ്രതീക്ഷയുടെ പൂര്‍ത്തീകരണത്തിനുവേണ്ടി ഒന്നരനൂറ്റാണ്ടുകള്‍ നീണ്ട പരിശ്രമങ്ങളൊന്നും തന്നെ ഫലം കണ്ടിട്ടില്ല. ഭൂമിയില്‍ നിലനില്‍ക്കുന്നതായി കണക്കാക്കപ്പെടുന്ന 87 ലക്ഷം ജീവിവര്‍ഗങ്ങളില്‍ ഏതെങ്കിലും രണ്ട് ജീവി വര്‍ഗങ്ങളെ കൂട്ടിയോജിപ്പിക്കുന്ന മുറിഞ്ഞ കണ്ണിയെ കണ്ടെത്തുവാന്‍ ഫോസില്‍ പഠനങ്ങള്‍ക്ക് കഴിഞ്ഞിട്ടില്ല.

3. ഫോസില്‍ പഠനങ്ങള്‍ വ്യക്തമാക്കുന്നത് വ്യത്യസ്ത ജീവവര്‍ഗങ്ങള്‍ക്കിടയില്‍ പ്രകടമായ വ്യത്യാസമുണ്ടെന്നും പ്രസ്തുത വ്യത്യാസം എന്നെന്നും നിലനിന്നിരുന്നുവെന്നും തന്നെയാണ്. ഡാര്‍വിന്റെ പ്രകൃതിനിര്‍ധാരണതത്ത്വവും നിയോഡാര്‍വിനിസ്റ്റുകളുടെ ഉല്‍പരിവര്‍ത്തന സിദ്ധാന്തവുമുപയോഗിച്ച് ജീവവര്‍ഗങ്ങള്‍ തമ്മില്‍ നിലനില്‍ക്കുന്ന വിടവ് വിശദീകരിക്കാനാവാത്തതുകൊണ്ടാണ് ‘വിരാമമിടുന്ന സന്തുലിതാവസ്ഥ’ (Punctuated equilibriium) എന്ന, ഓരോ ജീവിവര്‍ഗവും മറ്റേ ജീവിവര്‍ഗത്തില്‍ നിന്ന് പെട്ടെന്ന് പരിണമിക്കുകയായിരുന്നുവെന്ന സിദ്ധാന്തത്തില്‍ ഇക്കാലത്തെ പരിണാമവാദികള്‍ക്ക് അഭയം തേടേണ്ടിവരുന്നത്. ഓരോ ജീവവര്‍ഗവും പ്രത്യേകമായി സൃഷ്ടിക്കപ്പെടുകയാണുണ്ടായത് എന്നു തന്നെയാണ് ഇത് വ്യക്തമാക്കുന്നത്.

4. ജീവന്റെ ഉല്‍പത്തിയെക്കുറിച്ച് വിശദീകരിക്കുകയെന്ന, ജൈവലോകത്ത് നടന്നുവെന്ന് സങ്കല്‍പിക്കപ്പെട്ട പരിണാമത്തെക്കുറിച്ച് പറയുന്ന ഒരു സിദ്ധാന്തത്തിന്റെ ബാധ്യത നിര്‍വഹിക്കുവാന്‍ പരിണാമവാദത്തിന് ഇതുവരെ കഴിഞ്ഞിട്ടില്ല. ഭൂമിയില്‍ നിലനിന്ന ഏതെങ്കിലുമൊരു സാഹചര്യത്തില്‍ ജീവന്‍ യാദൃച്ഛികമായി ഉണ്ടാകുവാന്‍ യാതൊരു സാധ്യതയുമില്ലെന്ന വസ്തുത പരിണാമവാദികള്‍ തന്നെ സമ്മതിക്കുന്നുണ്ട്.(18) നക്ഷത്രാന്തരപടലത്തിലെവിടെയോ നിലനിന്നിരുന്ന ഒരു പ്രത്യേക സാഹചര്യത്തില്‍ അവിടെ രൂപം കൊള്ളുകയും പിന്നീട് ഭൂമിയോട് അടുത്തു വന്ന ഏതോ ധൂമകേതു വഴി ഭൂമിയിലെത്തുകയും ചെയ്ത പ്രതിഭാസമാണ് ജീവനെന്ന വിശദീകരണത്തില്‍ അഭയം തേടുകയാണ് പരിണാമവാദികള്‍ ഇപ്പോള്‍ ചെയ്യുന്നത്. (Sir Fred Hoyle& Chandra Wickramasinghe: Evolution from Space, New York, 1984)

5. പരിണാമത്തിന് അനുകൂലമായി അവതരിപ്പിക്കപ്പെട്ടിരുന്ന തെളിവുകളൊന്നും തന്നെ ജീവപരിണാമത്തെ സാധൂകരിക്കുന്നില്ലെന്ന വസ്തുത വ്യക്തമാക്കുകയാണ് പിന്നീടുള്ള ഗവേഷണങ്ങള്‍ ചെയതത്. സസ്തനികളുടെ ഭ്രൂണഘട്ടങ്ങളിലെ സാദൃശ്യം കെട്ടിച്ചമയ്ക്കപ്പെട്ടതാണെന്ന് ഈ രംഗത്തെ ആദ്യകാല പരിശ്രമങ്ങളിലൊന്നായ ഏണസ്റ്റ് ഹെയ്ക്കലിന്റെ ഭ്രൂണപരിണാമഘട്ടങ്ങളുടെ താരതമ്യചിത്രത്തെ പഠനവിധേയമാക്കിയ പരിണാമവാദികള്‍ തന്നെ വ്യക്തമാക്കിയിട്ടുള്ളതാണ്.(Michael K. Richardson& Gerhard Keuck: Haeckel’s ABC of evolution and development, ýBiological Reviews, 2002, No: 77, Pages 495–528)

വ്യത്യസ്ത ജീവികളുടെ ജനിതകഘടനയുടെ താരതമ്യം വഴിയുള്ള പരിണാമവൃക്ഷത്തിന്റെ നിര്‍മാണമെന്ന ആശയവും പരിഹരിക്കാനാവാത്ത പ്രശ്‌നങ്ങളാല്‍ വഴിമുട്ടി നില്‍ക്കുകയാണ്. മനുഷ്യനും ആള്‍കുരങ്ങും തമ്മിലുള്ള ജനിതകസാദൃശ്യം 96 ശതമാനത്തോളം വരുമെന്നും അതിനാല്‍ ആള്‍ക്കുരങ്ങില്‍ നിന്നാണ് മനുഷ്യനുണ്ടായതെന്നും വാദിക്കുകയാണെങ്കില്‍ പശുവിനോട് കുതിരയെക്കാള്‍ ബന്ധം ഡോള്‍ഫിനാണെന്നുകൂടി വാദിക്കേണ്ടിവരുമെന്നും ഇതേപോലെയുള്ള കാരണങ്ങളാല്‍ ജനിതകവസ്തുവിന്റെ സാദൃശ്യത്തിന്റെ വെളിച്ചത്തില്‍ മാത്രമായി പരിണാമത്തെ സമര്‍ത്ഥിക്കാനാവില്ലെന്നും വാദിക്കുന്ന വലിയൊരു വിഭാഗം ജീവശാത്രജ്ഞന്മാരുണ്ട്.(Michael Denton:  Evolution: Theory in Crisis, London, 1985. Pages 233-249)

ജനിതകരേഖകള്‍ക്കിടയില്‍ കാണപ്പെടുന്ന, ധര്‍മങ്ങളൊന്നും ഇല്ലാത്തതായി  കരുതിയിരുന്ന ചവറ് ഡി.എന്‍.എകള്‍ (Junk DNA) പരിണാമത്തിനുള്ള തെളിവുകളായി കരുതിയത് തെറ്റാണെന്ന് അവ ചവറുകളല്ലെന്നും അവയ്ക്ക് ധര്‍മങ്ങളുണ്ടെന്നും മനസ്സിലായതോടെ ജീവശാസ്ത്രലോകത്തിന് ബോധ്യപ്പെട്ടു. (Alice Park: Junk DNA — Not So Useless After All, Time Magazine, Sept. 06, 2012) പരിണാമത്തിന് അനുകൂലമായി വ്യാഖ്യാനിക്കപ്പെട്ടിരുന്ന തെളിവുകള്‍ യഥാര്‍ത്ഥ തെളിവുകളല്ലെന്ന് ബോധ്യപ്പെട്ടുകൊണ്ടിരിക്കുന്നത് പരിണാമവാദത്തിന്റെ വിശ്വാസ്യത തകര്‍ക്കുന്നതാണെന്നതില്‍ സംശയമില്ല.

6. ജീവപരിണാമമെന്ന ആശയം പ്രകൃതി നിയമങ്ങള്‍ക്ക് വിരുദ്ധമാണെന്നതിനാല്‍ അശാസ്ത്രീയമാണ്. ‘ഒരു വ്യവസ്ഥയുടെ എന്‍ട്രോപ്പി ഒരിക്കലും കുറയുകയില്ല, അത്തരം വ്യവസ്ഥകള്‍ പരമാവധി എന്‍ട്രോപ്പിയിലെത്തി താപഗതിക സന്തുലിതത്വം പാലിക്കുവാനാണ് സദാ ശ്രമിച്ചുകൊണ്ടിരിക്കുകയെന്ന രണ്ടാം താപഗതിക നിയമത്തിന് എതിരാണ് ജീവപരിണാമം എന്ന ആശയം. ഒറ്റപ്പെട്ട ഒരു വ്യവസ്ഥയില്‍ സ്വാഭാവികമായും സങ്കീര്‍ണവസ്തുകള്‍ വിഘടിച്ച് ലഘുവായിത്തീരുകയാണ് ചെയ്യുകയെന്നാണ് പരമാവധി എന്‍ട്രോപ്പിയിലെത്താനാണ് വ്യവസ്ഥ പരിശ്രമിക്കുകയെന്ന് പറഞ്ഞാല്‍ അതിനര്‍ത്ഥം. ജീവപരിണാമം എന്ന ആശയം തന്നെ ലഘു ജീവവസ്തുകളില്‍ നിന്ന് സങ്കീര്‍ണ ജീവജാലങ്ങളിലേക്കുള്ള സ്വാഭാവികപരിവര്‍ത്തനത്തെയാണല്ലോ കുറിക്കുന്നത്. അങ്ങനെ സംഭവിക്കുവാന്‍ ഒരു ബാഹ്യ ഇടപെടലില്ലാതെ   യാതൊരു സാധ്യതയുമില്ലെന്നാണ് രണ്ടാം താപഗതിക നിയമം അര്‍ഥശങ്കയ്ക്കിടയില്ലാത്തവണ്ണം വ്യക്തമാക്കുന്നത്. സൂക്ഷ്മജീവികളില്‍ നിന്ന് സങ്കീര്‍ണജീവികളുണ്ടാവുകയെന്ന പരിണാമം സംഭവിച്ചുവെന്നതിന് സമൃദ്ധമായ മറ്റു തെളിവുകളുണ്ടെങ്കില്‍ പോലും അങ്ങനെ സംഭവിക്കണമെങ്കില്‍ ഒരു ബാഹ്യശക്തിയുടെ ഇടപെടലുണ്ടായിട്ടുണ്ടെന്ന് സമ്മതിക്കേണ്ടിവരും.

പരിണാമവാദം ശാസ്ത്രീയമായി സ്ഥിരീകരിക്കപ്പെടാത്ത ഒരു സിദ്ധാന്തമായി മാത്രം നില നിൽക്കുന്നിടത്തോളം അതേക്കുറിച്ച് ഇസ്ലാമികനിലപാടെന്ത് എന്ന ചോദ്യം അപ്രസക്തമാണ്. മനുഷ്യൻ പടച്ചവന്റെ ഒരു സവിശേഷസൃഷ്ടിയാണെന്ന് ക്വുർആൻ അസന്നിഗ്ധമായി പ്രഖ്യാപിക്കുന്നു. ജീവികൾക്കിടയിൽ നടന്നേക്കാവുന്ന മാറ്റങ്ങളെ ഇസ്‌ലാം നിഷേധിക്കുന്നില്ല. പക്ഷെ, പ്രസ്തുത മാറ്റങ്ങൾക്കു പിന്നിൽ അന്ധമായ യാദൃച്ഛികതയാണെന്ന വാദം ഇസ്‌ലാം നിരാകരിക്കുന്നു. മാറ്റങ്ങളുണ്ടായിട്ടുണ്ടെങ്കിൽ അവയടക്കം അല്ലാഹുവിന്റെ വിശാലമായ ആസൂത്രണത്തിന്റെ ഭാഗമായി നടന്നതായിരിക്കുമെന്നാണ് ഇസ്‌ലാമിന്റെ വീക്ഷണം.

വിഷയവുമായി ബന്ധപ്പെട്ട വീഡിയോ

ബീജസങ്കലനമെന്ന പദം ഖുർആൻ പ്രയോഗിച്ചിട്ടില്ലെന്നത് ശരിയാണ്. എന്നാൽ, സ്ത്രീ-പുരുഷ ബീജങ്ങളുടെ സംയോജനത്തിൽ നിന്നാണ് കുഞ്ഞുണ്ടാവുന്നതെന്ന വസ്തുതയിലേക്ക് ക്വുർആൻ വെളിച്ചം വീശിയിട്ടുണ്ട്. ഇവ്വിഷയകമായ ഖുർആൻ പരാമർശങ്ങളുടെ കൃത്യതയും സൂക്ഷ്മതയുമറിയണമെങ്കിൽ അക്കാലത്ത് നിലവിലുണ്ടായിരുന്ന സങ്കൽപ്പങ്ങൾ എന്തൊക്കെയായിരുന്നുവെന്ന് മനസ്സിലാക്കണം. പുരുഷ ശുക്ലവും ആര്‍ത്തവരക്തവും ചേര്‍ന്നാണ് കുഞ്ഞുണ്ടാവുന്നതെന്ന് കരുതിയ പിപ്പിലാദ ഋഷി മുതല്‍(1) പാലില്‍നിന്ന് തൈരുണ്ടാവുന്നതുപോലെ ശുക്ലദ്രാവകം ഘനീഭവിച്ചാണ് ശിശുനിര്‍മിതി നടക്കുന്നതെന്ന് വിചാരിച്ച ബൈബിളിലെ  ഇയ്യോബ്(2) പുസ്തകത്തിന്റെ കര്‍ത്താവ് വരെയുള്ളവരുടെ വീക്ഷണങ്ങള്‍ വ്യത്യസ്ത അറ്റങ്ങളിലുള്ളവയായിരുന്നു. പുരുഷന്റെയും സ്ത്രീയുടെയും ശുക്ലങ്ങളിലുള്ള ബീജങ്ങള്‍ കൂടിച്ചേര്‍ന്നാണ് കുഞ്ഞുണ്ടാവുന്നതെന്ന് കരുതിയ ഹിപ്പോക്രാറ്റസ്,(3) മാതൃരക്തത്തെ പുരുഷശുക്ലം ഘനീഭവിപ്പിച്ചാണ് ശിശുവുണ്ടാകുന്നതെന്ന് കരുതിയ അരിസ്റ്റോട്ടില്‍,(4) ശുക്ലത്തെ മാതൃരക്തം പരിപോഷിപ്പിക്കുമ്പോഴാണ് അതിന്റെ നിര്‍മിതി നടക്കുന്നതെന്ന് വിചാരിച്ച ഗാലന്‍(5) എന്നിവരുടെ വീക്ഷണങ്ങള്‍ പാശ്ചാത്യന്‍ വൈജ്ഞാനിക മണ്ഡലത്തില്‍ സജീവമായിരുന്ന കാലത്താണ് ക്വുര്‍ആന്‍ അവതരിക്കുന്നത്.

“നുത്വ്ഫ (ബീജം) യില്‍ നിന്നാണ് കുഞ്ഞുണ്ടാവുന്നതെന്ന് പറഞ്ഞതോടൊപ്പം തന്നെ കൂടിച്ചേര്‍ന്നുണ്ടായ നുത്വ്ഫയാണ് ശിശുനിര്‍മിതിക്ക് നിമിത്തമാവുന്നതെന്നു കൂടി ക്വുര്‍ആന്‍ വ്യക്തമാക്കുന്നുണ്ട്. ക്വുര്‍ആന്‍ പറയുന്നത് നോക്കുക: ”നുത്വ്ഫതുന്‍ അംശാജിൽ (കൂട്ടിച്ചെർന്നുണ്ടായ ബീജം) നിന്ന് തീർച്ചയായും നാം മനുഷ്യനെ സൃഷ്ടിച്ചിരിക്കുന്നു; നമുക്ക് അവനെ പരീക്ഷിക്കുവാന്‍. അങ്ങനെ നാം അവനെ കേള്‍ക്കുന്നവനും കാണുന്നവനുമാക്കിയിരിക്കുന്നു.’‘(6)

മനുഷ്യസൃഷ്ടി നടന്നത് ‘നുത്വ്ഫതുന്‍ അംശാജി’ല്‍ നിന്നാണെന്നാണ് ഈ വചനത്തില്‍ വ്യക്തമാക്കിയിരിക്കുന്നത്. മീം, ശീന്‍, ജീം അക്ഷരത്രയത്തില്‍നിന്ന് നിഷ്പന്നമായ മാശിജിന്റെ ബഹുവചനമാണ് അംശാജ്. കൂട്ടിച്ചേര്‍ക്കുക, ആശയക്കുഴപ്പത്തിലാക്കുക, ഒന്നിനെ മറ്റൊന്നുമായി ഒന്നിച്ചുചേര്‍ക്കുക എന്നീ അര്‍ത്ഥങ്ങളിലാണ് ഈ അക്ഷരത്രയം ഉപയോഗിക്കാറുള്ളത്.(7) ‘നുത്വ്ഫതുന്‍ അംശാജുന്‍’ എന്നാല്‍ കൂടിച്ചേര്‍ന്നുണ്ടായ നുത്വ്ഫയെന്നാണ് അര്‍ത്ഥമെന്ന് ഇത് വ്യക്തമാക്കുന്നു. ക്വുര്‍ആനില്‍ ഈ വചനത്തിലല്ലാതെ മറ്റൊരിടത്തും ഈ പദം പ്രയോഗിച്ചിട്ടില്ല. പുരുഷബീജവും അണ്ഡവും ചേര്‍ന്ന സിക്താണ്ഡത്തെ കുറിക്കാനാണ് ക്വുര്‍ആന്‍ ഇങ്ങനെ പ്രയോഗിച്ചതെന്നാണ് മനസ്സിലാവുന്നത്.

പുരുഷ-സ്ത്രീ സ്രവങ്ങളുടെ സംയോജനത്തില്‍നിന്നാണ് കുഞ്ഞുണ്ടാവുന്നതെന്ന വസ്തുത പ്രവാചകന്‍(സ) വ്യക്തമാക്കിയിട്ടുണ്ട്. ശിശുനിര്‍മിതിയെക്കുറിച്ച ജൂത ചോദ്യത്തിനുള്ള പ്രവാചക മറുപടിയില്‍ ”പുരുഷസ്രവം വെളുത്തതും സ്ത്രീസ്രവം മഞ്ഞയുമാണ്; അവ രണ്ടും കൂടിച്ചേര്‍ന്നാല്‍…” എന്നു കാണാം.(8) പുരുഷന്റെ നുത്വ്ഫയും സ്ത്രീയുടെ നുത്വ്ഫയും കൂടിച്ചേര്‍ന്നുണ്ടാവുന്ന “നുത്വ്ഫയെക്കുറിച്ചാണ് ക്വുര്‍ആനില്‍ ‘നുത്വ്ഫതിന്‍ അംശാജിന്‍’’എന്ന് പറഞ്ഞിരിക്കുന്നതെന്ന് ഇതില്‍നിന്ന് വ്യക്തമാണ്. പ്രത്യുല്‍പാദനത്തെയും കുഞ്ഞിന്റെ ലിംഗനിര്‍ണയം, വിധി എന്നിവയെയുമെല്ലാം കുറിച്ച് പ്രതിപാദിക്കുന്ന ഹദീഥുകളിലും സ്ത്രീ-പുരുഷ സ്രവങ്ങളുടെ സംയോജനത്തെക്കുറിച്ച പരാമര്‍ശങ്ങള്‍ കാണാം. (9)

പ്രവാചകനില്‍നിന്ന് മതം പഠിച്ച സ്വഹാബിമാര്‍ സ്ത്രീ-പുരുഷ സ്രവങ്ങളുടെ സംയോജനമാണ് “നുത്വ്ഫതിന്‍ അംശാജിന്‍’ എന്നതുകൊണ്ട് മനസ്സിലാക്കിയതെന്ന് ക്വുര്‍ആന്‍ വ്യാഖ്യാന ഗ്രന്ഥങ്ങള്‍ വ്യക്തമാക്കുന്നുണ്ട്. ”പുരുഷസ്രവവും സ്ത്രീസ്രവവും; അവ യോജിക്കുമ്പോള്‍”’ എന്നാണ് ഇബ്‌നുഅബ്ബാസ്(റ) ഈ വചനത്തെ വ്യാഖ്യാനിച്ചതെന്ന് ഇമാം ത്വബ്‌രി തന്റെ ജാമിഉല്‍ ബയാന്‍ ഫീ തഫ്‌സീറില്‍ ക്വുര്‍ആനില്‍ സമര്‍ത്ഥിക്കുന്നു.(10) ഇക്‌രിമ(റ)യാകട്ടെ, “”പുരുഷസ്രവവും സ്ത്രീസ്രവവും; അതിലൊന്ന് മറ്റേതുമായി കൂടിച്ചേരുമ്പോള്‍” എന്നാണ് ഈ വചനത്തെ വ്യാഖ്യാനിക്കുന്നത്. റബീഉബ്‌നു അനസ് (റ), ഹസന്‍(റ), മുജാഹിദ്(റ) എന്നിവരും ഇതേപോലെ തന്നെയാണ് ഈ വചനത്തെ വ്യാഖ്യാനിച്ചതെന്ന് ഇമാം ത്വബ്‌രി(റ) വിശദീകരിക്കുന്നുണ്ട്. സ്ത്രീയുടെയും പുരുഷന്റെയും നുത്വ്ഫകളുടെ സംയോജനത്തില്‍നിന്നാണ് കുഞ്ഞുണ്ടാകുന്നതെന്നായിരുന്നു സ്വഹാബിമാരും താബിഉകളുമെല്ലാം മനസ്സിലാക്കിയതെന്ന് ഇമാം റാസി(റ) തന്റെ ക്വുര്‍ആന്‍ വ്യാഖ്യാനഗ്രന്ഥമായ മഫാതീഹുല്‍ ഗൈബില്‍, ഈ വചനത്തെ വ്യാഖ്യാനിച്ചുകൊണ്ട് പരാമര്‍ശിക്കുന്നു.(11)

“നുത്വ്ഫതുന്‍ അംശാജുന്‍ എന്നാല്‍ കൂടിച്ചേര്‍ന്നുണ്ടായ ബീജം എന്നു തന്നെയാണ് അര്‍ത്ഥമെന്ന് മുസ്്‌ലിംകളല്ലാത്ത ക്വുര്‍ആന്‍ വ്യാഖ്യാതാക്കള്‍ പോലും സമ്മതിക്കുന്നതാണ്. നടേ പറഞ്ഞ സൂക്തത്തിന് പതിനെട്ടാം നൂറ്റാണ്ടുകാരനായ ഓറിയന്റലിസ്റ്റ് ക്വുര്‍ആന്‍ പരിഭാഷകന്‍ ജോര്‍ജ് സെയില്‍ നല്‍കുന്ന പരിഭാഷ ”Verily, we have created man of mingled seed of btoh sexes” എന്നാണ്.(12) ഇരുപതാം നൂറ്റാണ്ടുകാരനായ ബ്രിട്ടീഷ് ഓറിയന്റലിസ്റ്റ് എ. ജെ. ആര്‍ബെറി ഈ വചനത്തെ പരിഭാഷപ്പെടുത്തിയിരിക്കുന്നത്  ”we created man of a spermdrop, a mingling” എന്നാണ്.(13) സ്ത്രീയുടെയും പുരുഷന്റെയും ബീജങ്ങളുടെ സങ്കലനത്തില്‍നിന്നാണ് അതുണ്ടാവുന്നതെന്ന് ജോര്‍ജ് സെയില്‍ ഈ വചനത്തില്‍നിന്ന് മനസ്സിലാക്കിയത് ഏതെങ്കിലും ഇസ്്‌ലാമിക പ്രബോധകരുടെ സ്വാധീനം കൊണ്ടല്ല, പ്രത്യുത അറബിഭാഷയിലൂടെ ക്വുര്‍ആന്‍ പഠിച്ചപ്പോള്‍ അദ്ദേഹത്തിന് അങ്ങനെ മനസ്സിലായതാണ്. മുന്‍ധാരണയില്ലാതെ ക്വുര്‍ആനെ സമീപിക്കുന്നവര്‍ക്കെല്ലാം ഈ വചനത്തില്‍നിന്ന് സ്ത്രീ-പുരുഷ ബീജങ്ങളുടെ സംഗമമാണ് കുഞ്ഞുണ്ടാവുന്നതിന് നിമിത്തമാകുകയെന്നാണ് മനസ്സിലാവുകയെന്ന് സെയ്‌ലിന്റെ പരിഭാഷ തെര്യപ്പെടുത്തുന്നുണ്ട്.

പുരുഷബീജവും അണ്ഡവും കൂടിച്ചേരുന്ന ബീജസങ്കലന(fertilization)മെന്ന പ്രക്രിയയെക്കുറിച്ച് കൂടുതല്‍ മനസ്സിലാക്കുമ്പോഴാണ് കൂടിച്ചേര്‍ന്നുണ്ടായ നുത്വ്ഫയെന്ന പ്രയോഗം എത്രമാത്രം കൃത്യമാണെന്ന് ബോധ്യപ്പെടുക. ആര്‍ത്തവചക്രത്തിന്റെ മധ്യത്തില്‍ നടക്കുന്ന അണ്ഡോല്‍സര്‍ജ്ജന(Ovulation)മാണ് പെണ്‍ശരീരത്തില്‍ നടക്കുന്ന ബീജസങ്കലനത്തിലേക്കുള്ള ആദ്യപടി. അണ്ഡോല്‍സര്‍ജ്ജനം കഴിഞ്ഞാല്‍ ഒരു ദിവസത്തിലധികം അണ്ഡം ജീവിച്ചിരിക്കില്ല. അതിനകം ബീജസങ്കലനം നടന്നില്ലെങ്കില്‍ അണ്ഡം നശിച്ചുപോകും. അണ്ഡാശയത്തില്‍നിന്ന് പുറത്തുവന്ന് ഫലോപ്പിയന്‍ നാളിയിലെത്തി ബീജത്തെ പ്രതീക്ഷിച്ചുകൊണ്ട് നില്‍ക്കുന്ന അണ്ഡത്തിനടുത്തെത്തുന്ന ഇരുന്നൂറോളം വരുന്ന പുരുഷബീജങ്ങളില്‍ ഒരെണ്ണത്തിന് മാത്രമാണ് അതിന്റെ ‘ഭിത്തി ഭേദിച്ച് അകത്തുകടക്കാനാവുക. ഒരു തവണ സ്ഖലിക്കുന്ന കോടിക്കണക്കിന് ബീജങ്ങളില്‍നിന്ന് നീന്തി അണ്ഡത്തിനടുത്തെത്തുന്നതില്‍ വിജയിക്കുന്ന ഇരുനൂറോളമെണ്ണത്തില്‍നിന്ന് ഒരേ ഒരെണ്ണത്തിനുമാത്രം! അണ്ഡത്തെ പൊതിഞ്ഞുനില്‍ക്കുന്ന മോളിക്കുളാര്‍ കോശങ്ങളുടെ നിരയായ കൊറോണ റേഡിയാറ്റ(Corona radiata)യിലൂടെ വലിഞ്ഞ് അകത്തുകയറി അണ്ഡഭിത്തിയായ സോണ പെല്ലുസിഡ(zona pellucida)യെ  ഭേദിച്ച് അണ്ഡകോശദ്രവ്യത്തിനകത്തെത്തുവാന്‍ കെല്‍പുള്ള ഒരേയൊരു ബീജത്തിനുമാത്രം ലഭിക്കുന്ന അവസരം! ഇങ്ങനെ ഒരു ബീജാണു അകത്തു കയറിക്കഴിഞ്ഞാല്‍ ഉടന്‍ നടക്കുന്ന കോര്‍ട്ടിക്കല്‍ പ്രതിപ്രവര്‍ത്തനങ്ങള്‍ (cortical  reactions) വഴി പിന്നെയൊരു ബീജവും അണ്ഡത്തിനകത്തേക്ക് കടക്കാത്ത സ്ഥിതി സംജാതമാവുന്നു. അതിനുശേഷമാണ് ബീജകോശകേന്ദ്രത്തിലെ ജനിതക വസ്തുക്കള്‍ അണ്ഡകോശത്തിന്റെ കോശദ്രവ്യത്തില്‍ കലരുകയും അവയും അണ്ഡജനതിക വസ്തുക്കളും തമ്മില്‍ യോജിക്കുകയും ചെയ്യുന്നത്. അണ്ഡകോശത്തിലെ 23 ക്രോമോസോമുകളും ബീജകോശത്തിലെ 23 ക്രോമോസോമുകളും കൂടിച്ചേര്‍ന്ന് 46 ക്രോമോസോമുകളുള്ള ഒരു പൂര്‍ണകോശമായിത്തീരുന്ന പ്രക്രിയയാണ് ബീജസങ്കലനം.

ബീജത്തിന്റെയും അണ്ഡത്തിന്റെയും ന്യൂക്ലിയസ്സുകള്‍ ഒരുമിച്ചുചേര്‍ന്ന് 46 ക്രോമോസോമുകളുള്ള ഒരു പൂര്‍ണ്ണ ന്യൂക്ലിയസ് ആകുന്നതിന് മുമ്പ് അണ്ഡത്തിന്റെ കോശദ്രവ്യത്തിനകത്ത് രണ്ട് പ്രോന്യൂക്ലിയസുകള്‍ (pronuclei) ഉള്ള ഒരു ഘട്ടമുണ്ട്. ആണ്‍ പ്രോന്യൂക്ലിയസും പെണ്‍ പ്രോന്യൂക്ലിയസും അണ്ഡകോശദ്രവ്യത്തിനകത്ത് സ്ഥിതി ചെയ്യുന്ന ഘട്ടം. ഈ സമയത്തെ സംയോജിത കോശത്തിന്റെ പുരുഷ പ്രോന്യൂക്ലിയസ് ഒഴികെയുള്ള ഭാഗങ്ങളെല്ലാം പഴയ അണ്ഡത്തിന്റേതിനു സമാനമായിരിക്കും. ശുക്ലകോശത്തിന്റെ കോശസ്തരം അണ്ഡത്തിന്റെ സ്തരവുമായി ചേര്‍ന്ന് അപ്രത്യക്ഷമാവും. ശുക്ലത്തിന്റെ വാലും കോശദ്രവ്യത്തിലുള്ള മൈറ്റോകോണ്‍ട്രിയയുമെല്ലാം പുരുഷ പ്രോന്യൂക്ലിയസ് ഉണ്ടാകുന്നതോടെ നശിച്ചുപോവും. അതുകൊണ്ടാണ് നമ്മുടെയെല്ലാം -പുരുഷനായാലും സ്ത്രീയായാലും- കോശങ്ങള്‍ക്കകത്തെ മൈറ്റോകോണ്‍ട്രിയ നമുക്ക് മാതാവില്‍നിന്ന് ലഭിച്ചതാണെന്ന് പറയുന്നത്. അഥവാ പുരുഷബീജവും സ്ത്രീബീജവും കൂടിച്ചേര്‍ന്ന് ഒരു മൂന്നാം വസ്തുവുണ്ടാവുകയല്ല, സ്ത്രീ ബീജത്തിനകത്ത് പുരുഷന്റെ ജനിതകവസ്തുവിന്റെ കൂടിച്ചേരല്‍ നടക്കുക മാത്രമാണ് ബീജസങ്കലനത്തില്‍ സംഭവിക്കുന്നത്. അണ്ഡത്തിന്റെ കോശദ്രവ്യവും കോശസ്തരവും മൈറ്റോകോണ്‍ട്രിയയുമെല്ലാം തന്നെയാണ് സിക്താണ്ഡത്തിനുമുണ്ടാവുക. അതിന്റെ ന്യൂക്ലിയസിലേക്ക് പുരുഷബീജത്തിന്റെ ജനിതക വസ്തു കൂടിച്ചേരുക മാത്രമാണ് ബീജസങ്കലനത്തില്‍ നടക്കുന്നത്.(14) രണ്ട് അര്‍ധകോശങ്ങള്‍ ചേര്‍ന്ന് പൂര്‍ണകോശമാകുന്ന പ്രക്രിയയെന്ന, ബീജസങ്കലനത്തിന് സാധാരണയായി പറയാറുള്ള നിര്‍വചനത്തിനുപകരം പൂര്‍ണകോശത്തിന്റെ കോശദ്രവ്യവും അര്‍ധന്യൂക്ലിയസുമുള്ള അണ്ഡത്തിലേക്ക് പുരുഷബീജത്തിനകത്തെ അര്‍ധന്യൂക്ലിയസിലെ ജനിതക വസ്തുവിനെ കടത്തിവിട്ട് അതിനെ പൂര്‍ണകോശമാക്കുന്ന പ്രക്രിയയാണ് ബീജസങ്കലനം എന്നു പറയുന്നതാകും കൃത്യമായ നിര്‍വചനം.

ന്യൂക്ലിയസിനെ മാറ്റിനിര്‍ത്തിയാല്‍ അണ്ഡം ഒരു പൂര്‍ണകോശം തന്നെയാണ്. പൂര്‍ണകോശത്തിന്റേതുപോലെയുള്ള ദ്രവ്യവും സ്തരവും മൈറ്റോകോണ്‍ട്രിയയുമെല്ലാമാണ് അണ്ഡകോശത്തിലുമുള്ളത്. അതിനെ പൂര്‍ണകോശമാക്കിതീര്‍ക്കുന്നതിന് ഒരു അര്‍ധന്യൂക്ലിയസ് കൂടി മാത്രം മതി. പ്രസ്തുത അര്‍ധന്യൂക്ലിയസാണ് പുരുഷബീജം നല്‍കുന്നത്. അങ്ങനെ നോക്കുമ്പോള്‍ അണ്ഡത്തിലേക്ക് അര്‍ധന്യൂക്ലിയസ് കൂട്ടിച്ചേര്‍ക്കുന്ന പ്രക്രിയയാണ് ബീജസങ്കലനമെന്ന് പറയാം. സ്ത്രീ നുത്വ്ഫയിലേക്ക് പുരുഷ ജനിതകവസ്തുവിനെ കൂട്ടിച്ചേര്‍ക്കുന്ന പ്രക്രിയ. ഇങ്ങനെ കൂടിച്ചേര്‍ന്നു കഴിഞ്ഞാലും സ്ത്രീ നുത്വ്ഫ, നുത്വ്ഫ തന്നെയായിരിക്കും. അതിന്റെ കോശദ്രവ്യത്തിനോ സ്തരത്തിനോ ആകൃതിക്കോ മാറ്റങ്ങളൊന്നുമുണ്ടാവുകയില്ല. പുരുഷന്റെ ജനിതക വസ്തു കൂടിച്ചേര്‍ന്ന് അംശാജ് ആയിത്തീര്‍ന്നതായിരിക്കും ആ നുത്വ്ഫയെന്നതുമാത്രമാണ് വ്യത്യാസം. മനുഷ്യനെ സൃഷ്ടിച്ചിരിക്കുന്നത് നുത്വ്ഫതുന്‍ അംശാജില്‍’ നിന്നാണെന്ന ക്വുര്‍ആന്‍ പരാമര്‍ശത്തിന്റെ കൃത്യതയാണ് നമുക്കിവിടെ ബോധ്യപ്പെടുന്നത്. പുരുഷജനിതക വസ്തു കൂട്ടിച്ചേര്‍ത്ത സ്ത്രീ നുത്വ്ഫയില്‍ നിന്നാണല്ലോ നമ്മുടെയെല്ലാം തുടക്കം. പ്രസ്തുത നുത്വ്ഫ വിഭജിക്കപ്പെട്ടാണ് നമ്മുടെ ശരീരവും ഇന്ദ്രിയങ്ങളുമെല്ലാം ഉണ്ടായിട്ടുള്ളത്. ”നുത്വ്ഫത്തിന്‍ അംശാജിന്‍” എന്ന പ്രയോഗത്തില്‍നിന്ന് പുരുഷസ്രവത്തിന്റെയും സ്ത്രീസ്രവത്തിന്റെയും സമ്മേളനം വഴിയാണ് കുഞ്ഞുണ്ടാവുന്നതെന്നാണ് സ്വഹാബിമാര്‍ മനസ്സിലാക്കിയതെന്ന് പറയുമ്പോള്‍ അവരാരും തന്നെ ബീജത്തെയും അണ്ഡത്തെയും കുറിച്ച് പറഞ്ഞിട്ടില്ലല്ലോയെന്ന് തര്‍ക്കിക്കുന്നത് ശുദ്ധ അസംബന്ധമാണ്. നുത്വ്ഫയെന്നാല്‍ പുരുഷ സ്രവത്തിന്റെയോ സ്ത്രീ സ്രവത്തിന്റെയോ ഒരു ഒരു തുള്ളിയോ ചെറിയ അളവോയെന്നാണ് അവര്‍ മനസ്സിലാക്കിയിരുന്നതെന്ന് ഹദീഥുകളില്‍ നിന്ന് നമുക്ക് വ്യക്തമായി. അണ്ഡത്തെ വഹിച്ചുകൊണ്ടുള്ള ഫോളിക്കുളാര്‍ ദ്രാവകത്തിന്റെ ചെറിയൊരു അംശമാണ് അണ്ഡമെന്നും ശുക്ലദ്രാവകത്തിന്റെ ചെറിയൊരു അംശമായ ശുക്ലാണുവാണ് അതുമായി യോജിക്കുന്നതെന്നും ഇന്ന് നമുക്കറിയാം. പരീക്ഷണങ്ങളിലൂടെ ആധുനിക മനുഷ്യര്‍ കണ്ടെത്തിയതെല്ലാം പൗരാണികര്‍ക്ക് അറിയാമായിരുന്നുവെന്ന് ആരും വാദിക്കുന്നില്ല. ദിവ്യവെളിപാടുകളുടെ അടിസ്ഥാനത്തില്‍ അന്തിമപ്രവാചകന്‍ പറഞ്ഞതൊന്നുംതന്നെ ആധുനികശാസ്ത്രം കണ്ടെത്തുന്ന വസ്തുതകള്‍ക്ക് എതിരാവുകയില്ലെന്ന് മാത്രമാണ് മുസ്്‌ലിംകളുടെ വാദം. പുരുഷസ്രവത്തിന്റെയോ സ്ത്രീസ്രവത്തിന്റെയോ ചെറിയൊരു അംശമാണ് നുത്വ്ഫയെന്ന് മനസ്സിലാക്കിയവര്‍ നുത്വ്ഫത്തിന്‍ അംശാജിന്‍ എന്ന പ്രയോഗത്തെ വ്യാഖ്യാനിക്കുമ്പോള്‍ സ്ത്രീസ്രവവും പുരുഷസ്രവവും കൂടിച്ചേര്‍ന്നുണ്ടാവുന്ന നുത്വ്ഫയില്‍നിന്നുള്ള മനുഷ്യസൃഷ്ടിയെ കുറിച്ചാണ് ഇവിടെ പറയുന്നതെന്ന് സ്വാഭാവികമായും പരാമര്‍ശിക്കും. അതില്‍നിന്ന് പുരുഷസ്രവവും സ്ത്രീസ്രവവും പൂര്‍ണമായാണ് ശിശുനിര്‍മിതിയില്‍ പങ്കെടുക്കുന്നതെന്നാണ് അവര്‍ മനസ്സിലാക്കിയതെന്ന് കരുതിക്കൂടാത്തതാണ്. സ്ത്രീസ്രവത്തിന്റെ ഭാഗമായ നുത്വ്ഫയും പുരുഷസ്രവത്തിന്റെ ഭാഗമായ നുത്വ്ഫയും കൂടിച്ചേര്‍ന്ന നുത്വ്ഫത്തിന്‍ അംശാജിനില്‍നിന്നാണ് കുഞ്ഞുണ്ടാവുന്നതെന്നാണ് അവര്‍ കരുതിയിരുന്നതെന്നുതന്നെയാണ് അവരുടെ പരാമര്‍ശങ്ങള്‍ വ്യക്തമാക്കുന്നത്.

പുരുഷബീജത്തെയും അണ്ഡത്തെയും കുറിച്ച് നുത്വ്ഫയെന്ന് പ്രയോഗിച്ച ക്വുര്‍ആന്‍ സിക്താണ്ഡത്തെ (zygote) കുറിക്കാന്‍ “നുത്വ്ഫത്തിന്‍ അംശാജിന്‍’ എന്നാണ് പ്രയോഗിച്ചതെന്ന വസ്തുത ശ്രദ്ധേയമാണ്. സ്ത്രീയുടെ നുത്വ്ഫയിലേക്ക് പുരുഷ ജനിതകവസ്തുവിനെ കൂട്ടിച്ചേര്‍ക്കുന്ന പ്രക്രിയയാണ് ബീജസങ്കലനമെന്ന് നാം മനസ്സിലാക്കി. പ്രസ്തുത പ്രക്രിയ കഴിഞ്ഞ ശേഷമുള്ള ബീജത്തെ കുറിക്കാന്‍ ഏറ്റവും കൃത്യമായ പദം തന്നെയാണ് ക്വുര്‍ആന്‍ ഉപയോഗിച്ചിരിക്കുന്നത്. കൂട്ടിച്ചേര്‍ക്കപ്പെട്ട ബീജം-നുത്വ്ഫത്തിന്‍ അംശാജിന്‍! സര്‍വജ്ഞനായ അല്ലാഹുവിനല്ലാതെ ആര്‍ക്കാണ് ഇത്ര കൃത്യമായി പദങ്ങള്‍ പ്രയോഗിക്കാന്‍ കഴിയുക!

  • കുറിപ്പുകള്‍:
  1. ഗര്‍ഭോപനിഷത്ത്, വചനങ്ങള്‍ 2,3; ഉപനിഷദ്‌സര്‍വസ്വം, തൃശൂര്‍, 2001, പുറം 63-68.
  2. ഇയ്യോബ് 10: 9-11.
  3. Hippocrates: ‘The Seed’, Sections 5-7, Hippocratic Writings, Page 319-320.
  4. Aristotle: On the Generation of Animals, Montana, 2004, page 3-229.
  5. Phillip de Lacy: Corpus Medicorum Graecorum: Galeni de Semine (Galen: On Semen) (Greek text with Englisht yrans), Akademie Verlag, 20-Nov-1992, section I: 9:1-10, page 107-109.
  6. വിശുദ്ധ ഖുര്‍ആന്‍ 76: 2.
  7. ലിസാനുല്‍ അറബ്.
  8. സ്വഹീഹു മുസ്‌ലിം, കിതാബുല്‍ ഹൈദ്വ്.
  9. സ്വഹീഹു മുസ്‌ലിം, കിതാബുണ്‍ ക്വദ്യ്യ.
  10. തഫ്‌സീര്‍ അത്ത്വബ്‌രി 76: 2.
  11. ഇമാം റാസി: ജാമിഉല്‍ ബയാന്‍ ഫീ തഫ്‌സീറില്‍ ഖുര്‍ആന്‍ (http://www.altafsir.com/)
  12. George Sale : The Koran (Al-Qur’an) (http://www.gutenberg.org/).
  13. Arthur John Arberry: The Koran Interpreted, Page 315.
  14. Elaine N. Marieb& Katja Hoehn: Anatomy & Physiology,  London, 2012, Pages 1119- 1121.
വിഷയവുമായി ബന്ധപ്പെട്ട വീഡിയോ
ല്ല. സ്രവിക്കപ്പെടുന്ന ശുക്ലത്തിന്റെ ചെറിയ ഒരു ഭാഗം മാത്രമായ ഒരേയൊരു ബീജം മാത്രമാണ് കുഞ്ഞിന്റെ നിർമ്മിതിയിൽ പങ്കെടുക്കുന്നതെന്നാണ് ക്വുർആനും നബിവചനങ്ങളൂം വ്യക്തമാക്കുന്നത്.
 പുരുഷ ശുക്ലം ഘനീഭവിച്ചാണ് കുഞ്ഞുണ്ടാകുന്നതെന്ന് ധരിച്ചവരും സ്ത്രീശുക്ലമോ ആര്‍ത്തവരക്തമോ കട്ടിയായാണ് ഭ്രൂണമുണ്ടാകുന്നതെന്ന് കരുതിയവരുമായ പൗരാണികരെല്ലാം വിചാരിച്ചത് ഭ്രൂണനിര്‍മാണത്തില്‍ പങ്കെടുക്കുന്നത് സ്രവം പൂര്‍ണമായിട്ടാണെന്നായിരുന്നുവെന്ന് ഗര്‍ഭോപനിഷത്ത് മുതല്‍ ഗാലന്റെ ഗ്രന്ഥങ്ങള്‍ വരെയുള്ളവ പരിശോധിച്ചാല്‍ വ്യക്തമാവും. പുരുഷ ശുക്ലത്തില്‍ ഒളിഞ്ഞിരിക്കുന്ന കുഞ്ഞിന്റെ പ്രാഗ് രൂപം സ്ത്രീശരീരത്തില്‍നിന്ന് പുറത്തുവരാതെ തങ്ങിനില്‍ക്കുന്ന ആര്‍ത്തവരക്തത്തില്‍നിന്ന് പോഷണങ്ങള്‍ സ്വീകരിച്ച് ഗര്‍ഭാശയത്തില്‍വെച്ച് വളരുകയാണ് ചെയ്യുന്നതെന്ന് വാദിച്ച നടേരൂപകരണ സിദ്ധാന്തക്കാരും (Preformationists) ആര്‍ത്തവരക്തം പുരുഷ ശുക്ലത്താല്‍ പ്രചോദിതമാകുമ്പോള്‍ അത് ഘനീഭവിക്കുകയും അതിനുശേഷം സ്ത്രീ ശരീരത്തില്‍നിന്ന് പോഷണങ്ങള്‍ സ്വീകരിച്ച് അവയവങ്ങള്‍ രൂപീകരിക്കപ്പെടുകയുമാണ് ചെയ്യുന്നതെന്ന് വാദിച്ച സ്വയം ഉല്‍പാദന സിദ്ധാന്തക്കാരും (epigenesists) തമ്മില്‍ നടന്ന ആശയ സംഘട്ടനങ്ങളുടെ പശ്ചാത്തലത്തിലാണ് ആധുനിക ഭ്രൂണശാസ്ത്രം ജനിക്കുന്നത്.  സ്ത്രീയുടെ ശുക്ലത്തില്‍നിന്നോ ആര്‍ത്തവരക്തത്തില്‍നിന്നോ ഏതിൽനിന്നാണ് കുഞ്ഞുണ്ടാവുന്നതെന്ന് തർക്കിച്ചവരെല്ലാം പക്ഷെ, പ്രസ്തുത സ്രവങ്ങളില്‍നിന്ന് പൂര്‍ണമായാണ് കുഞ്ഞിന്റെ സൃഷ്ടി  നടക്കുന്നതെന്ന് തന്നെയാണ് വിചാരിച്ചിരുന്നത്. സ്രവമേതാണെങ്കിലും അത് പൂര്‍ണമായി തന്നെയാണ് ഭ്രൂണനിര്‍മിതിയില്‍ പങ്കെടുക്കുന്നതെന്നായിരുന്നു എല്ലാവരും കരുതിയിരുന്നത് എന്നര്‍ത്ഥം. (1)
ഇംഗ്ലീഷ് ജീവശാസ്ത്രജ്ഞനായിരുന്ന റോബര്‍ട്ട് ഹുക്കിന്റെ കോശ നിരീക്ഷണമാണ് ഭ്രൂണ ശാസ്ത്രരംഗത്ത് വഴിത്തിരിവായിത്തീര്‍ന്ന പ്രധാനപ്പെട്ട ഒരു സംഭവം ഓസ്‌കാര്‍ ഹെര്‍ട്്‌വിഗും റിച്ചാര്‍ഡ് ഹെര്‍ട്‌വിഗും കൂടി കടല്‍ച്ചൊരുക്കുകളില്‍ നടക്കുന്ന ബീജസങ്കലനത്തെക്കുറിച്ച് നടത്തിയ വിശദമായ പഠനങ്ങളോടെയാണ് പുരുഷശുക്ലത്തിലെ നിരവധി ബീജങ്ങളിലൊന്ന് മാത്രമാണ് അണ്ഡമായി ചേര്‍ന്ന് കുഞ്ഞുണ്ടാകുന്നതില്‍ പങ്കാളിയാവുന്നതെന്ന് ശാസ്ത്രലോകത്തിന് മനസ്സിലായത്. 1677ല്‍ ആന്റണി വാന്‍ ല്യൂവെന്‍ ഹോക്ക് തന്റെ സൂക്ഷ്മ ദര്‍ശിനിയിലൂടെ ശുക്ലദ്രാവകത്തിനകത്തെ ബീജാണുക്കളെ കണ്ടിരുന്നുവെന്നതിനാല്‍ അദ്ദേഹമാണ് പുരുഷബീജം കണ്ടുപിടിച്ചതെന്നാണ് പൊതുവെ വ്യവഹരിക്കുന്നതെങ്കിലും ശുക്ലദ്രാവകത്തിലെ നിരവധി ബീജാണുക്കളില്‍ ഒരെണ്ണം മാത്രമാണ് ബീജസങ്കലനത്തില്‍ പങ്കെടുക്കുന്നതെന്ന് ശാസ്ത്രലോകം പൂര്‍ണാര്‍ത്ഥത്തില്‍ അംഗീകരിച്ചത് ഇരുപതാം നൂറ്റാണ്ടിന്റെ തുടക്കത്തില്‍ മാത്രമാണ് (2)
സ്ത്രീ ശരീരത്തില്‍ വെച്ചുള്ള കുഞ്ഞിന്റെ നിര്‍മിതിയുടെ പ്രഥമഘട്ടത്തെക്കുറിച്ച് ക്വുര്‍ആന്‍ പറയുന്നത് ‘നിസ്സാരമായ ഒരു ദ്രാവകത്തില്‍നിന്ന്’ എന്നാണ്:
  ”അവന്‍ തന്നെയാണ് വെള്ളത്തില്‍ നിന്ന് മനുഷ്യനെ സൃഷ്ടിക്കുകയും, അവനെ രക്തബന്ധമുള്ളവനും വിവാഹബന്ധമുള്ളവനും ആക്കുകയും ചെയ്തിരിക്കുന്നത്. നിന്റെ രക്ഷിതാവ് കഴിവുള്ളവനാകുന്നു.”(3)
  ”നിസ്സാരമായ ഒരു ദ്രാവകത്തില്‍ നിന്ന് നിങ്ങളെ നാം സൃഷ്ടിച്ചില്ലേ?”(4)
മനുഷ്യനെ ജലത്തില്‍ (മാഅ്) നിന്നാണ് സൃഷ്ടിച്ചതെന്ന സൂറത്തുല്‍ ഫുര്‍ക്വാനിലെ വചനത്തിന്റെ വിശദീകരണമാണ് ‘നിസ്സാരമായ ദ്രാവക'(മാഇന്‍ മഹീന്‍)ത്തില്‍ നിന്നാണ് അത് നിര്‍വഹിച്ചതെന്ന സൂറത്തുല്‍ മുര്‍സലാത്തിലെ വചനം. നിസ്സാരമായ ദ്രാവകമെന്നതുകൊണ്ടുള്ള വിവക്ഷ പുരുഷ ശുക്ലമാണെന്ന് വ്യക്തമാണ്. നിസ്സാരവും വിലയൊന്നുമില്ലാത്തതുമായി പരിഗണിക്കപ്പെടുന്ന ശുക്ല ദ്രാവകത്തെക്കുറിച്ച് ‘മാഇന്‍ മഹീന്‍’ എന്ന് പറഞ്ഞതോടൊപ്പം പ്രസ്തുത ദ്രാവകത്തില്‍നിന്ന് പൂര്‍ണമായല്ല മനുഷ്യ സൃഷ്ടി നടക്കുന്നതെന്ന് ക്വുര്‍ആന്‍ തന്നെ വ്യക്തമാക്കുന്നുണ്ട്.
സൂറത്തുസ്സജദയിലെ എട്ടാം വചനം നോക്കുക:”പിന്നെ അവന്റെ സന്തതിയെ നിസ്സാരമായ ഒരു വെള്ളത്തിന്റെ സത്തില്‍ നിന്ന് അവന്‍ ഉണ്ടാക്കി.”(5)
‘സുലാലത്തിന്‍ മിന്‍ മാഇന്‍ മഹീന്‍’ എന്ന പ്രയോഗത്തെയാണ് ഇവിടെ ‘നിസ്സാരമായ ഒരു ദ്രാവകത്തിന്റെ സത്തില്‍നിന്ന്’ എന്ന് പരിഭാഷപ്പെടുത്തിയിരിക്കുന്നത്. ആള്‍ക്കൂട്ടത്തില്‍നിന്ന് വ്യക്തികള്‍ ചോര്‍ന്നുപോവുകയെന്ന അര്‍ത്ഥത്തിലുള്ളതാണ് ഈ പ്രയോഗം. സീന്‍, ലാം, ലാം അക്ഷരത്രയത്തില്‍നിന്ന് നിഷ്പന്നമായ ഇതിന്റെ ക്രിയാധാതു പൊടിയില്‍നിന്ന് മുടിയെടുക്കുന്നതുപോലെയോ ഉറയില്‍നിന്ന് വാള്‍ എടുക്കുന്നതുപോലെയോ വലിയ ഒന്നില്‍നിന്ന് ചെറിയ ഒന്നിനെ പുറത്തെടുക്കുന്നതിനാണ് പൊതുവെ ഉപയോഗിക്കാറുള്ളതെന്നും എന്തെങ്കിലും ഒന്ന് പിഴിഞ്ഞ് അതിന്റെ സത്തെടുക്കുന്നതിനും ഇത് പ്രയോഗിക്കാറുണ്ടെന്നും താജുൽ ഉറൂസിനെപ്പോലെയുള്ള അറബി ശബ്ദതാരാവലികളും (7) ലെയിനിന്റെ അറബി പദവിജ്ഞാനകോശവും വിശദീകരിക്കുന്നുണ്ട്..(8)
ഒരു സാധനത്തിന്റെ സത്ത് എന്നോ അതിന്റെ ഏറ്റവും നല്ല ഭാഗം എന്നോ സംശുദ്ധമായ അതിന്റെ അംശം എന്നോ എല്ലാം സുലാലത്തിന് അര്‍ത്ഥം പറയാം.. പുരുഷന്‍ സ്ഖലിക്കുന്ന രണ്ട് കോടിയോളം വരുന്ന ബീജങ്ങളില്‍ ലക്ഷണമൊത്ത ഒരെണ്ണം, അഥവാ സ്രവത്തിന്റെ ഏറ്റവും അനുയോജ്യമായ അംശം മാത്രമാണ് അണ്ഡവുമായി സംയോജിച്ച് കുഞ്ഞായി തീരുന്നത്. ശുക്ലദ്രാവകത്തില്‍നിന്നുള്ള ഏറ്റവും നല്ല ഭാഗം മാത്രം! ശുക്ല ദ്രാവകത്തിന്റെ ‘സുലാലത്ത്’ അഥവാ സംശുദ്ധമായ സത്ത് തന്നെയാണ് ബീജസങ്കലനത്തില്‍ പങ്കെടുത്ത് കുഞ്ഞിന്റെ ജനനത്തിന് നിമിത്തമായിത്തീരുന്നത്.
പുരുഷന്‍ സ്രവിക്കുന്ന ശുക്ലദ്രാവകത്തിന് അറബിയില്‍ പറയുക ‘മനിയ്യ്’ എന്നാണ്. ക്വുര്‍ആനില്‍ ഒരേയൊരു തവണ മാത്രമാണ് ഈ നാമരൂപം പ്രയോഗിച്ചിരിക്കുന്നത്. പ്രസ്തുത പ്രയോഗമിങ്ങനെയാണ്:
”അവന്‍ സ്രവിക്കപ്പെടുന്ന ശുക്ലത്തില്‍ നിന്നുള്ള ഒരു കണമായിരുന്നില്ലേ?”(9)
ഇവിടെ‘മനിയ്യി’ല്‍ നിന്നാണ് മനുഷ്യന്‍ സൃഷ്ടിക്കപ്പെട്ടത് എന്നല്ല പറഞ്ഞിരിക്കുന്നതെന്ന വസ്തുത ശ്രദ്ധേയമാണ്. ‘നുത്വ്ഫത്തിന്‍ മിന്‍ മനിയ്യിന്‍ യുമ്‌ന’ യെന്നാണ് സ്ത്രീ ശരീരത്തില്‍വെച്ചുള്ള മനുഷ്യസൃഷ്ടിയുടെ പ്രാഥമിക ഘട്ടത്തെക്കുറിച്ച് അവിടെയുള്ള ക്വുര്‍ആനിക പ്രയോഗം. ‘മനിയ്യിന്‍ യുമ്‌ന’ എന്നാല്‍ സ്രവിക്കപ്പെടുന്ന ശുക്ലമെന്നാണ് അര്‍ത്ഥം. ‘മിന്‍’ എന്ന വിവേചക ഭേദകം ഉപയോഗിക്കാറുള്ളത് ‘ഒന്നില്‍നിന്ന്’ എന്ന അര്‍ത്ഥത്തിലാണ്. ‘ഹുദന്‍ മിന്‍ റബ്ബിഹിം’ എന്ന പ്രയോഗം ക്വുര്‍ആനില്‍ നിരന്തരമായി കാണാം. ‘നിങ്ങളുടെ നാഥനില്‍ നിന്നുള്ള മാര്‍ഗദര്‍ശനം’ എന്നാണ് ഇതിനര്‍ത്ഥം. അതുകൊണ്ടാണ് ‘നുത്വ്ഫത്തിന്‍ മിന്‍ മനിയ്യി’ന് ‘ശുക്ലത്തില്‍നിന്നുള്ള ഒരു കണം’ എന്ന് അര്‍ത്ഥം നല്‍കിയിരിക്കുന്നത്. സ്രവിക്കപ്പെടുന്ന ശുക്ലത്തില്‍നിന്നുള്ള ഒരു നുത്വ്ഫയെന്നാണ് ക്വുര്‍ആന്‍ പറഞ്ഞതെന്ന കാര്യം ശ്രദ്ധിക്കുക. ‘നുത്വ്ഫ’ ഏകവചനമാണ്. നിത്വാഫ്, നുത്വ്ഫ് എന്നിവയാണ് അതിന്റെ ബഹുവചനരൂപങ്ങള്‍. ശുക്ലം നിരവധി നുത്വ്ഫകളുള്ള ദ്രാവകമാണെന്നും അതില്‍നിന്നുള്ള ഒരു നുത്വ്ഫയാണ് ബീജ സങ്കലനത്തില്‍ പങ്കെടുക്കുന്നതെന്നും ‘നുത്വ്ഫത്തിന്‍ മിന്‍ മനിയ്യ്’ എന്ന ക്വുര്‍ആന്‍ പ്രയോഗം വ്യക്തമാക്കുന്നുണ്ട്.
പുരുഷസ്രവത്തില്‍നിന്ന് പൂര്‍ണമായല്ല, അതിന്റെ ചെറിയൊരു ഭാഗത്തുനിന്നാണ് കുഞ്ഞുണ്ടാവുന്നതെന്ന വസ്തുത പ്രവാചകന്‍(സ) കൃത്യമായി പഠിപ്പിച്ചിട്ടുണ്ട്. ഒരു ഹദീഥ് കാണുക:
അബൂസഈദുല്‍ ഖുദ്‌രി(റ) നിവേദനം: നബി(സ)യോട് ചിലർ അസ്‌ലിനെപ്പറ്റി ചോദിച്ചു. . നിങ്ങളാരും അങ്ങനെ ചെയ്യരുത് എന്ന് നബി (സ) പറഞ്ഞില്ല. നബി (സ) പറഞ്ഞു: ‘അല്ലാഹുവല്ലാതെ ആരെയും സൃഷ്ടിക്കുന്നില്ല.....മൊത്തം സ്രവത്തില്‍നിന്നല്ല കുഞ്ഞുണ്ടാവുന്നത്. അല്ലാഹു ഒന്നിനെ സൃഷ്ടിക്കണമെന്ന് ഉദ്ദേശിച്ചാല്‍ അതിനെ ഒന്നും തടയുന്നതല്ല.'(10)
ശുക്ലസ്ഖലനത്തിനുമുമ്പ് യോനിയില്‍നിന്ന് ലിംഗം പിന്‍വലിച്ച് പുറത്തുകളയുന്ന മൈഥുനവിരാമ (coitus Interruptus/അസ്ല്‍)ത്തെക്കുറിച്ച ചോദ്യത്തിന് പ്രവാചകന്‍ (സ) നല്‍കിയ മറുപടിയാണ് ഈ ഹദീഥിലുള്ളത്. ലിംഗം യോനിയില്‍നിന്ന് പിന്‍വലിക്കുന്നതിനിടയില്‍ സംഭവിച്ചേക്കാവുന്ന സ്ഖലനത്തിനിടയില്‍ ശുക്ലദ്രാവകത്തിന്റെ അല്‍പമെങ്കിലും ജനനേന്ദ്രിയത്തില്‍ പതിക്കാനിടയായാല്‍ അത് ബീജസങ്കലനത്തിനും അതുവഴി കുഞ്ഞിന്റെ ജനനത്തിനും കാരണമായേക്കാം എന്ന വസ്തുതയാണ് പ്രവാചകന്‍ (സ) ഇവിടെ പഠിപ്പിക്കുന്നത്. ‘മൊത്തം സ്രവത്തില്‍നിന്നല്ല കുഞ്ഞുണ്ടാവുന്നത്’ ( മാ മിന്‍ കുല്ലില്‍ മാഇ യകൂനുല്‍ വലദു) എന്നാണ് ഇവിടുത്തെ പ്രവാചക പ്രയോഗം. പുരുഷ സ്രവത്തിന്റെ ചെറിയ ഒരു അംശം സ്ത്രീ ജനനേന്ദ്രിയത്തില്‍ പതിച്ചാലും കുഞ്ഞുണ്ടാവുമെന്നു പറഞ്ഞാല്‍ അതിനര്‍ത്ഥം ശിശുനിര്‍മിതിക്ക് നിദാനമായതെന്താണെങ്കിലും അത് ശുക്ലദ്രാവകത്തില്‍ പരന്നുകിടക്കുകയാണെന്നതാണ്. അങ്ങനെ പരന്നുകിടക്കുന്ന വസ്തുക്കളെയാണ് ക്വുര്‍ആന്‍ നുത്വ്ഫയെന്ന് വിളിക്കുന്നത്.
‘നുത്വ്ഫത്തിന്‍ മിന്‍ മനിയ്യിന്‍ യുമ്‌ന’യെന്ന ക്വുര്‍ആനിക പ്രയോഗത്തിനുള്ള വിശദീകരണം ഈ ഹദീഥ് നല്‍കുന്നുണ്ട്. സ്രവിക്കപ്പെടുന്ന മനിയ്യിന്റെ ചെറിയൊരു അംശമെങ്കിലും സ്ത്രീ ജനനേന്ദ്രിയത്തില്‍ പതിച്ചാല്‍ അതില്‍നിന്ന് കുഞ്ഞുണ്ടാവുമെങ്കില്‍ അതിന്നര്‍ത്ഥം പ്രസ്തുത അംശത്തില്‍ കുഞ്ഞിന്റെ നിര്‍മിതിക്കാവശ്യമായ നുത്വ്ഫയുണ്ടെന്നാണ്. സ്രവത്തിന്റെ ചെറിയൊരംശത്തിലും നുത്വ്ഫയുണ്ടാകുമെന്ന് പറഞ്ഞാല്‍ ദ്രാവകാംശങ്ങളിലെല്ലാം നുത്വ്ഫകളുടെ സാന്നിധ്യമുണ്ടാകുമെന്നും ഇത്തരം കുറേ നുത്വ്ഫകള്‍ അടങ്ങിയതാണ് ശുക്ല ദ്രാവകമെന്നുമാണ് സാരം. സ്രവിക്കപ്പെടുന്ന ശുക്ലദ്രാവകത്തില്‍ നിരവധി നുത്വ്ഫകളുണ്ടെന്നും അതില്‍ ഒരു നുത്വ്ഫയാണ് ബീജ സങ്കലനത്തില്‍ പങ്കെടുക്കുന്നതെന്നും നടേ പറഞ്ഞ ക്വുര്‍ആന്‍ വചനങ്ങളും ഹദീഥും കൂട്ടി വായിച്ചാല്‍ കൃത്യമായി ബോധ്യപ്പെടുന്നുണ്ട്.
എന്താണീ നുത്വ്ഫ? നൂന്, ത്വ, ഫ എന്നീ അക്ഷരത്രയങ്ങളില്‍ നിന്നുള്ള ക്രിയാധാതു മൃദുവായി ഒഴുക്കുക, പുറംതള്ളുക, സ്രവിക്കുക, വിയര്‍ക്കുക, ഉറ്റിവീഴുക, നിര്‍ഗളിക്കുക, ഒലിച്ചിറങ്ങുക എന്നീ അര്‍ത്ഥങ്ങളിലാണ് പ്രയോഗിക്കാറുള്ളത്. നുത്വ്ഫയെന്ന ഏകവചനനാമത്തിന് ശുദ്ധജലം, ജലകണം, ചെറിയ മുത്ത് എന്നിങ്ങനെയാണ് സാധാരണ നിഘണ്ടുക്കള്‍ അര്‍ത്ഥം പറഞ്ഞുവരുന്നത്.(11) നെറ്റിയില്‍ നിര്‍ഗളിക്കുന്ന വിയര്‍പ്പുകണങ്ങള്‍ക്ക് ‘നിത്വാഫ്’ എന്ന നുത്വ്ഫയുടെ ബഹുവചനമുപയോഗിക്കും. ചെറിയ അളവ് വെള്ളത്തിനോ വെള്ളം നിറക്കുന്ന പാത്രത്തില്‍ അവശേഷിക്കുന്ന അല്‍പം ജലത്തിനോ നുത്വ്ഫയെന്നു പറയുമെന്ന് പ്രസിദ്ധമായ ‘ലിസാനുല്‍ അറബ്’ അറബി ശബ്ദതാരാവലി വ്യക്തമാക്കുന്നുണ്ട്. (12) ജലപാത്രത്തിലുള്ള ചെറിയ അളവ് വെള്ളത്തെ സൂചിപ്പിച്ചുകൊണ്ട് ‘നുത്വ്ഫ’യെന്ന സ്വഹീഹു മുസ്്‌ലിമിലുള്ള ഒരു ഹദീഥില്‍ പ്രയോഗിച്ചിട്ടുണ്ട്.(13) മൊത്തം ദ്രാവകവുമായി താരതമ്യം ചെയ്യുമ്പോള്‍ ചെറിയൊരു അംശം ദ്രാവകത്തെ ഉദ്ദേശിച്ചുകൊണ്ടാണീ പ്രയോഗമെന്നാണ് ഇതെല്ലാം വ്യക്തമാക്കുന്നത്. പ്രസിദ്ധമായ ലെയിനിന്റെ അറബി- ഇംഗ്ലീഷ് ലെക്‌സിക്കണ്‍ നുത്വ്ഫക്ക് നല്‍കുന്ന അര്‍ത്ഥം Sperma of a man or a woman എന്നാണ്.(14) ലാറ്റിനില്‍ Sperma എന്നാല്‍ വിത്ത് അല്ലെങ്കില്‍ ബീജമെന്നാണ് അര്‍ത്ഥം. സ്ത്രീശരീരത്തില്‍ വെച്ചുള്ള കുഞ്ഞിന്റെ നിര്‍മിതിയുടെ പ്രഥമ ഘട്ടത്തെക്കുറിച്ച് നുത്വ്ഫയെന്ന് ക്വുര്‍ആനില്‍ നിരവധി തവണ പ്രയോഗിച്ചിട്ടുണ്ട്:
”മനുഷ്യനെ അവന്‍ ഒരു നുത്വ്ഫയില്‍ നിന്ന് സൃഷ്ടിച്ചു. എന്നിട്ട് അവനതാ വ്യക്തമായ എതിര്‍പ്പുകാരനായിരിക്കുന്നു.”(15)
”അവന്റെ ചങ്ങാതി അവനുമായി സംവാദം നടത്തിക്കൊണ്ടിരിക്കെ പറഞ്ഞു: മണ്ണില്‍നിന്നും അനന്തരം നുത്വ്ഫയില്‍ നിന്നും നിന്നെ സൃഷ്ടിക്കുകയും, പിന്നീട് നിന്നെ ഒരു പുരുഷനായി സംവിധാനിക്കുകയും ചെയ്തവനില്‍ നീ അവിശ്വസിച്ചിരിക്കുകയാണോ?”(16)
”മനുഷ്യരേ, ഉയിര്‍ത്തെഴുന്നേല്‍പിനെ പറ്റി നിങ്ങള്‍ സംശയത്തിലാണെങ്കില്‍ (ആലോചിച്ച് നോക്കുക:) തീര്‍ച്ചയായും നാമാണ് നിങ്ങളെ മണ്ണില്‍നിന്നും, പിന്നീട് നുത്വ്ഫയില്‍ നിന്നും, പിന്നീട് ഭ്രൂണത്തില്‍നിന്നും, അനന്തരം രൂപം നല്‍കപ്പെട്ടതും രൂപം നല്‍കപ്പെടാത്തതുമായ മാംസപിണ്ഡത്തില്‍നിന്നുംസൃഷ്ടിച്ചത്.”(17)
”അല്ലാഹു നിങ്ങളെ മണ്ണില്‍ നിന്നും പിന്നീട് നുത്വ്ഫയില്‍ നിന്നും സൃഷ്ടിച്ചു. പിന്നെ അവന്‍ നിങ്ങളെ ഇണകളാക്കി......”(18)
”മനുഷ്യന്‍ കണ്ടില്ലേ; അവനെ നാം ഒരു നുത്വ്ഫയില്‍ നിന്നാണ് സൃഷ്ടിച്ചിരിക്കുന്നതെന്ന്? എന്നിട്ട് അവനതാ ഒരു പ്രത്യക്ഷമായ എതിര്‍പ്പുകാരനായിരിക്കുന്നു.”(19)
”മണ്ണില്‍നിന്നും, പിന്നെ നുത്വ്ഫയില്‍നിന്നും, പിന്നെ ഭ്രൂണത്തില്‍ നിന്നും നിങ്ങളെ സൃഷ്ടിച്ചത് അവനാകുന്നു. പിന്നീട് ഒരു ശിശുവായി നിങ്ങളെ അവന്‍ പുറത്തുകൊണ്ടുവരുന്നു.”(20)
”ആണ്‍, പെണ്‍ എന്നീ രണ്ട് ഇണകളെ അവനാണ് സൃഷ്ടിച്ചതെന്നും; ഒരു നുത്വ്ഫ സ്രവിക്കപ്പെടുമ്പോള്‍ അതില്‍ നിന്ന്.”(21)
”ഏതൊരു വസ്തുവില്‍ നിന്നാണ് അല്ലാഹു അവനെ സൃഷ്ടിച്ചത്? ഒരു നുത്വ്ഫയില്‍ നിന്ന് അവനെ സൃഷ്ടിക്കുകയും, എന്നിട്ട് അവനെ വ്യവസ്ഥപ്പെടുത്തുകയും ചെയ്തു.”(22)
പുരുഷസ്രവമായ മനിയ്യിന്റെ ഒരു അംശമായ നുത്വ്ഫയെക്കുറിച്ചാണ് ഈ വചനങ്ങളിലെല്ലാം  പറഞ്ഞിരിക്കുന്നതെന്നാണ് ക്വുര്‍ആനിന്റെ പ്രഥമ സംബോധിതര്‍ മനസ്സിലാക്കിയതെന്ന് വ്യക്തമാക്കുന്ന രേഖകളുണ്ട്. ആദ്യകാല ക്വുര്‍ആന്‍ വ്യാഖ്യാതാക്കളില്‍ പ്രമുഖനായ ഇമാം ത്വബ്‌രി തന്റെ ഹിജ്‌റ 270ല്‍ (ക്രിസ്താബ്ദം 883) പൂര്‍ത്തിയാക്കിയ ക്വുര്‍ആന്‍ വ്യാഖ്യാന ഗ്രന്ഥത്തില്‍ സൂറത്തുല്‍ കിയാമയിലെ 37ാം വചനത്തെ വ്യാഖ്യാനിച്ചുകൊണ്ട് പറയുന്നത്് ‘പുരുഷ അരക്കെട്ടുകളില്‍നിന്നുള്ള ശുക്ലത്തിലെ (മനിയ്യ്) വളരെ കുറഞ്ഞ അളവിലുള്ള ദ്രാവകം’ (മാഉന്‍ ഖലീലുന്‍ ഫീ സ്വുല്‍ബിര്‍റജുലി മിന്‍ മനിയ്യി) എന്നാണ്.(23)
ഇരുപതാം നൂറ്റാണ്ടിലെ ആദ്യപാതിയില്‍ ജീവിച്ച പ്രസിദ്ധ ഓറിയന്റലിസ്റ്റ് ക്വുര്‍ആന്‍ വിവര്‍ത്തകനായ ആര്‍തര്‍ ജോണ്‍ ആര്‍ബെറി നുത്വ്ഫക്ക്  നല്‍കുന്ന പരിഭാഷ sperm drop എന്നാണ്.(24) ജൂത വിവര്‍ത്തകനായ എന്‍. ജെ. ദാവൂദ് ‘നുത്വ്ഫത്തുന്‍ മിന്‍ മനിയ്യിന്‍ യുമ്‌ന’ക്ക് നല്‍കുന്ന പരിഭാഷ മ drop of ejaculated semen എന്നാണ്.(25) ഒരുവിധം എല്ലാ ക്വുര്‍ആന്‍ വ്യാഖ്യാതാക്കളും ഈ വചനത്തിലെ നുത്വ്ഫക്ക് നല്‍കുന്ന അര്‍ത്ഥം ഒരു തുള്ളിയെന്നോ ഒരു കണം എന്നോ ആണ്. സ്രവിക്കപ്പെടുന്ന ശുക്ലദ്രാവകത്തിന്റെ ഒരു തുള്ളിയില്‍ നിന്നോ ഒരു കണത്തില്‍നിന്നോ ആണ് ബീജസങ്കലനവും അങ്ങനെ കുഞ്ഞിന്റെ നിര്‍മിതിയും നടക്കുന്നതെന്നായിരുന്നു ക്വുര്‍ആനില്‍നിന്നും ഹദീഥുകളില്‍നിന്നുമെല്ലാം അവര്‍ മനസ്സിലാക്കിയിരുന്നതെന്നാണ് ഇതെല്ലാം വ്യക്തമാക്കുന്നത്.
പുരുഷ ശരീരത്തില്‍നിന്ന് സ്രവിക്കപ്പെടുന്ന നിസ്സാരമായ ഒരു ദ്രാവകത്തിന്റെ സത്തില്‍നിന്നാണ് മനുഷ്യ സൃഷ്ടി നടക്കുന്നതെന്നും ശുക്ലദ്രാവകത്തിലുള്ള നിരവധി നുത്വ്ഫകളില്‍ ഒരു നുത്വ്ഫയാണ് ഭ്രൂണനിര്‍മ്മാണത്തില്‍ പങ്കെടുക്കുന്നതെന്നും ക്വുര്‍ആന്‍ വചനങ്ങളും ഹദീഥുകളും സൂചിപ്പിക്കുന്നത് പതിനാലു നൂറ്റാണ്ടുകള്‍ക്കുമുമ്പാണെന്ന് നാം ഓര്‍ക്കണം. ശുക്ലത്തില്‍നിന്നോ ആര്‍ത്തവരക്തത്തില്‍നിന്നോ ഏതില്‍ നിന്നാണ് കുഞ്ഞുണ്ടാവുന്നതെന്ന് തത്ത്വജ്ഞാനികള്‍ തര്‍ക്കിച്ചിരുന്ന കാലത്താണീ സൂചന നല്‍കുന്നത്. പുരുഷസ്രവത്തിന്റെ പൂര്‍ണതയെ ദ്യോതിപ്പിക്കുന്ന മനിയ്യ് മുഴുവനായിട്ടാണ് ശിശുനിര്‍മിതിയില്‍ പങ്കാളിയാവുന്നതെന്ന ഒരു സൂചന പോലും ക്വുര്‍ആനിലോ ഹദീഥുകളിലോ കാണാനാവുന്നില്ല.
കുറിപ്പുകള്‍:
1. Stephen Ruffenach: Caspar Friedrich Wolff (1734-1794), The Embryo Project Encyclopedia,  (http://embryo.asu.edu/)
2. Professor Scott Gilbert (Ed.): A Conceptual History of Modern Embryology,  Maryland,1994, 8-21
3. വിശുദ്ധ ക്വുര്‍ആന്‍ 25:54
4. വിശുദ്ധ ക്വുര്‍ആന്‍ 77:20
5. വിശുദ്ധ ക്വുര്‍ആന്‍ 32:8
6. വിശുദ്ധ ക്വുര്‍ആന്‍ 24:63
7. താജല്‍ അറൂസ്:  https://archive.org
8. Edward William Lane: Arabic-English Lexicon,London,1863, Volume 4, Page1397
9. വിശുദ്ധ ക്വുര്‍ആന്‍ 75:37
10. സ്വഹീഹുമുസ്‌ലിം
11. താജല്‍ അറൂസ്
12. ലിസാനുല്‍ അറബ്
13. സ്വഹീഹുമുസ്‌ലിം, കിതാബുലുക്തത്
14. Lane’s Arabic-English Lexicon, Page 2810
15. വിശുദ്ധ ക്വുര്‍ആന്‍ 16: 4.
16. വിശുദ്ധ ക്വുര്‍ആന്‍ 18: 37.
17. വിശുദ്ധ ക്വുര്‍ആന്‍ 22: 5.
18. വിശുദ്ധ ക്വുര്‍ആന്‍ 35: 11.
19. വിശുദ്ധ ക്വുര്‍ആന്‍ 36: 77.
20. വിശുദ്ധ ക്വുര്‍ആന്‍ 40: 67.
21. വിശുദ്ധ ക്വുര്‍ആന്‍ 53: 45, 46.
22. വിശുദ്ധ ക്വുര്‍ആന്‍ 80:18, 19.
23. ഇമാംറാസി: ജാമിഉല്‍ബയാന്‍ ഫീ തഫ്‌സീറില്‍ ഖുര്‍ആന്‍ (http://www.altafsir. com/)
24. Arthur John Arberry: The Koran Interpreted, Page 352
25. N.J. Dawood:The Koran, Page 380
26. വിശുദ്ധ ക്വുര്‍ആന്‍ 53:3,4
വിഷയവുമായി ബന്ധപ്പെട്ട വീഡിയോ

തൊരു വൈജ്ഞാനിക മേഖലയിലേക്കും ക്വുര്‍ആനും ഹദീഥുകളും നല്‍കുന്ന വെളിച്ചത്തെക്കുറിച്ച് ചര്‍ച്ച ചെയ്യുമ്പോള്‍ നാം പ്രാഥമികമായി മനസ്സിരുത്തേണ്ട വസ്തുത, ശാസ്ത്രത്തെക്കുറിച്ചോ ഭൗതിക വിജ്ഞാനീയങ്ങളെകുറിച്ചോ അറിവു നല്‍കുന്നതിനുവേണ്ടി അവതരിപ്പിക്കപ്പെട്ട വെളിപാടുകളല്ല ഇവയെന്നുള്ളതാണ്. മനുഷ്യരുടെ ജീവിതവിജയത്തിനാവശ്യമായ മാര്‍ഗനിര്‍ദേശങ്ങള്‍ നല്‍കുകയും മരണാനന്തര ജീവിതത്തിലെ ശാശ്വത ശാന്തിയിലേക്ക് അവരെ നയിക്കുകയുമാണ് വെളിപാടുകള്‍ നിര്‍വഹിക്കുന്ന ധര്‍മം. പ്രസ്തുത ധര്‍മ നിര്‍വഹണത്തിനിടയില്‍, ചുറ്റുപാടുകളെയും തന്നെ തന്നെയും നിരീക്ഷിച്ചുകൊണ്ട് സര്‍വ്വലോക സ്രഷ്ടാവിന്റെ അനുഗ്രഹങ്ങളെയും മാര്‍ഗദര്‍ശനത്തിന്റെ അനിവാര്യതയെയും കുറിച്ച് സ്വയം ബോധ്യപ്പെടുത്തുവാന്‍ മനുഷ്യരോട് ആഹ്വാനം ചെയ്യുന്നതിനിടയിലാണ് ഭൗതിക വിജ്ഞാനീയങ്ങളിലേക്ക് പ്രധാനമായും ക്വുര്‍ആനും ഹദീഥുകളും വെളിച്ചം വീശുന്നത്. തലച്ചോറിന്റെ ഉപയോഗത്തിലൂടെ മനുഷ്യര്‍ നേടിയെടുക്കേണ്ട വിവരങ്ങളോ പ്രസ്തുത വിവരങ്ങളുടെ വെളിച്ചത്തില്‍ വികസിപ്പിച്ചെടുക്കേണ്ട സാങ്കേതികവിദ്യയെയോ കുറിച്ച് പഠിപ്പിക്കുകയല്ല, പ്രത്യുത തലച്ചോറിന് മാത്രമായി മനസ്സിലാക്കിയെടുക്കാനാവാത്ത യഥാര്‍ത്ഥമായ അറിവു നല്‍കുകയാണ് വെളിപാടുകളുടെ ധര്‍മം എന്നതുകൊണ്ടുതന്നെ ഭൗതിക വിജ്ഞാനീയങ്ങളുടെ ഏതെങ്കിലുമൊരു ശാഖയെക്കുറിച്ച പൂര്‍ണമായ വിവരങ്ങളോ വിവരണങ്ങളോ തേടി ക്വുര്‍ആനിലോ ഹദീഥുകളിലോ പരതുന്നത് വിഡ്ഢിത്തമാണ്.

മസ്തിഷ്‌കത്തിന് മനസ്സിലാക്കാനാവുന്ന വസ്തുതകളെ ചൂണ്ടിക്കാണിച്ച് അവയുടെ അപഗ്രഥനത്തിലൂടെ മനസ്സിലാക്കാനാവാത്ത ലോകത്തെക്കുറിച്ച് മനസ്സിലാക്കുവാന്‍ മനുഷ്യരോട് പറയുമ്പോള്‍, പ്രസ്തുത വസ്തുതകളെക്കുറിച്ച പരാമര്‍ശങ്ങളിലൊന്നും അബദ്ധങ്ങള്‍ കടന്നുവരുന്നില്ലെന്നതാണ് ഈ വെളിപാടുകളുടെ സവിശേഷത. എഴുതപ്പെട്ട കാലത്തെ അറിവില്ലായ്മയുടെ സ്വാധീനമില്ലാത്ത മതപരമോ മതേതരമോ ആയ ഗ്രന്ഥങ്ങളൊന്നുമില്ലെന്ന സ്വാഭാവികതയ്ക്ക് അപവാദമാണ് ക്വുര്‍ആനും സ്വഹീഹായ ഹദീഥുകളുമെന്ന വസ്തുത വ്യത്യസ്ത വൈജ്ഞാനിക മേഖലകളെക്കുറിച്ച് ഈ വെൡപാടുകളിലുള്ള പരാമര്‍ശങ്ങളെ ഇന്നു നിലനില്‍ക്കുന്ന തെളിയിക്കപ്പെട്ട യാഥാര്‍ത്ഥ്യങ്ങളുമായി താരതമ്യം ചെയ്താല്‍ സുതരാം ബോധ്യപ്പെടും. തെറ്റുപറ്റാത്തവനില്‍നിന്നുള്ളതാണ് ഈ വെളിപാടുകളെന്ന വസ്തുത വ്യക്തമാക്കുവാന്‍ ഇത്തരം താരതമ്യങ്ങള്‍ നിമിത്തമാകുമെന്നാണ് ഇവ്വിഷയകമായ ഇസ്‌ലാമിക പ്രബോധകരുടെ അവകാശവാദം.

മനുഷ്യരെ സ്വന്തത്തെപ്പറ്റി ചിന്തിക്കുവാന്‍ പ്രേരിപ്പിച്ചുകൊണ്ട് സ്രഷ്ടാവിന്റെ അസ്തിത്വത്തെയും പുനരുത്ഥാനത്തിന്റെ സത്യതയെയും കുറിച്ച് ബോധ്യപ്പെടുത്തുന്ന ക്വുര്‍ആന്‍ സൂക്തങ്ങളിലും താന്‍ പ്രവാചകനാണെന്നുള്ള യാഥാര്‍ത്ഥ്യം വ്യക്തമാക്കിക്കൊണ്ടുള്ള നബിവചനങ്ങളിലുമാണ് മനുഷ്യഭ്രൂണത്തിന്റെ ഉല്‍പത്തിയെയും പരിണാമത്തെയും കുറിച്ച പരാമര്‍ശങ്ങളിലധികവും കടന്നുവരുന്നത്. ക്വുര്‍ആനിലും സ്വഹീഹായ ഹദീഥുകളിലും പ്രതിപാദിക്കപ്പെട്ട ഭ്രൂണശാസ്ത്ര വസ്തുതകളെ ആധുനിക പഠനങ്ങള്‍ വെളിച്ചത്തുകൊണ്ടുവന്ന യാഥാര്‍ത്ഥ്യങ്ങളുമായി താരതമ്യം ചെയ്തു പഠിക്കുന്നവര്‍ക്കൊന്നും തന്നെ ഈ സ്രോതസുകളിലുള്ളത് ദൈവിക വെളിപാടാണെന്ന യാഥാര്‍ത്ഥ്യത്തെ നിഷേധിക്കാനാവുകയില്ല.

അതുകൊണ്ടാണല്ലോ, കാനഡയില്‍ ടൊറന്റോ സര്‍വകലാശാലയിലെ പ്രൊഫസറും അറിയപ്പെടുന്ന ഭ്രൂണശാസ്ത്രജ്ഞനും മെഡിക്കല്‍ കോളേജുകളില്‍ പഠിപ്പിക്കുന്ന ഗ്രന്ഥങ്ങളുടെ കര്‍ത്താവുമായ ഡോക്ടര്‍ കീത്ത് മൂര്‍ ഇങ്ങനെ പറഞ്ഞത്: ”മനുഷ്യ പ്രത്യുല്‍പാദനത്തെയും ഭ്രൂണവളര്‍ച്ചയെയും സംബന്ധിച്ച് വിവരിക്കുന്ന ക്വുര്‍ആനിലെയും സുന്നത്തിലെയും വചനങ്ങളെ വ്യാഖ്യാനിക്കുവാനായി സുഊദി അറേബ്യയിലെ ജിദ്ദ കിംഗ് അബ്ദുല്‍ അസീസ് സര്‍വകലാശാലയിലെ ഭ്രൂണശാസ്ത്ര സമിതിയെ സഹായിക്കുവാനും അവരോടൊപ്പം പ്രവര്‍ത്തിക്കുവാനും കഴിഞ്ഞ മൂന്ന് വര്‍ഷമായി എനിക്കു സാധിച്ചു. ഭ്രൂണശാസ്ത്രം തന്നെ സ്ഥാപിക്കപ്പെട്ടിട്ടില്ലാത്ത ക്രിസ്താബ്ദം ഏഴാം നൂറ്റാണ്ടില്‍ രേഖപ്പെടുത്തപ്പെട്ട കാര്യങ്ങളുടെ കൃത്യത കണ്ട് ആദ്യമേ തന്നെ അത്ഭുതപരതന്ത്രനായിതീര്‍ന്നു. ക്രിസ്താബ്ദം പത്താം നൂറ്റാണ്ടില്‍ ജീവിച്ച മുസ്്‌ലിം ശാസ്ത്രജ്ഞന്‍മാരുടെ മഹത്തായ ചരിത്രത്തെക്കുറിച്ചും രോഗശുശ്രൂഷാരംഗത്തെ അവരുടെ സംഭാവനകളെക്കുറിച്ചും എനിക്ക് അറിയാമായിരുന്നുവെങ്കിലും ക്വുര്‍ആനിലും സുന്നത്തിലുമടങ്ങിയിരിക്കുന്ന മതപരമായ കാര്യങ്ങളെപ്പറ്റി എനിക്ക് യാതൊരുവിധ അറിവുമുണ്ടായിരുന്നില്ല.”(L. Keith Moore and Abdul-Majeed al-Zindani: The Developing Human with Islamic Additions, Third Edition, Philadelphia, 1982.)

”മനുഷ്യവളര്‍ച്ചയെക്കുറിച്ച ക്വുര്‍ആന്‍ പരാമര്‍ശങ്ങളെ വ്യക്തമാക്കുവാനായി സഹായിക്കാനാവുകയെന്നത് എന്നെ സംബന്ധിച്ചിടത്തോളം ഏറെ സന്തോഷകരമാണ്. ക്വുര്‍ആനില്‍ പറഞ്ഞ ഈ വിജ്ഞാനങ്ങളില്‍ ഭൂരിഭാഗവും അതിന്റെ അവതരണത്തിന് ശേഷം നൂറ്റാണ്ടുകള്‍ കഴിഞ്ഞുമാത്രം കണ്ടുപിടിക്കപ്പെട്ടവയാണ് എന്നതുകൊണ്ടുതന്നെ അവ മുഹമ്മദിന് ദൈവത്തില്‍നിന്ന് അഥവാ അല്ലാഹുവില്‍നിന്ന് ലഭിച്ചതായിരിക്കുവാനേ നിര്‍വാഹമുള്ളു. മുഹമ്മദ് ദൈവത്തിന്റെ അഥവാ അല്ലാഹുവിന്റെ ദൂതന്‍ തന്നെയാണെന്ന കാര്യമാണ് ഇത് സമര്‍ത്ഥിക്കുന്നത്.”(Abdul-Majeed al-Zindani: This is the Truth (video tape).)

ഭ്രൂണത്തിന്റെ ഉല്‍പത്തിയെയും പരിണാമത്തെയും കുറിച്ച് ആധുനികശാസ്ത്രം നമുക്ക് നല്‍കുന്ന അറിവുകളുടെ വെളിച്ചത്തില്‍ ഈ പരാമര്‍ശങ്ങള്‍ പഠനവിധേയമാക്കുമ്പോള്‍ ഇതിലെ കൃത്യതയും സൂക്ഷ്മതയും ആരെയും ആശ്ചര്യഭരിതരാക്കുമെന്ന കാര്യത്തില്‍ സംശയമില്ല. ഏതെങ്കിലുമൊരു മനുഷ്യന്റെ തലച്ചോറിനകത്ത് രൂപീകരിക്കപ്പെട്ട ആശയങ്ങളുടെ സമാഹാരമാണ് ക്വുര്‍ആനെങ്കില്‍ മുഹമ്മദ് നബി (സ)യുടെ കാലത്ത് നിലനിന്നിരുന്ന അബദ്ധധാരണകളിലേതെങ്കിലും ക്വുര്‍ആനില്‍ ഉണ്ടാവേണ്ടിയിരുന്നു. അത്തരം അബദ്ധങ്ങളൊന്നുമില്ലെന്നു മാത്രമല്ല, ആധുനിക യന്ത്രങ്ങളുടെ സഹായത്തോടെ മാത്രം നാം മനസ്സിലാക്കിയ കാര്യങ്ങള്‍ പോലും വളരെ കൃത്യമായി ക്വുര്‍ആനിലും ഹദീഥുകളിലും പരാമര്‍ശിക്കപ്പെടുന്നുവെന്ന വസ്തുത എന്തുമാത്രം അത്ഭുതകരമല്ല! ആധുനികഭ്രൂണശാസ്ത്രത്തിന്റെ കണ്ണടയിലൂടെ ക്വുര്‍ആനിലും ഹദീഥുകളിലും പരാമര്‍ശിക്കപ്പെട്ട ഭ്രൂണഘട്ടങ്ങളെ നോക്കുന്ന സത്യസന്ധരായ ആര്‍ക്കും ഈ സ്രോതസുകളുടെ ദൈവികത നിഷേധിക്കാനാവില്ല. അതുകൊണ്ടാണല്ലോ ജീവിച്ചിരിക്കുന്നവരില്‍ ഏറ്റവും പ്രഗത്ഭനായ ഡോ. കീത്ത് മൂറിനെപ്പോലുള്ള ഒരു ഭ്രൂണശാസ്ത്രജ്ഞനുപോലും അത് സമ്മതിക്കേണ്ടിവന്നത്!

നുഷ്യസൃഷ്ടിപ്പിന്റെ വ്യത്യസ്ത ഘട്ടങ്ങളെക്കുറിച്ച് പ്രതിപാദിച്ചുകൊണ്ട് സ്രഷ്ടാവിനെ അറിയുവാനും പുനരുത്ഥാനത്തെ നിഷേധിക്കാതിരിക്കാനും മനുഷ്യരെ ഉദ്്‌ബോധിപ്പിക്കുന്ന ക്വുര്‍ആനിക വചനങ്ങള്‍ അനേകമുണ്ട്. ആധുനിക ഉപകരണങ്ങളുടെ സഹായത്തോടെ മാത്രം മനുഷ്യര്‍ക്ക് മനസ്സിലാക്കാനായ ഭ്രൂണോല്‍പത്തിയെയും പരിണാമത്തെയും കുറിച്ച പരാമര്‍ശങ്ങള്‍ നടത്തുമ്പോഴും ക്വുര്‍ആന്‍ പുലര്‍ത്തുന്ന കൃത്യതയും സൂക്ഷ്മതയും വിസ്മയാവഹമാണ്.

മുഹമ്മദ് നബി(സ)യുടെ മസ്തിഷ്‌കത്തില്‍ വിരിഞ്ഞ ചിന്തകളുടെ സമാഹാരമാണ് ക്വുര്‍ആന്‍ എന്ന് വാദിക്കുന്നവര്‍ക്ക് അതില്‍ പ്രതിപാദിക്കപ്പെട്ട സൂക്ഷ്മവും കൃത്യവുമായ പ്രസ്താവനകള്‍ തലവേദനയുണ്ടാക്കാറുണ്ട്. പതിനാലു നൂറ്റാണ്ടുകള്‍ക്കു മുമ്പു ജീവിച്ച ഒരു അറബിനിരക്ഷരന്റെ വായില്‍നിന്ന് ലോകം ശ്രവിച്ച ഒരു ഗ്രന്ഥത്തില്‍ ഇത്ര കൃത്യമായ പരാമര്‍ശങ്ങളുണ്ടാകുന്നതെങ്ങനെയെന്ന ചോദ്യത്തിന് ഉത്തരം പറയുവാന്‍ അവര്‍ ഏറെ പ്രയാസപ്പെടാറുമുണ്ട്. പ്രസ്തുത തലവേദനയില്‍നിന്നുള്ള മോചനത്തിന് ക്വുര്‍ആന്‍ വിമര്‍ശകര്‍ കണ്ടെത്തിയ പരിഹാരമാണ് മുഹമ്മദ് നബി(സ)ക്കു മുമ്പുതന്നെ ലോകത്തെങ്ങും ഭ്രൂണവിജ്ഞാനീയങ്ങള്‍ നിലനിന്നിരുന്നുവെന്നും പ്രസ്തുത വിജ്ഞാനീയങ്ങളില്‍നിന്ന് കടമെടുത്തതാണ് ക്വുര്‍ആനിലും ഹദീഥുകളിലുമുള്ള ഭ്രൂണശാസ്ത്ര പരാമര്‍ശങ്ങളെന്നുമുള്ള വാദം. ക്വുര്‍ആനിന്റെ അവതരണ കാലത്ത് ലോകത്തിന്റെ വിവിധ വശങ്ങളില്‍ നിലനിന്നിരുന്ന ഭ്രൂണവിജ്ഞാനീയങ്ങളെക്കുറിച്ച പഠനം ഈ വാദത്തിന്റെ നട്ടെല്ലൊടിക്കുമെന്നതാണ് സത്യം

ഭാരതീയ സങ്കൽപം:

മനുഷ്യഭ്രൂണത്തിന്റെ പരിണാമദശകളെക്കുറിച്ച് പ്രതിപാദിക്കുന്ന ലഭ്യമായതില്‍വെച്ച് ഏറ്റവും പുരാതനമായ ഗ്രന്ഥം കൃഷ്ണയജുര്‍വേദത്തിന്റെ സംന്യാസ ഉപനിഷത്തായി ഗണിക്കപ്പെടുന്ന ഗര്‍ഭോപനിഷത്താണ്. ക്രിസ്തുവിന് ആയിരത്തിനാനൂറ് വര്‍ഷങ്ങള്‍ക്കുമുമ്പ് പിപ്പലാദ ഋഷി രചിച്ചതായി  കരുതപ്പെടുന്ന ഗര്‍ഭോപനിഷത്തില്‍ ശുക്ലത്തിന്റെ ആവിര്‍ഭാവത്തെയും ഭ്രൂണനിര്‍മാണത്തില്‍ പെണ്‍ശരീരത്തിന്റെ സംഭാവനയെയും ഗര്‍ഭാവസ്ഥയിലെ ഭ്രൂണമാറ്റങ്ങളെയും കുറിച്ച വിശദമായ പരാമര്‍ശങ്ങളുണ്ട്. ഗര്‍ഭോപനിഷത്തിലെ രണ്ടും മൂന്നും വചനങ്ങളിലുള്ള ആശയങ്ങളെ ഇങ്ങനെ സംഗ്രഹിക്കാം:

1. വിവിധങ്ങളായ പദാര്‍ത്ഥങ്ങളില്‍നിന്ന് ഉല്‍ഭൂതമാകുന്ന രസത്തില്‍നിന്ന് രക്തവും അതില്‍നിന്ന് മാംസവും അതില്‍നിന്ന് മേദസ്സും അതില്‍നിന്ന് സ്‌നായുവും അതില്‍നിന്ന് അസ്ഥിയും അതില്‍നിന്ന് മജ്ജയുമുണ്ടാകുന്നു. മജ്ജയില്‍നിന്നാണ് ശുക്ലമുണ്ടാകുന്നത്.

2. പുരുഷന്റെ ശുക്ലവും സ്്ത്രീയുടെ ആര്‍ത്തവരക്തവും (ശോണിതം) ചേര്‍ന്നാണ് ഗര്‍ഭമുണ്ടാകുന്നത്.

3. ഋതുസമയത്ത് ഉചിതമായ രീതിയില്‍ ഗര്‍ഭധാരണം നടന്നാല്‍ ശുക്ലവും ശോണിതവും ചേര്‍ന്ന് ഒരൊറ്റ രാത്രികൊണ്ട് കലലമായിത്തീരും.

4. ഗര്‍ഭസ്ഥശിശു ഏഴു രാത്രികള്‍കൊണ്ട് ബുദ്ബുദവും പതിനഞ്ച് ദിവസങ്ങള്‍കൊണ്ട് പിണ്ഡാകാരവും ഒരു മാസം കൊണ്ട് കഠിനവുമായിത്തീരുന്നു.

5. രണ്ടുമാസം കൊണ്ട് തലയും മൂന്ന് മാസം കൊണ്ട് കാലും നാലാം മാസം മുട്ടുകളും അരയും വയറും അഞ്ചാം മാസം പൃഷ്ഠം, നട്ടെല്ല് എന്നിവയും രൂപപ്പെടുന്നു.

6. ആറാം മാസത്തിലാണ് വായ്, മൂക്ക്, ചെവി, കണ്ണുകള്‍ എന്നിവ രൂപപ്പെടുന്നത്.

7. ഏഴാം മാസത്തില്‍ കുഞ്ഞ് ജീവസ്പന്ദത്തോടെ പുഷ്ടിപ്പെടുന്നു.

8. പരിപൂര്‍ണ ശരീരം രൂപം പ്രാപിക്കുന്നത് എട്ടാം മാസത്തിലാണ്.

9. ശുക്ലമാണ് അധികമെങ്കില്‍ ആണ്‍കുഞ്ഞും ആര്‍ത്തവരക്തമാണ് അധികമെങ്കില്‍ പെണ്‍കുഞ്ഞും രണ്ടും തുല്യമാണെങ്കില്‍ നപുംസകവുമാണുണ്ടാവുക.

10. സ്ത്രീപുരുഷ സംയോഗം നടക്കുന്ന സന്ദര്‍ഭത്തിലെ ദമ്പതിമാരുടെ മനോവ്യഥ കാരണമാണ് കുഞ്ഞിന് അംഗവൈകല്യങ്ങളുണ്ടാകുന്നത്.

11. വായുവിനാല്‍ ശുക്ല-ശോണിതങ്ങള്‍ ഭേദിക്കപ്പെടുന്നതിനാലാണ് ഇരട്ടകളും മറ്റുമുണ്ടാകുന്നത്.

12. ജ്ഞാനേന്ദ്രിയാദികള്‍ ചേര്‍ന്ന് ശിശു പൂര്‍ണത പ്രാപിക്കുന്നത് ഒമ്പതാം മാസത്തിലാണ്.

ഇസ്രായീല്യരുടെ സങ്കല്‍പം:

ഭാഷാശാസ്ത്ര പ്രകാരം പരിശോധിച്ചാല്‍ എഴുതപ്പെട്ടതില്‍ വെച്ച് ഏറ്റവും പുരാതനമായ ബൈബിള്‍ പുസ്തകമെന്ന് മനസ്സിലാക്കാനാകുന്ന ഇയ്യോബിന്റെ പുസ്തകത്തില്‍ തന്നെ ഭ്രൂണോല്‍പാദനത്തെക്കുറിച്ച പരിമിതമായ ചില പരാമര്‍ശങ്ങളുണ്ട്. അതിങ്ങനെ വായിക്കാം:

”കളിമണ്ണുകൊണ്ടാണ് അങ്ങ് എന്നെ സൃഷ്ടിച്ചതെന്ന് അനുസ്മരിക്കേണമേ! പൊടിയിലേക്കുതന്നെ അങ്ങ് എന്നെ തിരിച്ചയക്കുമോ? അങ്ങ് എന്നെ പാലുപോലെ പകര്‍ന്ന് തൈരുപോലെ ഉറ കൂട്ടിയില്ലേ? അങ്ങ് ചര്‍മവും മാംസവും കൊണ്ട് എന്നെ ആവരണം ചെയ്തു; അസ്ഥിയും സ്‌നായുക്കളുംകൊണ്ട് എന്നെ തുന്നിച്ചേര്‍ത്തു.”( ഇയ്യോബ് 10: 9-11).

മനുഷ്യരെ പാല്‍ പോലെ പകരുകയും തൈരുപോലെ ഉറ കൂടിപ്പിക്കുകയും അതിനുശേഷം ത്വക്കും മാംസവും ധരിപ്പിക്കുകയും അസ്ഥിയും ഞരമ്പും കൊണ്ട് മെടയുകയും ചെയ്തവനായി സ്രഷ്ടാവിനെ അഭിസംബോധന ചെയ്യുന്ന ഇയ്യോബിന്റെ വരികള്‍ ഭ്രൂണോല്‍പത്തിയെക്കുറിച്ച് അന്ന് നിലനിന്നിരുന്ന സങ്കല്‍പമെന്തായിരുന്നുവെന്ന് മനസ്സിലാക്കിത്തരുന്നുണ്ട്. പുരുഷ ശുക്ലം കട്ട പിടിച്ചാണ് കുഞ്ഞുണ്ടാവുന്നതെന്നും അതില്‍നിന്നാണ് അവയവങ്ങള്‍ രൂപീകരിക്കപ്പെടുന്നതെന്നുമായിരുന്നു അന്നു നിലനിന്നിരുന്ന സങ്കല്‍പം. ഇയ്യോബില്‍നിന്ന് വ്യത്യസ്തമായ വീക്ഷണങ്ങളെന്തെങ്കിലും ഔദ്യോഗിക കാനോനില്‍ ഉള്‍പ്പെടുത്തപ്പെട്ട ബൈബിള്‍ ഗ്രന്ഥങ്ങളുടെ കര്‍ത്താക്കള്‍ക്ക് ഉണ്ടെന്ന് വ്യക്തമാക്കുന്ന പരാമര്‍ശങ്ങളൊന്നും തന്നെ ബൈബിള്‍ പുസ്തകങ്ങളിലില്ല.

പുരുഷബീജത്തില്‍ നിന്നാണ് കുഞ്ഞുണ്ടാവുന്നതെങ്കിലും അത് പുഷ്ടി പ്രാപിക്കുന്നത് മാതാവിന്റെ ആര്‍ത്തവരക്തത്താലാണെന്നായിരുന്നു ഇസ്രായീല്യരുടെ വിശ്വാസമെന്ന് വ്യക്തമാക്കുന്ന ഒരു പരാമര്‍ശം കത്തോലിക്കാ ബൈബിളിലുണ്ട്. യഹൂദന്‍മാരുടെ കാനോനിലോ പ്രൊട്ടസ്റ്റന്റ് ബൈബിളിലോ ഉള്‍പ്പെടുത്തിയിട്ടില്ലാത്ത, കത്തോലിക്കര്‍ ഉത്തര കാനോനിക ഗ്രന്ഥങ്ങളെന്ന് വിളിച്ച് അവരുടെ ബൈബിളിന്റെ ഭാഗമായി ആദരിക്കുന്ന സോളമന്റെ വിജ്ഞാനത്തിലാണ് പ്രസ്തുത പരാമര്‍ശമുള്ളത്. അതിങ്ങനെ വായിക്കാം: ”ദാമ്പത്യത്തിന്റെ ആനന്ദത്തില്‍ പുരുഷബീജത്തില്‍ നിന്ന് ജീവന്‍ ലഭിച്ചു, പത്തു മാസം കൊണ്ട് അമ്മയുടെ രക്തത്താല്‍ പുഷ്ടി പ്രാപിച്ചു.”( (വിജ്ഞാനം 7: 2) ഇവയിൽ നിന്നും അന്നത്തെ യഹൂദറബ്ബിമാരുടെ തൽമുദിൽ നിന്നുമായി ഇസ്രാഈല്യർക്കിടയിൽ നിലനിന്നിരുന്ന ഭ്രൂണഘട്ടങ്ങളെക്കുറിച്ച വിവരങ്ങൾ നമുക്ക് ലഭിക്കും. അവയെ ഇങ്ങനെ സംക്ഷേപിക്കാം: 1. പുരുഷ ശുക്ലം ഗര്‍ഭാശയത്തിലെത്തിയ ശേഷം പാലില്‍നിന്ന് തൈരുണ്ടാകുന്നതുപോലെ കട്ടിയായിത്തീര്‍ന്നാണ് ഭ്രൂണമുണ്ടാകുന്നത്.

2. ശുക്ലത്തില്‍നിന്നുണ്ടാകുന്ന ഭ്രൂണം വളരുമ്പോള്‍ ആര്‍ത്തവരക്തം അതിനെ പരിപോഷിപ്പിക്കുന്നു.

3. പിതാവില്‍നിന്നുണ്ടാകുന്ന വെളുത്ത ശുക്ലത്തില്‍നിന്നാണ് അസ്ഥികള്‍, സ്‌നായുക്കള്‍, നഖങ്ങള്‍, മസ്തിഷ്‌കം എന്നിവയുണ്ടാകുന്നത്.

4. മാതാവില്‍നിന്നുള്ള ചുവന്ന ശുക്ലത്തില്‍നിന്നാണ് തൊലി, മാംസം, മുടി, രക്തം, കണ്ണിന്റെ കറുപ്പ് എന്നിവയുണ്ടാകുന്നത്.

5. ദൈവദത്തമായ ആത്മാവിന്റെ പ്രവര്‍ത്തനഫലമായാണ് ജീവനും ജൈവപ്രതിഭാസങ്ങളുമുണ്ടാകുന്നത്.

ഹിപ്പോക്രാറ്റസിന്റെ സങ്കല്‍പം:

പാശ്ചാത്യ ചികിത്സാ ശാസ്ത്രത്തിന്റെ പിതാവായി അറിയപ്പെടുന്ന ഹിപ്പോക്രാറ്റസാണ് (460 BC-377 BC) ഭ്രൂണത്തിന്റെ രൂപീകരണത്തെയും പരിണാമത്തെയും കുറിച്ച വീക്ഷണങ്ങളവതരിപ്പിച്ച ഗ്രീക്കുകാരില്‍ പ്രഥമ ഗണനീയന്‍. ഹിപ്പോക്രാറ്റസ് തന്റെ ഭ്രൂണശാസ്ത്ര വീക്ഷണങ്ങള്‍ അവതരിപ്പിച്ചിരിക്കുന്നത് ചികിത്സാക്രമം (Regimen), വിത്ത് (The Seed), കുഞ്ഞിന്റെ പ്രകൃതി (The Nature of the Child) എന്നീ രചനകളിലാണ്. ഈ കൃതികളിലൂടെ ഹിപ്പോക്രാറ്റസ് അവതരിപ്പിച്ച മനുഷ്യോല്‍പത്തി സങ്കല്‍പങ്ങളെ ഇങ്ങനെ സംഗ്രഹിക്കാം.

1. എല്ലാ പ്രാപഞ്ചിക പ്രതിഭാസങ്ങളും അഗ്നിയും ജലവും ചേര്‍ന്നാണ് ഉണ്ടാകുന്നത്. ഇതില്‍ അഗ്നിയാണ് ഭ്രൂണത്തിന് കാരണമായിത്തീരുന്നത്.

2. മാതാവും പിതാവും ശുക്ലം ഉല്‍പാദിപ്പിക്കുന്നുണ്ട്. ഗര്‍ഭധാരണത്തിന്റെ സമയത്തല്ലാതെ പിതാവും മാതാവും വിസര്‍ജിക്കാത്ത ശുക്ലങ്ങള്‍ മാതാവിന്റെ ജനനേന്ദ്രിയത്തില്‍നിന്ന് പുറംതള്ളപ്പെടുകയാണ് ചെയ്യുന്നത്. ഗര്‍ഭധാരണ കാലത്തെ ഈര്‍പ്പത്താല്‍ ഗര്‍ഭാശയരന്ധ്രം ചുരുങ്ങുന്നതിനാല്‍ രണ്ടുപേരുടെയും ശുക്ലങ്ങള്‍ അവിടെ തങ്ങുകയും കൂടിച്ചേരുകയും ചെയ്യുന്നു.

3. ശരീരത്തിന്റെ വ്യത്യസ്ത ഭാഗങ്ങളില്‍നിന്നാണ് ശുക്ലമുണ്ടാകുന്നത്. ശക്തമായ ശാരീരികാവയവങ്ങളില്‍നിന്ന് ബലമുള്ള ബീജവും അശക്തമായ ശാരീരികാവയവങ്ങളില്‍നിന്ന് അബല ബീജവുമുണ്ടാകുന്നു.

4. രണ്ടുപേരുടെയും ശുക്ലത്തില്‍ സ്ത്രീബീജവും പുരുഷബീജവും ഉണ്ടായിരിക്കും.

5. ബലബീജവും അബല ബീജവും എങ്ങനെ യോജിക്കുന്നുവെന്നതിനനുസരിച്ചായിരിക്കും കുഞ്ഞിന്റെ ലിംഗം നിര്‍ണയിക്കപ്പെടുക. സ്ത്രീയും പുരുഷനും ഉല്‍പാദിപ്പിക്കുന്നത് ശക്തമായ ബീജങ്ങളാണെങ്കില്‍ അവ കൂടിച്ചേര്‍ന്നുണ്ടാകുന്നത് ആണ്‍കുഞ്ഞും അശക്തങ്ങളായ ബീജങ്ങളാണെങ്കില്‍ അവ കൂടിച്ചേര്‍ന്നുണ്ടാകുന്നത് പെണ്‍കുഞ്ഞുമായിരിക്കും.

6. ഒരാള്‍ ശക്തമായ ബീജവും മറ്റേയാള്‍ അശക്തമായ ബീജവുമാണ് ഉല്‍പാദിപ്പിക്കുന്നതെങ്കില്‍ ഏത് ബീജമാണോ കൂടുതലുള്ളത് അതിനനുസരിച്ചാണ് കുഞ്ഞിന്റെ ലിംഗം നിര്‍ണയിക്കപ്പെടുക.

7. ബീജങ്ങള്‍ കൂടിച്ചേര്‍ന്നുണ്ടാകുന്ന ഭ്രൂണത്തിലേക്ക് മാതൃരക്തമെത്തി അത് കട്ടപിടിച്ചുകൊണ്ടാണ് ഗര്‍ഭസ്ഥശിശുവിന്റെ ശരീരത്തില്‍ മാംസം വളരുന്നത്.

8. മാംസം വളരുന്നതനുസരിച്ച് അവയവങ്ങളെല്ലാം ഒന്നിച്ചാണ് രൂപീകരിക്കപ്പെടുന്നത്.

9. ആണ്‍കുഞ്ഞിന്റെ അവയവങ്ങള്‍ വളരാന്‍ മുപ്പതുദിവസങ്ങള്‍ മതി. പെണ്‍കുഞ്ഞിന്റെ അവയവവളര്‍ച്ചയ്ക്ക് പരമാവധി നാല്‍പത്തിരണ്ടു ദിവസങ്ങളാണ് വേണ്ടത്.

അരിസ്റ്റോട്ടിലിന്റെ സങ്കൽപം:

  ഹിപ്പോക്രാറ്റസിന് ശേഷം ശ്രദ്ധേയമായ ഭ്രൂണപരിണാമപരാമര്‍ശങ്ങള്‍ നടത്തിയ ഗ്രീക്കുകാരന്‍ തത്ത്വജ്ഞാനിയും ശാസ്ത്രജ്ഞനുമായിരുന്ന അരിസ്റ്റോട്ടിലാണ് (384-322 BC).താനെഴുതിയ നാനൂറിലധികം പുസ്തകങ്ങളില്‍ ഭ്രൂണത്തിന്റെ ഉല്‍പത്തിയെയും പരിവര്‍ത്തനങ്ങളെയും കുറിച്ച് അരിസ്റ്റോട്ടില്‍ വിവരിക്കുന്നത് ജന്തുക്കളുടെ ഉല്‍പത്തിയെപ്പറ്റി (On the Generation of Animals) എന്ന ഗ്രന്ഥത്തിലാണ്.

അതിൽ പറഞ്ഞിരിക്കുന്ന അരിസ്റ്റോട്ടിലിന്റെ വീക്ഷണങ്ങളെ ഇങ്ങനെ സംഗ്രഹിക്കാം:

1. പാല്‍ ഉറ കൂട്ടാന്‍ വേണ്ടി റെന്നറ്റ് ഉപയോഗിക്കപ്പെടുന്നതുപോലെ മാതൃശരീരത്തില്‍നിന്നുള്ള സ്രവങ്ങളെ കട്ടിയാക്കുകയാണ് പുരുഷശരീരത്തില്‍നിന്ന് സ്രവിക്കപ്പെടുന്ന ശുക്ലത്തിന്റെ ധര്‍മം. അതിനാല്‍ മാതൃസ്രവമാണ് യഥാര്‍ഥത്തില്‍ അടിസ്ഥാന വസ്തു. പാലിനെ കട്ടിയാക്കുകയെന്നതില്‍ കവിഞ്ഞ ധര്‍മമൊന്നും തന്നെ ഉറ കൂട്ടുന്ന പ്രക്രിയയില്‍ റെന്നറ്റിന് ഇല്ലാത്തതുപോലെ മാതൃരക്തത്തെ കട്ടിയാക്കുക മാത്രമാണ് ശുക്ലം ചെയ്യുന്നത്.

2. ഇങ്ങനെ കട്ടിയായിത്തീര്‍ന്ന് ഉറ കൂടിയ മാതൃരക്തത്തില്‍നിന്ന് രൂപപ്പെടുന്ന ഭ്രൂണം മണ്ണില്‍ നട്ടുപിടിപ്പിച്ച വിത്ത് വളരുന്നതുപോലെ മെല്ലെ വളര്‍ന്നുവരികയാണ് ചെയ്യുന്നത്.

3. ഗര്‍ഭസ്ഥ ശിശുവിന്റെ ശരീരത്തില്‍ ആദ്യമായി പ്രവര്‍ത്തനമാരംഭിക്കുന്നത് ഹൃദയമാണ്.

4. ഹൃദയത്തില്‍ നിന്നാരംഭിക്കുന്ന രക്തധമനികള്‍ ശരീരത്തിലാകമാനം വിന്യസിക്കപ്പെടുന്നു.

5. ശരീരാവയവങ്ങളുടെ രൂപീകരണം നടക്കുന്നത് താപത്തിന്റെയും തണുപ്പിന്റെയും പ്രവര്‍ത്തനഫലമായിട്ടാണ്. ചില അവയവങ്ങള്‍ ഘനീഭവിക്കപ്പെടുന്നത് താപത്താലാണെങ്കില്‍ മറ്റ് ചിലവയുണ്ടാകുന്നത് തണുപ്പിനാലാണ്.

6. പോഷകാഹാരം രക്തത്തിലൂടെ ശരീരാവയവങ്ങളിലേക്ക് ഒലിച്ചിറങ്ങുമ്പോഴാണ് അതിന്റെ ഒപ്പം തന്നെയുള്ള മാംസം രൂപീകരിക്കപ്പെടുന്നത്. മാംസം രൂപീകരിക്കപ്പെടുന്നതോടെ തണുപ്പ് അതിനെ ഘനീഭവിപ്പിക്കുന്നു.

7. ആന്തരികതാപത്താല്‍ ദ്രാവകങ്ങള്‍ ദൃഢമായിത്തീരുന്നതിനാലാണ് എല്ലുകളും സ്‌നായുക്കളുമെല്ലാം ഉണ്ടാകുന്നത്. അതുകൊണ്ടാണ് ഇവയെ തീ കൊണ്ട് വിഘടിപ്പിച്ച് നശിപ്പിക്കാന്‍ കഴിയാത്തത്.

8. നാം ഭക്ഷണപദാര്‍ത്ഥങ്ങള്‍ ചൂടാക്കിയും തണുപ്പിച്ചുമെല്ലാം നിര്‍മിച്ചെടുക്കുന്നതുപോലെ തന്നെ ഗര്‍ഭാശയത്തിനകത്തെ ചൂടും തണുപ്പും നമ്മുടെ അവയവങ്ങളെയെല്ലാം ചുട്ടെടുക്കുകയാണ് ചെയ്യുന്നത്.

9.  ചൂടാക്കിയ ദ്രാവകങ്ങള്‍ക്കു മുകളിലുണ്ടാവുന്ന പാടയെപ്പോലെയാണ് മാംസത്തിന് പുറത്തെ പാടയായി തൊലിയുണ്ടാകുന്നത്.

10. എല്ലാവിധ ഇന്ദ്രിയാനുഭവങ്ങളുടെയും കേന്ദ്രമായ ഹൃദയം രൂപീകരിക്കപ്പെട്ടശേഷം അതിന്റെ താപഫലമായിട്ടാണ് മസ്തിഷ്‌കം രൂപപ്പെടുന്നത്. മറ്റു ശാരീരികാവയവങ്ങള്‍ ഉണ്ടാകുന്നതിന് മുമ്പുതന്നെ മസ്തിഷ്‌കത്തെ പൊതിഞ്ഞുകൊണ്ടുള്ള ശാരീരികവ്യവസ്ഥകള്‍ രൂപപ്പെടുന്നത് അതുകൊണ്ടാണ്.

  ഗാലന്റെ സങ്കൽപം

ഹിപ്പോക്രാറ്റസിന്റെയും അരിസ്‌റ്റോട്ടിലിന്റെയുമെല്ലാം ഭ്രൂണപരിണാമത്തെക്കുറിച്ച വീക്ഷണങ്ങളെ അപഗ്രഥിച്ചുകൊണ്ട് തന്റേതായ ഭ്രൂണശാസ്ത്ര കാഴ്ചപ്പാടുകള്‍ അവതരിപ്പിച്ച ശ്രദ്ധേയനായ ഗ്രീക്ക് തത്ത്വജ്ഞാനിയാണ് ഗാലന്‍ എന്നറിയപ്പെടുന്ന ക്ലോഡിയസ് ഗാലെനെസ് (ക്രി.വ 129-200). വിവിധ ശാസ്ത്രവിഷയങ്ങളെപ്പറ്റി പ്രതിപാദിക്കുന്ന അഞ്ഞൂറോളം വരുന്ന പ്രബന്ധങ്ങളില്‍ ‘ശുക്ലത്തെപ്പറ്റി’ (On semen), ‘പ്രകൃതി പ്രഭാവങ്ങളെപ്പറ്റി’ (On the Natural Faculties), ‘ഗര്‍ഭസ്ഥ ശിശുവിന്റെ രൂപീകരണത്തെപ്പറ്റി’ (On the Formation of the Foetus) എന്നീ മൂന്നെണ്ണത്തിലാണ് തന്റെ ഭ്രൂണപരിണാമവീക്ഷണങ്ങള്‍ അദ്ദേഹം അവതരിപ്പിക്കുന്നത്.

ഇവയിലുള്ള ഗാലന്റെ ഭ്രൂണശാസ്ത്ര വീക്ഷണങ്ങളെ ഇങ്ങനെ സംഗ്രഹിക്കാം.

1. ഗര്‍ഭാശയത്തിലെത്തുന്ന ശുക്ലമാണ് ജന്തുവിന്റെ രൂപീകരണത്തിന് നിമിത്തമാകുന്ന സജീവമായ തത്ത്വം.

2. ആറാം ദിവസം ശുക്ലം ഇല്ലാതാവുകയും പകരം ഭ്രൂണം വളരാനാരംഭിക്കുകയും ചെയ്യും. ഭ്രൂണവളര്‍ച്ച നടക്കുന്നത് മാതൃശരീരത്തില്‍ രൂപപ്പെടുന്ന ആര്‍ത്തവരക്തം ശുക്ലത്തെ പോഷിപ്പിക്കുമ്പോഴാണ്.

3. ശുക്ലത്തെ രക്തം പരിപോഷിപ്പിക്കുമ്പോള്‍ അത് ഒരു മാംസപിണ്ഡമായിത്തീരുന്നു. ഹൃദയമോ കരളോ മസ്തിഷ്‌കമോ ഇല്ലാതെതന്നെ ഈ മാംസപിണ്ഡം നിലനില്‍ക്കുകയും വളരുകയും ചെയ്യുന്നു.

4. ശരീരത്തെ ഭരിക്കുന്ന മൂന്ന് അവയവങ്ങള്‍ ഒരു നിഴല്‍ ചിത്രത്തിലെന്നവണ്ണം പ്രത്യക്ഷപ്പെടുന്ന ഘട്ടമാണ് അടുത്തത്. ഈ അവയവങ്ങള്‍ കൂടുതല്‍ വ്യക്തതയോടെ ദൃശ്യമാകുന്നതോടൊപ്പം തന്നെ വയറിന്റെ ഭാഗങ്ങളും മെല്ലെ പ്രത്യക്ഷപ്പെടാനാരംഭിക്കുന്നു.

5. ഇതിനുശേഷം ശാരീരികാവയവങ്ങള്‍ വേര്‍പിരിയുകയും കൃത്യമായി കാണാനാവുന്ന അവസ്ഥ സംജാതമാവുകയും ചെയ്യുന്നു. 6. പിന്നീട് ശരീരത്തിലെ കൊമ്പുകളും ചില്ലകളുമെല്ലാം പ്രത്യക്ഷീഭവിക്കുന്നു.

 

ക്വുര്‍ആനിന്റെ അവതരണ കാലത്ത് ലോകത്തെങ്ങും നിലനിന്നിരുന്ന ഭ്രൂണ വിജ്ഞാനീയങ്ങളുടെ ഒരു ഏകദേശചിത്രമാണിത്. ഇതില്‍ ഹിപ്പോക്രാറ്റസിന്റെയും അരിസ്‌റ്റോട്ടിലിന്റെയും ഗാലന്റെയുമെല്ലാം ഭ്രൂണപരിണാമത്തെക്കുറിച്ച വീക്ഷണങ്ങളാണ് നവോത്ഥാനകാലം വരെ യൂറോപ്പില്‍ നിലനിന്നിരുന്നത്.   ഈ ധാരണകളില്ലെല്ലാം നിരവധി അബദ്ധങ്ങളുണ്ടെന്ന് ഇന്ന് നമുക്കറിയാം. എന്നാല്‍ ഈ അബദ്ധധാരണകള്‍ നിലനിന്നിരുന്ന കാലത്ത് അവതരിപ്പിക്കപ്പെട്ട ക്വുര്‍ആന്‍സൂക്തങ്ങളില്‍ ഈ ധാരണകളുടെ സൂക്ഷ്മമായ സ്വാധീനം പോലുമില്ല. അബദ്ധധാരണകള്‍ മാത്രം നിലനിന്നിരുന്ന ഭ്രൂണവിജ്ഞാനീയത്തെകുറിച്ച് പരാമര്‍ശിക്കുമ്പോള്‍ പ്രസ്തുത ധാരണകളുടെ യാതൊരു സ്വാധീനവുമില്ലാതെ കാര്യങ്ങള്‍ വിശദീകരിക്കുന്നത് തന്നെ അത്ഭുമാണ്. ക്വുര്‍ആനാകട്ടെ അബദ്ധധാരണകളുടെ സ്വാധീനമൊന്നുമില്ലാതെ കാര്യങ്ങള്‍ വ്യക്തമാക്കുക മാത്രമല്ല, പ്രത്യുത ആധുനികശാസ്ത്രവിദ്യകളുടെ സഹായത്താല്‍ മാത്രം നാം കണ്ടെത്തിയ കാര്യങ്ങള്‍ വളരെ കൃത്യമായി പരാമര്‍ശിക്കുക കൂടി ചെയ്യുന്നു. ഇത് ക്വുര്‍ആനിനെ അത്ഭുതങ്ങളുടെ അത്ഭുതമാക്കിത്തീര്‍ക്കുകയാണ് ചെയ്യുന്നത്. സര്‍വജ്ഞനായ അല്ലാഹുവിനല്ലാതെ ഇത്ര കൃത്യമായ പ്രസ്താവനകള്‍ നടത്താന്‍ കഴിയില്ല. അവതരണകാലത്ത് നിലനിന്നിരുന്ന അബദ്ധധാരണകളുടെ സ്വാധീനമില്ലാതെയും ആധുനികശാസ്ത്രം സ്ഥിരീകരിച്ച വസ്തുതകളിലേക്ക് വെളിച്ചംവീശിയും ക്വുര്‍ആന്‍ അതിന്റെ അപ്രമാദിത്വം വെളിപ്പെടുത്തുകയും ദൈവികത പ്രഖ്യാപിക്കുകയുമാണ് ചെയ്യുന്നത്

വിഷയവുമായി ബന്ധപ്പെട്ട വീഡിയോ